NURS 6540 FINAL EXAM LATEST 2023-2024/NURS 6540 WEEK 11 FINAL EXAM LATEST EXAM COMPLETE QUESTIONS AND CORRECT ANSWERS|ALREADY GRADED A +(WALDEN)

What are the physiological changes of aging

  1. Reduced physiological reserve of most body systems
    2.Reduced homeostatic mechanisms that fail to adjust
  2. Changes in the sympathetic response
  3. Impaired immunological function- infection risk is greater and autoimmune diseases are more prevalent.

What factors influence lab value interpretation in the older adult

  1. physiological changes with aging
  2. the prevalence of chronic disease
  3. changes in nutritional and fluid intake
  4. Lifestyle

What lab values is important
Creatinine clearance, GFR and renal function

What are examples of drugs are eliminated renally
Digoxin, H2 blockers, lithium and water-soluable antibiotics

What are the two formulas to evaluate the kidney function
The Cockcroft-Gault formula and the Modification of Diet in Renal Disease (MDRD)

What neurological conditions commonly have an initial onset in older age?
Parkinson’s, dementia, stroke, and complex partial seizures

What are geriatric syndromes
Diseases that involve multiple systems and have more than one underlying cause

What are the geriatric syndrome giants currently
Frailty, anorexia of aging, sarcopenia and cognitive impariment

What are bimodal presentations of disease
Diseases that can present in a younger age and an older age.

What are examples of bimodal diseases?
Psoriasis, Inflammatory bowel disease, Ulcerative Colitis, Crohn’s disease, Hodgkin’s lymphoma, and myasthenia gravis

What is primary prevention?
Activities to prevent the occurrence of a disease or adverse, including mental health

What is secondary prevention
Tasks directed towards detection of a disease or adverse health condition in an asymptomatic individual who has risk factors

What is tertiary prevention
The management of existing conditions to prevent disability and minimize complications to prevent disability for optimal function

What is the key to increasing geriatric physical activity
motivation

What is the nutrition screening tool for community dwelling older adults
Mini-Nutritional Assessment (MNA)

What factors affect maintaining a healthy balanced diet in geriatrics?

  1. Chronic illness and disability-related to ADLs like shopping and meal preparation,
  2. Financial hardships-limit food choices
  3. Prescribed medications-affects absorption of nutrients, sense of taste or appetite
  4. Depression or social isolation
  5. Obesity

What is a big influence of geriatric functionality and quality of life?
Injury prevention

What are important 6 areas of geriatric safety prevention

  1. Safe storage and removal of firearms
  2. Fall prevention
  3. Wearing seatbelts
  4. no drinking and driving
  5. Having a working smoke detector
  6. Keeping hot water set below 120 degrees

Periodic screening of gums and mouth can screen for what?

  1. Oral cancer
  2. ill-fitting or poorly functioning dentures
  3. Periodontal disease
  4. Erosion of dentin
  5. dependence on others for dental resources

What screening tools are covered by Medicare?
1.HIV

  1. Alcohol misuse
  2. Smoking cessation
  3. Depression
  4. High blood pressure
  5. Blood glucose r/t cardiovascular risk assessment
  6. Lipids screening
  7. Mammography until aged 75
  8. Osteoporosis
  9. Colorectal cancer

What are the immunizations recommended for geriatrics?

  1. influenza- annually for all over 50
  2. Tetanus-diphtheria (Tdap)-once in a lifetime booster, then every 10 years
  3. Pneumococcal -1 time dose of PCV13 at 65 years old then PPSV23 1 year (12 months) later
  4. Hep B for high risk persons 1st dose then 6 months later 2nd dose, the 4-6 mos later the 3rd dose
  5. Shingrix for zoster

What are contraindications for exercise therapy

  1. unstable angina
  2. uncompensated heart failure
  3. Severe anemia
  4. Uncontrolled blood glucose
  5. Unstable aortic aneurysm
  6. Uncontrolled hypertension or tachycardia
  7. Severe dehydration or heat stroke
  8. Low oxygen saturation

What are the benefits of exercise?
Strengthen muscles
Maintain flexibility
Promote balance
Build and maintain bone mass
Prevent injury
Improve daily functioning, especially in elderly
Prevent chronic disease

What assessment screening tool evaluates functional decline?
How long is the time frame?
The Timed Get up and go Test
16 seconds.

What screening tool is used for domestic violence?
HITS

What screening tool is used for cognitive and affective disorders like dementia, delirium,depression and spiritual health

  1. Mini-Mental SE,
  2. The mini-Cog and
  3. the Montreal Cognitive (MoCA),
  4. Geriatric Depression scale
  5. HOPE, and SPIRIT

agitation
Change in behavior,-repeating questions, arguing,
Labs- CBC, CMP, B12, TSH
Tx-Psychotropic meds ( Abilify, Risperdal)

  • antiseizure meds (Tegretol, Lamictal)
    anxiolytics- alprazolam, diazepam

Alcoholism
alcohol misuse-tachycardia, peripheral neuropathy, tremors
Labs- SBIRT
Tx- sobriety or total abstinence

what is anxiety? what labs? what meds?
excessive worry difficult to control-poor health, financial insecurity, etc.
Labs- CBC, CMP, TSH Geriatric Anxiety Inventory screening
TX- start low and go slow-SSRIs-Escitalopram, sertraline, citalopram

What are the 5 domains of a comprehensive geriatric assessment

  1. Physical health,
  2. functional health
  3. psychological health
  4. socioenvironmental supports
  5. quality of life

What 3 things do a comprehensive geriatric assessment do?

  1. Identifies care needs
  2. Plans care
  3. Improves outcomes

What is the most frequently used criteria for evaluating drug use?
The BEERS criteria, the STOPP criteria and the START criteria

What are 9 biomarkers of nutritional deficiency?

  1. Prealbumin,
  2. transferrin,
  3. albumin,
  4. chemistries,
  5. cbc,
  6. vitamin B12,
  7. folate,
  8. vitamin D and
  9. thyroid panel

What bipolar disorder? What labs? What meds?
S and Sx: Euphoria or irritability, depression, hypomania, rapid speech and irritability, can begin after 50, assess with DIGFAST or SIGECAPS
Labs- cbc, cmp, tox screen, u/a, thyroid function, rpr
Meds: Mood stabilizers-lithium valproic acid, and the antipsychotics- quetiapine and olanzapine

Depression symptoms? Labs? Tx?
SIGECAPS measures the disturbance in:
Sleep
Interest
Guilt
Energy
Concentration
Appetite
Psychomotor slowing
Suicidal Ideation
Labs assist in causative factors
Tx-Beahvior stabilizer-Haldol, mood stabilizer- olanzapine, risperidone, quetiapine

Dementia, Iabs and, Tx
A slowly progressive global cognitive decline- confusion, disorientation
Labs: cbc, electrolytes, glucose, BUN, Cr. LFTs, TSH, folate, syphilis serology and UA, Mini mental or MoCA
Tx- Cholinesterase inhibitors – donepezil, rivastigmine, galantamine or
Namenda

Depression, labs, tx?
A pervasive feeling of sadness, discouragement, can start at age 60.
Labs- electrolytes, hormone levels, nutritional deficiency, B13 and vitamin D deficiency
Tx- SNRI’s- duloxetine, Effexor, SNRIs-citalopram, escitalipram, sertraline

elder abuse
Any knowing, intentional, or negligent act by a caregiver or any other person to an older adult; the act causes harm or serious risk of harm
Labs-?
Tx-call the elder abuse hot line

T/F
Decreased size, number of cells in CNS (gyral atrophy), Ventricular dilation, and
cerebral metabolic rate of O2 utilization decline are normal processes of aging in CNS.
True

T/F

Cortical neuronal loss is part of normal aging.
False! – not part of normal aging

Areas where there is neuronal loss
Thalamus, straitum, basal forebrain, hippocampus

T/F

Not all older adults are frail and not all frail older adults are disabled.
True

T/F

Frail older adults have decreased rates of survival as they age.
True

Define frailty
Decreased ability to cope with acute stressors due to age-related physiological/functional decline

T/F

Frailty may be reversible via PT.
true

Typical characteristics in frailty
Decreased functional reserve
Impairment in multiple physiological systems
Reduced ability to regain physiological homeostasis after stressful event

Older adult shows weakness, slowness, exhaustion, weight loss, grip strength in lowest 20%, walking time in lowest 20%, unintentional weight loss over 10 lbs in past week.

How would you describe this patient?
Frail older adult (typical phenotype)

Fried’s frailty levels criteria
What groups are there?
Weight loss
Weakness
Slowness
Exhaustion
Low physical activity

Non frail: no criteria
Pre frail: one or two criteria
Frail: three or more criteria

This model describes frailty as a dynamic state, where frailty is based on the tenuous balance between assets and deficits; this balance determines a person’s ability to remain independent.
Rockwood model/ Frailty index (FI)

Rockwood model classifies people into what categories?

  1. Well elderly clients whose assets > deficits
  2. Frail, community-dwelling elderly who assets are in precarious balance with their deficits.
  3. Frail, institutionalized, elderly clients who deficits outweigh their assets.

T/F

Rapid change in frailty index (FI) is often seen in people within years before death.
True. These accelerations tend to be more predictive of death than chronological age.

T/F

When dementia is present, degree of frailty typically corresponds to degree of dementia.
True

Simple “Frail” Questionnaire screening tool
Fatigue
Resistance ex
Aerobic ex
Illnesses
Loss of weight

1-2 = prefrail
3 or more = frail

Function
Comorbidity
Geriatric syndromes
Nutrition
Polypharmacy
Economic resources
Social support

These are elements of what? (assessment tool)
Comprehensive geriatric assessment

List all tests for frailty
Prism 7 questionnaire
FI (frailty index)
Cardiovascular Health Study Frailty Screening
Scale
Simple “FRAIL” screening questionnaire

Other signficant tests to identify frailty
Gait speed
TUG
Observed Tasks of daily living

T/F

Medicaid is major payer of long term care.
True

How long is length of stay for medicare
Length of stay for late stage dementia
21 days

5 years

Static
vs
dynamic measures of frailty
Static: MMSE, poor vision/hearing,

Dynamic: Decline in peak flow, cogntiion, increase/decrease in anything

Slow gait velocity defined as
<1 m/sec

What score on TUG indicates needing assistance for transfers, stair climbing, and going out alone?

30 sec

T/F

Individuals with PAD do not have significantly different walk speed, endurance, or function.
False. all decreased over time.

T/F

Individuals who have COPD and decreased walk speed are at a higher risk for hospitalization.
True

Patient’s walking speed of ….is a community walking speed making the patient safe for walking in the community, less likely to be hospitalized, more independent in self care and less likely to have adverse events.

1.0 m/s

What type of PT intervention has demonstrated significant improved muscle strength, gait speed, stair climbing and overall activity level?
Strengthening intervention of hip and knee extensors

What does secondary prevention mean?
Decreasing risks of functional decline and impairment progression

What kind of exercise has been shown to increase max VO2 consumption whether in lower or moderate intensity?
Aerobic exercise

0.8-1.2 m/s gait speed indicates individual is considered a…
Community ambulator

0-0.4 m/s gait speed indicates individual is considered a…
Household ambulator

0.4-0.8 m/s gait speed indicates individual is considered a…
Limited community ambulator

T/F
Patient’s walking speed of <0.6 m/s indicates walking is severely impaired, the patient is only able to walk at home or very limited community walking, the patient has increased dependence in ADL’s and is at risk for hospitalization and falls.
True

One study showed that …. best discriminated between non-frail and frail.
Gait speed

Long term care includes what services…?
Personal care
Household chores
Life management

3 Settings of PT intervention to intervene for frailty
acute care
Home based
Community dwelling

What should be included in PT POC for frail older adult?
-Identify Fall risk and Prevention
-Muscle strengthening, balance intervention
-Endurance, Gait
-Environmental assessment and modification

The therapist receives a referral to examine the fall risk in an older adult who lives alone and has had two recent falls. The activity that represents the the common risk factor associated with falls in the elderly is:
A.climbing on a stepstool to reach overhead objects
b..walking with a roller walker with hand brakes
C. dressing while sitting on the edge of the bed.
D. turning around and sitting down in a chair
D

An older patient is referred to physical therapy for back pain. Medical tests reveal he is at the end stage of pancreatic cancer. The physician has told him he has cancer but has chosen not to inform the patient about the prognosis. The patient as!s the therapist what his prognosis is. The therapist’s best response is to tell him:
A.to discuss his concerns with the physician.
b. To ask the nurse practitioner.
c. “Everything usually works out for the best
D. the prognosis
A.

The highest level of provider risk in reimbursement is related to:
a. The cost-based payment method
b. The fee-for-service payment mehtod
c. the capitation payment method.
d. the per diem payment method
c.

A patient is referred to physical therapy following a fall injury (fractured left hip with ORIF); medical history reveals a diagnosis of Stage I Alzheimer’s disease. At this stage, the behaviors the therapist would expect to find are:
A.anxiety and irritability.
B. language comprehension problems.
c. fragmented memory.
d. .restlessness and sundowning
B.

A 73-year-old is referred to physical therapy for an examination of balance. He has a recent history of falls (two in the last 6 months). Based on knowledge of balance changes in the elderly and scoring of standardized balance measures, the test data that BEST indicates increased fall risk is:
A. Functional reach of 7 inches.
B. Berg Balance score of 50.
C. Tinetti Performance Oriented Mobility Assessment (POMA) of 27 seconds.
D. Timed Up-and-Go of 13 seconds.
A

Grip strength, walking speed, weight loss, fatigue, and low physical activity are all factors in what screen?
Fried Frailty screen

Self report measure of balance function, correlates with getting up from a fall and level of anxiety, 10 item scale,
Falls Efficacy Scale

What are 3 gait variables that differentiate fallers from non-fallers?
1 fallers demonstrated increased knee flexion and decreased trunk rotation with comfortable gait speed in over ground walking
2 fallers consistently took several small steps immediately prior to obstacle clearance
3 fallers had shortened step and stride length, slowed velocity and cadence decreased single support time during obstacle clearance

100 percent risk factors for falls in individuals with these 3 things?
hip weakness
unstable balance
over 4 medications

Unusual clumps of a protein plus degenerating bits of neurons and other cells that gunk up the brain seen in Alzheimer’s patients are called…?
Amyloid plaques

Which cognition test is a rapid screening assessment for mild cognitive dysfunction and assesses attention and concentration, executive functions, memory: language, visuoconstructional skills, conceptual thinking, calculations, and orientation?
MOCA

A 72-year-old patient is walking on a treadmill in the physical therapy department while his vital signs are being monitored. It is noted that his SaO2 drops from 97% to 95%. In this case, it would be BEST to:
A.place a 100% O2 face mask on the patient for the remainder of the exercise session.
B..place a 40% O2 face mask on the patient for the remainder of the exercise session.
C. place 2 liters of O2 by nasal cannula on the patient for the remainder of the exercise session.
D. Do not use supplemental oxygen
D.

A 72 year-old patient with a transfemoral amputation is unable to wrap his residual limb. Your BEST course of action is to:
A. apply a temporary prosthesis immediately.

B. Consult with the vascular surgeon about the application of an Unna’s paste dressing.

C. Redouble efforts to teach proper wrapping.

D. use a shrinker
D

A 99-year-old woman was found unconscious at home. Two days later the therapist examines her in the hospital. Findings include normal sensation and movement on the right side of the body with impaired sensation (touch, pressure, proprioception)and paralysis on the left side of the body. The left side of her lower face and her trunk are similarly impaired. The MOST LIKELY location of the lesion is the:
A.left side of brainstem.
B.left parietal lobe.
C.spinal cord.
D. right parietal lobe
B.

An older patient has lost significant functional vision over the past 4 years, and complains of blurred vision and difficulty reading. The patient frequently mistakes images directly in front of her, especially in bright light. When walking across a room, the patient is able to locate items on both sides of the environment using peripheral vision. Based on these findings, the therapist suspects:
a. Glaucoma
b. Cataracts
c. Homonymous hemianopsia
d. Bitemporal hemianopsia
b. Cataracts

or macular degeneration (loss of central vision) if it is a choice

A 91-year-old patient has reduced vision as a result of bilateral cataracts. Which of the following is NOT an appropriate intervention for this patient?
A. Using high illumination for reading and ADLs.
B. Avoiding having him walk ambulation on shiny floor surfaces.
C. highlighting steps with pastel colors of blues and greens.
D.minimizing visual distractions in his the immediate environment.
C.

An older patient has been hospitalized( on complete bedrest for 10 days. A physical therapy referral requests mobilization and ambulation. The patient complains that today his right calf is aching. If he gets up and moves around he is sure he will feel better. The therapist’s examination reveals calf tenderness with slight swelling and warmth. The therapist decides to:
A.ambulate the patient with his support stockings on.
B. postpone ambulation and report the findings immediately.
C. begin with ankle pump exercises in bed and then ambulate.
C. use only AROM exercises with the patient sitting at the edge of the be
B

A 72y.o. patient is recovering from a right total hip replacement (posterolateral incision, cementless fixation;. The MOST appropriate type of bed to wheelchair transfer to teach is to have the patient use a:
A.stand pivot transfer to the surgical side.
B.lateral slide transfer using a transfer board.
C.stand pivot transfer to the sound side
D..squat pivot transfer to the surgical side
C.

A 75 year-old resident received a cemented total hip replacement 2 days ago. The physical therapy plan of care should include:
a. AROM exercises and early ambulation using a walker, NWB
b. PROM exercises and gait training using crutches, weight bearing to tolerance
c. Proper technique for transferring to the toilet
d. Patient education regarding positions and movements to avoid.
D. Patient education regarding positioning is the most important thing to include

An 80-year-old patient with emphysema and a history of hypertension performs a 12 minute walking exercise tolerance test. He was able to walk 1,016 feet. His vital signs prior to exercise were: HR 104, BP 130/76, SaO2 93%. At peak exercise his vital signs were: HR 137, BP 162/74, SaO2 92%. To calculate his exercise intensity parameters, the BEST method to use is:
A. 40-85% HR reserve (Karvonen’s formula)

An 85-year-old wheelchair dependent resident of a community nursing home has a diagnosis of organic brain syndrome, Alzheimer’s type, Stage 2. During the therapist’s initial interview, the patient demonstrates limited interaction, mild agitation,and keeps trying to wheel her chair down the hall. As it is late in the day, the therapist decides to resume the examination the next morning. The patient is most likely exhibiting:
Sundowning behavior

A 74-year-old patient has had two recent falls coming home from bingo after dark. Her outside steps are well lit. She is unsure why she has fallen but tells the therapist both times she fell just as she came into her house, before she even had a chance to put her purse down and turn on the inside lights. The therapist suspect a problem with:
A. Loss of accomodation and near vision
B. Decreased corneal sensitivity
c. Poor light adapation
d. Decreased ocular scanning movements
c. Poor light adaptation

A 72-year-old male patient recovering from a fractured hip repaired with ORIF has recently been discharged home. During a home visit, his wife tells the therapist he woke up yesterday morning and told her he couldn’t remember much. Upon examination, the therapist finds some mild motor loss in his right hand and anomia. The therapist affirms the presence of short-term memory loss. The BEST course of action is to:
A.refer him to his physician as the therapist suspects Alzheimer’s dementia.
B.refer him to his physician as the therapist suspects a small stroke.
C. advise the family to document and record any new problems they notice over the next week, then report back to the therapist.
D. Ignore the findings as they are expected following surgical anesthesia
B.

When planning a PT program for elderly patients with AD who have difficulty with walking safely, it is important that the physical therapy session….
uses familiar/functioanl purpose to walk
Is functional; includes turns, looking around, different surfaces
Involve patient in process
Verbal/tactile/auditory cues

A 65 year-old patient with advanced coronary artery disease and diabetes is receiving functional mobility training in a P.T. clinic. While walking after lunch, the patient experiences difficulty breathing, starts hyperventilating, and suffers an episode of syncope. The MOST LIKELY cause of this problem is:
A.postprandial hypertension.
B.coronary artery disease.
C.hyperglycemia.
D.seizures
B.

A 77 year-old patient lives alone and is referred for home physical therapy services to improve functional mobility. He refuses to get out of his chair. Upon examination, he appears irritable, with poor concentration and memory. He tells the therapist he has not been sleeping well and has no energy or desire to do anything. He appears anorexic. The MOST LIKELY explanation for his symptoms is:
A.depression.
B.hypothyroidism.
C. alcohol abuse.
D.pseudodementia
A. Depression

A 59-year-old ex-machinist demonstrates significant age-related hearing loss, presbycusis. When trying to communicate with this patient you would NOT suspect:
A. bilateral hearing loss, at all frequencies since he has had this problem for a number of years.
B. decreased language comprehension.
C. poor auditory discrimination.
D. unilateral hearing loss
D.

A patient who is terminally ill with cancer is in tears, unable to cope with the changes in her life and with her current hospitalization. You have a referral for gait training so she can be discharged to home under hospice care. The BEST approach for you to take is:
A.ask the patient questions so you can gain a detailed history.
B.encourage denial so she can cope better with her life’s challenges.
C.ignore her tears and focus on her therapy but in a compassionate manner.
D .take time now to allow the patient to express her fears and frustrations
D

Your 102 year-old patient has been hospitalized for the past three days with an undisclosed ailment. After running numerous tests and finding nothing to explain her increasing weakness and fatigue, the physicians are being pressured to discharge her tomorrow. She lives alone in a first floor apartment. You have determined her ambulation endurance to be only up to 15 feet, not enough to allow her to get from her bed to the bathroom (a distance of 20 feet). You recommend:
A.a live-in nurse (24 hour coverage) until her condition improves.
B.a skilled nursing facility placement until her endurance increases.
C.environmental changes, a bedside commode, and referral for home health services.
D.postponing her discharge until she can complete the needed 20 feet
C.

A 72 year-old medically stable individual requires custodial care in the home. She is severely disabled with rheumatoid arthritis and is in a great deal of pain. She presents with significant deformities which limit her functional abilities and is dependent in all basic activities of daily living. A recent exacerbation of her disease has left her “bedbound for the past 2 weeks. Appropriate physical therapy services would be covered by:
A.Medicaid.
B. Medicare.
C.Medigap policies.
D.only by HMO or private insurance policies
B. (medicare part A)

A terminally ill patient completes a formal document that names his daughter as the individual to make healthcare decisions in the event that he is unable. This type of advance directive is termed:
a. Living will
b. Durable power of attorney
c. Directives to physicians
d. Euthanasia
b.

A 95 year-old has recently been admitted to your skilled nursing facility following a fall-related injury. Since she lived alone on the second floor, she was unable to return home. She is extremely agitated over her placement here and demonstrates early signs of dementia. She tells you “leave me alone, I just want to get out of here!” An important approach to take while working with this client is to:
A. Be calm and supportive, using only one or two level commands.
B. Establish the rules: tell her exactly what she is to do while you are with her.
C.use gesture or sign language to communicate with her.
D.promise her anything as long as she gets up and walks for you
A.

A 72 year-old woman is being treated for depression following the death of her husband. She is currently taking antidepressant medication (tricyclics) and has a recent history of a fall. You suspect the precipitating cause of the fall can be attributed to side effects of her medication resulting in:
A.cardiac arrhythmias
B .dyspnea.
C.hyperalertness.
D.postural hypotension
D. Postural hypotension

An 82 year-old patient and his caregivers should understand the common side effects of the medication that he is taking. He is continually in and out of congestive heart failure and has been taking digitalis (digoxin) to improve his heart function. You will know he and his caregivers understand the adverse side effects of this medication if they tell you they will contact the patient’s physician if he demonstrates:
A.confusion and memory loss.
B.involuntary movements and shaking.
C.slowed heart rate.
D. weakness and palpitations
D.

A 76 year-old patient exhibits impaired balance. A diagnostic work-up has failed to reveal any specific etiology. An initial intervention for this patient would NOT include:
A. limits of stability re-education including postural sway training.
B. practice in maintenance of a wide base of support during gait and turns.
C. sit-to-stand and stand-to-sit activity training.
D. tandem walking and single limb stance
D.

An older person with diagnosis of congestive heart failure should recognize the symptoms of exertional intolerance. You will know this client is properly informed if she can tell you these symptoms are:
A.dizziness, visual blurring especially with turns and quick movements.
B.overwhelming weakness with difficulty in standing up and walking.
C.severe, uncomfortable chest pain with shortness of breath.
D.shortness of breath at rest and with limited activity and sudden weight gain
D

Long term care for institutionalized elderly who have reduced their financial resources or qualify for low-income status is typically funded by:
A. Health Maintenance Organizations.
B. Medicaid.
C. Medicare.
D. Social Security Administration
B.

A 72 year-old patient with peripheral vascular disease has been referred for conditioning exercise. He demonstrates moderate claudication pain in both legs following a 12 minute walking test. The MOST appropriate exercise frequency and duration for this patient is:
A. 2 times/week, BID, 20 minutes/session.
B.3 times/week, 30 minutes/session.
C.3 times/week, 60 minutes/session.
D. 5 times/week, BID, 10 minutes/session.
D.

A physical therapist is working with a patient with metastatic breast cancer who has been told that she has only months to live. She is quite angry and disruptive during therapy. What is the MOST appropriate intervention for this patient?
A.allow the patient to express her anger while refocusing her on effective coping strategies.
B.forbid all expressions of anger as she is only hurting herself.
C.provide honest, accurate information about her illness and rehab plan of care.
D.provide opportunities for the patient to question her impending death but limit all expressions of anger
A.

A 62 year-old patient has chronic obstructive pulmonary disease. The MOST LIKELY pulmonary test result would be:
A.decreased functional residual capacity.
B.decreased residual volume.
C.increased total lung capacity.
D.increased vital capacity
C.

A 77 year-old patient has been confined to bed for a period of 2 months and now demonstrates limited ROM in both lower extremities. Range in hip flexion. The MOST appropriate intervention to improve flexibility and ready this patient for standing is:
A.dynamic lower extremity splints, applied for 2 hours daily.
B.hold-relax techniques followed by passive ROM 3 times a week.
C.manual passive stretching, 10 repetitions each joint, 3 times a day.
D.mechanical stretching using traction and ) l”. weights$ ‘ hours$ twice daily
D. Mechanical stretching (low intensity over prolonged time)

You are a home health physical therapist. During one of your regularly scheduled visits with a 72 year-old male patient, you find him to be confused with shortness of breath and significant generalized weakness. Assessing these symptoms and given his history of hypertension and hyperlipidemia, you suspect:
A.he forgot to take his hypertension medication.
B .he may be experiencing unstable angina.
C.he may be presenting with early signs of myocardial infarction.
D.his mental changes are indicative of early Alzheimer’s disease
C.

A 62-year-old lives at home with his wife and adult daughter. He has recently been diagnosed with multi-infarct dementia and is recovering from a fractured hip following a fall injury. In your initial interview with his wife you would expect to find:
A.agitation and sundowning.
B.history of steady progression of loss of judgment and poor safety awareness.
C.history of sudden onset of new cognitive problems and patchy distribution of deficits.
D.perseveration on a thought or activity
C.

During a physical therapy session for low back pain, a 67 year-old patient tells you that she has had urinary incontinence for the last year. It is particularly problematic when she has a cold and coughs a lot. She has not told her doctor about this problem because she is too embarrassed. Your BEST course of action is to:
A.examine the patient and proceed with her back treatment.
B.examine the patient, document, and discuss your findings with the doctor.
C.examine the patient, document the problems, then send her back to her doctor.
D. Refer the patient back to her doctor
B.

An older person with a diagnosis of degenerative joint disease should understand his disease, its progression, and its management. You will know your patient education has been effective if he can tell you:
A.aerobic conditioning is not appropriate when pain is present and medications must be used.
B. Joint protection strategies are important but cannot reduce the expected loss of function.
C.loss of ROM and immobility are expected and irreversible.
D.pain and stiffness are worse in the early morning and should decrease with moderate activity
D

Fastest growing subpopulation in next 30 yrs?
85 and older (oldest-old adults)
Or older adults with DD

A 72-year old patient is receiving outpatient physical therapy at your private clinic. The clinic is an approved medicare outpatient provider. Your patient is concerned that she will not be able to pay for her continuing care and worries that her medicare benefits will run out soon. You tell her:
A.coverage is limited only for hospital-based outpatient PT services.
B.currently there is no limit to her medicare coverage for outpatient PT services.
C.there is currently a limit of $1000.00 for coverage of outpatient PT services.
D.there is currently a limit of $1590.00 for coverage of outpatient PT services
B

Your patient is a 72 year-old man with an 8 year history of Parkinson’s disease. He demonstrates significant rigidity, decreased PROM in both upper extremities in the typical distribution, and frequent episodes of akinesia. The exercise that BEST deals with these problems would be:
A.modified plantigrade, isometric holding, stressing upper extremity shoulder flexion.
B.PNF bilateral symmetrical upper extremity D2 flexion patterns, rhythmic initiation.
C.Quadruped position, upper extremity PNF D2 flexion and extension.
D.resistance training, free weights for shoulder flexors at 80% of 1 repetition max
B

A 72 year old lives at home with his wife and adult daughter. He has recently been diagnosed with dementia and is at home recovering from a fractured hip following a fall injury. In your initial PT assessment you would most likely expect to find
Fear of falling, decreased hip strength, balance impairments

Poor quality of life and recovery rate results following hip fracture surgery for those with dementia, higher mortality rate.

Many of these patients with hip fracture are deemed poor candidates for rehabilitation due to cognitive difficulties in following directions

The therapist receives a referral to examine the fall risk in an 82 year old that lives alone and has had two recent falls. The activity that represents the MOST common activity associated with falls in the elderly is…?
*Turning around and sitting down in a chair

ADLs -getting up to go to bathroom from bed

An elderly patient is recovering in the ICU from a severe case of pneumonia and dehydration. You recognized his disorientation is due to delirium rather than dementia because…
a. Hallucinations are present throughout the day
b. Persistent personality changes are evident
c. Symptoms are intermittent
d. Level of arousal is significantly depressed
C. Symptoms are intermittent

Delirium is acute and reversible –> precipitating factors include acute illness, surgery, dehydrations, infections

An 85 year old resident of a continuing care residential home is diagnosed with organic brain syndrome, Alzheimer’s type. Which is the BEST physical therapy intervention to recommend?
High rep/low variable = errorless learning
Space retrieval
Interdisciplinary

A frail older adult has low vision. The patient recently returned home from a 2 week hospitalization for stabilization of diabetes. The PT’s goal is to mobilize and increase ambulation level and safety. The best intervention strategy for this patient is to:
a. Practice walking in areas of high illumination and low clutter
b. Color code stairs with pastel shades of blue and green to highlight steps
c. Practice walking by having the patient look down at all times
d. Keep window shades wide open to let in as much light as possible
a. Practice walking in areas of high illumination and low cluter

A 75 year old resident received a cemented total hip replacement 2 days ago. The physical therapy plan of care should include…
Stand pivot transfer to sound side
WBAT with walker immediately after surgery
Hip precautions

Components of advanced care medical directive

  1. Durable power of attorney: Naming Health Care Proxy, who can advocate for your care when you may not be able to
  2. Living Will: Stating the types of medical treatments you do and do not want at the end of your life

Criteria for mild cognitive impairment (MCI)
Concern regarding change in cognition
Impairment in 1 or more cognitive domains
Preservation of functional ADL’s
Not demented

2 types of MCI
amnestic MCI – affects memory
non-amnestic MCI – affects thinking skills other than memory

A home care PT receives a referral to evaluate the fall risk potential of and older community dweller with chronic coronary artery disease (CAD). The patient has fallen three times in the past 4 months, with not history of fall injury except for minor bruising. The patient is currently taking a number of medications. The drug MOST likely to contribute to dizziness and increased fall risk is…
*Nitroglycerin
Calcium Channel Blockers (amlodipine-norvasc, bepridil-vascor)
Narcotic Analgesic Meds (opiods), Antihypertensives
Benzodiazepines
TCA’s

The 6-minute walk test is a helpful outcome measure as it has:
The 6-minute walk test shows a good correlation with function, as the 6MWT is a submax test and function is performed at a submax work level.

High-level training for Cognitive Function Level VII should focus on
Involving the patient in decision-making and monitoring for safety

Describe delirium
Acutely ill, hospitalized elderly patients frequently exhibit delirium, a fluctuating attention state – intermittent sympoms; lucid: then not

history of recent falls (two in the past 2 months) and mild balance instability with the eldery patient. which is best device?
front wheel rolling walker that folds
-A rolling walker will provide added stability, while maintaining gait as a continuous movement sequence.

The patient has fallen three times in the past 4 months, with no history of fall injury except for minor bruising. The patient is currently taking a number of medications. The drug MOST likely to contribute to dizziness and increased fall risk is
Nitroglycerin due to postural hypotension

Elderly patient with persistent balance difficulty and a history of recent falls (two in the past 3 months) is referred for physical therapy examination and evaluation. During the initial session, it is crucial to examine..
a. Level of dyspnea during functional transfers
b. Cardiovascular endurance during a 6-minute walk test
c. Sensory losses and sensory organization of balance
d. Spinal musculoskeletal changes secondary to degenerative joint disease.
c. Sensory losses and sensory organization of balance

A 78 year-old male with diabetes and bilateral lower extremity amputation is to be discharged from an acute care hospital 2 weeks post-surgery. The incision on the residual limbs are not healed and continue to drain. The patient is unable to transfer because the venous graft sites in the upper extremities are painful and not fully healed. Endurance out of bed is limited. The BEST choice of discharge destination for this patient is:
a. Skilled Nursing facility
b. Custodial care facility
c. Home
d. Rehab hopsital
a. Skilled nursing facility – pt continues to require nursing care for open wounds

patient with congestive heart failure is on a regimen of diuretics (chlorothiazide). The potential adverse side effects of this medication that the PT should be alert for include:
Orthostatic hypotension and dizziness

Diuretics are associated with fall risk

Elderly patient with hyperthyroidism is referred to physical therapy following a period of prolonged bedrest. During exercise, the therapist recognizes that this patient will demonstrate increased risk for..
aggravating preexisting heart disease

Discuss the characteristics of Community Dwelling Older Adults in terms of functional capabilities to live independently
1000 feet required to complete an errand in community
Gait speed of 1.2 m/s
Need to carry an average of 6.7 lb package (or 10 lbs)
Going up 10 steps

What changes are noted in the cardiovascular system during the normal aging process?
Decreased SV, CO
Decreased HR max, VO2max
Thickening of LV up to 50% by age 80
Enlargement of LA

A therapist is asked to advise a health 67 year-old individual who wants to take part in a graduated conditioning program by joining the “Mall Walkers Club”. The therapist’s best approach to prescribing the intensity of exercise for this individual is…
HR reserve formula and RPE

What are contraindications for high intensity exercise, especially WB?
Acute metabolic distress (ie: DM)
Inflammatory conditions
Extreme osteoporosis
Unstable chronic cardiac disease

What are some questions that you might ask a frail older adult to find out about their functional status/activity level history?
Have you fallen in the past year? If so, how many times?

  • Do you have a fear of falling?
  • How many times did you leave your home in the last week?
  • Do you need assistance to leave your home?
  • Do you avoid activities because you are afraid of falling?

A home care therapist receives a referral to evaluate the fall risk potential of an 82 year old community dwelling eldery with chronic COPD. The patient has fallen three time in the last four months with no history of fall injury except for minor bruising. The patient is currently taking a number of medications. The drug LEAST likely to contribute to increased fall risk is:
*Buffered aspirin (non-opioid)

WRONG answers:
Antidepressants
Sedatives
Benzodiazepines
Neuroleptics
Diuretircs
ACE inhibitors
Digoxin

Discuss the changes in pharmacodynamics that occur in the normal population of older adults
Changes at cellular level: Drug receptor affinity, cells ability to generate a response, transmembrane signalling

Changes at systematic level: decline in hemostatic mechanism (orthostatic circulatory response, posture/balance, cognitive function, muscle strength/endurance, others)

Discuss 3 consequences of polypharmacy in older adults that would have a specific impact on your physical therapy intervention
Impaired cognition: Following directions, delirium
Postural hypotension
Weakness

Describe risk factors for potential for developing ADR in elderly clients

75 yrs old, Small stature, organ dysfunction
Decreased lean body mass/high fat mass
Dehydration
Hepatic or renal insufficiency
Multiple co-morbidities

Changes in motor function, change in hemodynamics, increased ICP, changes in emotion/mood swings, alterations in arousal are examples of…
Neurologic dysfunction

T/F
Cognitive dysfunction make up the largest group of functionally disabled individuals.
True

T/F
Acute cognitive dysfunction of rapid onset without underlying damage to brain tissue carries the best prognosis for recovery.
True

confusion and restlessness, reversal of sleep-wake cycles, paranoia, acute onset, fluctuation in nature, and hallucinations are characteristic of…
Delirium

slow onset of increasing intellectual impairment, including disorientation, memory loss, and loss of social skills, and development of regressed/antisocial behavior are characteristic of…
Dementia

T/F

Serious mental decline is part of normal aging.
FALSE

Individual presents with poor judgement or decision making, inability to manage a budget, difficulty with having a conversation, and losing track of date. These are signs of…
Dementia

What kind of dementia presents with fluctuating cognition, hallucinations, and PD signs?
Dementia w/Lewy Bodies

What kind of dementia has rapid onset in younger individuals and frequently coincides with HTN, atherosclerosis, and DM?
Multi-Infarct Dementia

T/F
Mild cognitive impairment describes individuals with subjective memory symptoms that decreases their capacity to perform ADL’s.
FALSE. Individuals with MCI have normal ADL’s

T/F
Vitamin B12 deficiency, chronic alcoholism, metabolic imbalances, hydrocephalus, and depression are all treatable causes of dementia.
True

T/F

Alzheimer’s Disease is a form of progressive dementia that is irreversible.
True

Define implicit processing
The use of unconscious and automatic processing for learning and retrieving information.

Define explicit processing
The use of conscious and effortful processing for learning and retrieving information
“intentional learning”

Define metacognition
Higher-level processing skills used in goal-directed behavior

What would you include in a cognitively active lifestyle for older adults? (3 things)
Physical activity
Cognitively stimulating activities and challenges
Social interactions

Diagnostic tools for AD
Screening
Depression scales
MRI and CT

Global Deterioration Scale Staging for AD
1: No cognitive decline
2: Very mild – you think you have memory lapses
3: Mild (MCI) – friends notice
4: Moderate – moody/wihdrawn
5: Moderately severe – help with ADLs
6: Severe – lose awareness, get lost
7: Very severe – response to env, self-care

Medications for Alzheimer’s disease
NMDA receptor antagonists
Antidepressants
ACE inhibitors

Communication strategies during PT for pt with AD
*AVOID quizzing patient ie. Do you remember what you had for breakfast today
Calm manner
Don’t Rush
Assess for distractions
Avoid correcting pt
Monitor nonverbal communication
Eye contact and touch to get pt’s attention

T/F

Constant and blocked practice that is consistent during PT sessions is good for pts with AD.
True

T/F

Training to specific and functional task is important for AD.
True – may not see transfer of training

T/F

Decrease in number of anterior horn cells and nerve conduction velocity are normal age-related changes.
True

3 components of Successful Aging
Low level of disability
High cognitive and functional capacity
Active engagement

T/F

MMT, isokinetics, and 1-RM are appropriate interventions for strength for the elderly.
True

T/F
Intermediate care facilities provide the most intensive group home setting for individuals with health problems, multiple disabilities or vary limited daily living skills.
True

T/F
Community residencies are group home setting for individuals with moderate abilities to care for themselves. Individuals whose primary disability is moderate mental retardation or autisms often reside in these facilities.
True

These places are for individuals with a significant level of independence. These facilities are often apartments and usually consist of two to three roommates. Monitoring by supervisor is done weekly or as needed.
Supportive residencies

T/F
Family Care homes are for individuals of all degrees of disability.
True. An individual resides with a family who has been trained and licensed to care for individuals with DD.

T/F
Community-based model of care is needs-based, provides individualized, specialized services, and bridges services with age eligibility for services.
True

T/F

CVD is leading cause of death in those with intellectual disabilities, except for those with Down syndrome.
True

T/F

Down syndrome has not been found to be linked to AD and Depression.
False – Down syndrome is related to both.

Tests to use in exam and eval of older adults with DD.
FIM, Katz
Scale of Independent Behavior revised
Support Intensity scale (program planning,Home living, community living, lifelong learning, employment, health & safety, social activities, protection and advocacy)

T/F
Older adults with intellectual disabilities have lower levels of physical fitness than those who do not have IDs.
True

T/F
Balance, comfortable gait speed, strength and muscular endurance have been shown to be significant predictors for falls in older adults with ID.
False. Balance, comfortable gait speed, strength and muscular endurance were not significant predictors for falls in older adults with ID.

T/F

Obesity and CVD are linked to older adults with DD.
True

When do you need to do a fall and risk factor evaluation?
When the pt reports 2 or more falls in 12 months

T/F
Research has shown that people with greater cognitive reserve are better able to stave off the degenerative brain changes associated with dementia or other brain diseases, such as Parkinson’s disease, multiple sclerosis, or a stroke.
True

T/F

Cogntivie impaimrents become apparent when cognitive or neurologic resources become depleted
True

Define prospective memory
Remmebering to execute an action planned for the future

Rehabilitation vs habilitation services
Rehabilitation – get back or maintain skills that have been lost
Habilitation – keep/learn/maintain skills for daily living

T/F

Medicare A covers inpatient hospital stay for 90 days per episode of illness with 60 lifetime reserve days.
True

This medicare offers alternative to traditional medicare fee-for-service coverage; allows beneficiary to choose to join a Medicare approved health plan operated by private insurance – lower out-of-pocket costs, but limited to restricted provider networks.
Medicare C

Must join a plan run by insurance
company in their state
Coverage of medications up to certain
dollar amount

Which Medicare is this?
Medicare D

Coding system use in all US health
setting for coding diagnosis and
inpatient procedures
ICD-10

T/F
Analyzing, Modifying, Establishing, Developing, Acute exacerbation, Evaluating the effectiveness, Individualized, and safety are effective words for documentation.
True

T/F
Monitoring, Chronic, Observing, Repetitious, Supervising, Encouraging, Refused, ambulation, Endurance, and uncooperative are effective words for documentation.
False.

patient’s timed up and go (TUG) score is 33 seconds. Based on these results the therapist determines that the patient’s fall risk is:
scores over 30 are indicative of impaired functional mobility and high risk for falls.

Global Deterioration scale, Cognitive Disabilities Model and FAST are used to describe…
Dementia

Use errorless learning for…
Dementia/AD

Spaced retrieval – when is info considered learned
High repetition to facilitate recall of facts
Designed to promote recall of info over longer time intervals

Considered learned if they can repeat it after 16 minutes

How to assess pain in someone with dementia?
Nonverbal pain signs
-Facial expression ie. grimace, clenched jaw
-Body posturing

T/F

Palliative care begins at diagnosis and at the same time as treatment
True

T/F

When communicating with the person facing end of life, you should assure them using phrases like “we’re going to get you better and “you’re going to be fine”.
FALSSEEE. Can’t tell the this, not true

Karnofsky Scale (what population is it used for)
Developed for cancer patients to assess function

Cessation of eating/drinking/ more awareand somnolent, blue nail beds, and change in breathing pattern may indicate…
impending death

T/F

People often really with a last effort before they decline.
True

Ethical principles of PT
Autonomy
Non-malficence
Beneficence
Justice

Professional duties
Competency
Confidentiality
Fidelity
Truth

T/F

Competent pts have the moral and legal right to refuse treatment.
True. “Right to refuse”

Unexplained injury, skin irritation, burns, annoyance over personal Q’s during history, inappropriate withdrawal to touch, poor hygiene, and soiled clothes are all signs of…
Elder abuse

T/F

Residents can refuse restraints.
True

What do you need in order to use restraints?
Documentation for why they are necessary
Informed consent

What is CDC recommendation for frequency of exercise for older adults?
150 min of moderate intensity PA/week
including muscle strengthening at least 2 days/wk

Functional makrer for gait speed
0.8-1.5 m/sec

Functional marker for 6MWT
350-500 m

Medicare for…
Medicaid for…
people over 65 years old
Federal and state funding for low income people

T/F
Medicare lacks coverage for preventative care, hospital, skilled nursing care, long term care at home or nursing home.
True

Diagnosis-Related Groups (DRGs)
Hospitals paid a fixed amount for each pt made on principal diagnosis for each

Payment method where an amount is assigned to a specific procedure or service is called…

Provider risk is….
Fee for service

Provider risk is lowest

Payment method where an amount is associated with all services provided in each day of care is called…

In this, provider risk is….(low/high?)
Per diem
Provider risk is low

Payment method where there is an amount for all procedures and services associated with an episode of hospitalization

Provider risk is…
Episode of illness

Provider risk is moderate

A fixed payment given to providers at specific intervals for all care provided to patients in managed care plans

Provider risk is …
Capitation

Provider risk is high

Choice of payment methodology by providers, utilization management, benefit restrictions, increased patient cost sharing, limitations on covered services, and financial caps on services are all ….
Cost control mechanisms

T/F

Medicare Part A covers Inpatient hospital care, SNF, Home health care, and hospice care.
True

T/F

Medicare Part A covers outpatient services, doctor’s services, and durable medical equipment.
False. That’s part B

T/F

Medicare C provides parts A, B, and D
True

T/F

Medicare D pays for prescription drugs.
True

T/f

Medicaid supports long term services, but not nursing home care.
False. It’s the opposite. Medicaid does not support long term services, principally supports nursing home care and adult daycare

T/F

Pulmonary changes associated with normal aging include increased lung compliance, decreased chest wall compliance, decreased lung volumes, and decreased cough effectiveness.
True

T/F

There is a linear relationship between HR and oxygen intake.
True

T/F

Ejection fraction at maximal exercise tends to decline at advanced age.
True

T/F

Max CO does not decline with age.
False. It does decline due to decreases in HR and stroke volume

Modified Chair Step test and 2 min step test are examples of…
Sub max exercise tests

T/F

Unstable angina, uncontrolled symptomatic HF, and acute systematic infection are relative contraindications for aerobic exercise in older adults.
False. These are ABSOLUTE contraindications.

T/F

Fatigue, SOB, leg cramps, ST depression >2 mm, and bundle branch block are absolute indications for terminating exercise.
False. These are relative indications for terminating exercise

What are some absolute indications to terminate exercise?
Drop of systolic BP>10 mmHg w/ischemia
Increase in nervous system symtpoms
Sustained ventricular tachycardia

Symptoms such as confusion, alteration of sleep-wake cycle, increased CHF, anorexia, and failure to thrive may indicate…
Pneumonia

Loss of skeletal muscle mass associated w/normal aging is called…
sarcopenia

T/F

Exercise is not appropriate for the older adult with COPD.
False. Exercise increases functional activities, decreases exertional dyspnea, and improves muscle function in COPD

T/F

Systematic heat and exercise using large muscle groups may cause peripheral vasodilation, which can result in drop in BP.
True

What test defines the highest level of locomotion?
FIM (functional independence measure)

Hormonal deficiency associated with menopause, nutritional deficiency of calcium, excessive alcohol and caffeine consumption, hyperthyroidism, and corticosteroid usage are risk factors associated with…
development of osteoporosis

What are the major side effects of the use of pain medications in the geriatric population that can be a concern for physical therapists?
Increased fall risk, disorientation, and sedation

Most important components of initial examination of a patient with dementia?
Onset of symptoms
Cognitive function ie. MMSE
Communication and perception
Behavioral changes, self-care, motor function

Loss of peripheral vision and missing objects to the side and out of corner of eye is associated with…
Glaucoma

Among the changes in the spine that occur with normal aging include less flexibility in the soft tissue, decreased mineralization of bones and…
Intervertebral discs flatten : DJD
Postural changes – FHP, thoracic kyphosis, flattening of lumbar spine

93 year old is referred to PT with back pain. The patient has pancreatic cancer and moderate senile dementia and is unaware of the dx or prognosis at the family’s request. The patient asks the therapist what is the matter and how long will PT take place. The therapist BEST response is:
a. Ill treat the symptoms and see how well you do

An elderly individual has limited endurance as a result of a sedentary lifestyle. There is no hx of cardiorespiratory problems. Following an exercise tolerance test, which was negative, the MOST appropriate initial exercise prescription parameters for this individual would be:
a. 60-90% HRmax (because patient is asymptomatic—it is the equivalent of 50-85 of VO2 max)

A patient is referred to PT follow fall injury (left hip fx with ORIF). Medical hx reveals a dx of state I Alzheimer’s disease. At this stage, the behaviors the therapist would NOT expect to find are:
a. Language comprehension problems
-Stage I is characterized by memory loss, absentmindness, difficulty concentrating and occasional word finding problems)

An elderly and frail resident of an extended care facility has intractable constipation. During a schedule visit from PT, the patient complains of abdominal pain and tenderness. The therapist recognized the patient may also experience:
a. Pain and tenderness in the anterior hip, groin or thigh region

Elderly and frail individual is receiving PT in the home environment to improve general strengthening and mobility. The patient has a 4 year hx of taking (NSAIDS) for joint pain and recently began taking CCB (verapamil) the therapist examines the patient for possible Adverse side effects which could include:
a.Stomach pain, hypertension, confusion

Elderly and frail resident of an extended care facility presents with hot, red and edementous skin over the shins of both lower extremities. Along with mild fever. The MOST likely cause of the symptoms is:
a. Cellulitis

A community dwelling elder fell at home and suffered multiple fractures of the right arm including colle’s fracture of the right wrist, and humeral fracture of the right shoulder. The patient is hospitalized for ORIF of the right radius. Which is not an expected finding with this patient?
a.Decreased pain and tenderness at the fx site

Nursing homes that receive Medicare reimbursement for eligible residents are required by law to provide for rehabilitation services including PT based on:
a.Needs assessment performed by a physical therapist

An elderly resident of a community nursing home is dx with alzheimer’s type dementia. In formulating a plan of care, it is important to understand that the patient:
a.Will likely be resistant to activity training if unfamiliar activities are used

An elderly and frail patient of a nursing home has developed a stage III decubitus pressure ulcer. The wound is open with necrosis of the subcutaneous tissue down to the fascia. This elderly patient when compared to a younger patient with the same type of ulcer can be expected to demonstrate…
a.Decreased vascular and immune responses result in in impaired healing
-Rate of healing in the elderly is slower, scarring is less, elasticity and eccrine sweating are decreased.

79 year old is referred to PT for an exam of functional mobility skills and safety in the home environment. The family reports that the patient is demonstrating increasing forgetfulness and some memory deficits. From the exam the therapist would expect to find…
a.Impairments in short term memory
-Elderly patients with memory impairments typically demonstrate intact immediate recall→ can repeat words
-Impairments are often noted in memory of recent events→ why did I come into this room? Who came to see me yesterday
*Long-term memory is generally intact

What medicare pays for DME (durable medical equipment)?
Medicare B

T/F

Macular degeneration is the leading cause of vision loss and is incurable.

The biggest risk factor for macular degeneration is…
True
Increasing age.

T/F

Macular degeneration results to deficits in central vision.
True

1.Which statement made by a nurse reflects a lack of understanding regarding sexual intimacy and the older adult patient?
a. “Older adults express less interest in intimacy as both acute and chronic illnesses develop.”
b. “Sexual expression is considered an enhancement to the quality of the older adult’s life.”
c. “Expressing sexual needs may be difficult or impossible for some older adults.”
d. “Interest in physical contact tends to persist throughout life for both genders.”
A

  1. A 70-year-old female patient shares with the nurse her concern that recently it takes more time to achieve an orgasm. The nurse responds most therapeutically when answering:
    a. “You’ve described a common result of aging for both men and women.”
    b. “If you experience difficulty achieving orgasms, you should discuss that with your doctor.”
    c. “Your body produces fewer sex hormones now, and you need more stimulation to climax.”
    d. “I understand your concern. Let’s talk more about the changes you’ve noticed.”
    C
  2. A type 2 insulin-dependent diabetic 70-year-old recently lost his wife and is experiencing impotence. Besides educating the patient on the normal effects of aging on sexual function, the nurse should initially include information regarding:
    a. the effect that stress has on sexual performance.
    b. the effect of diabetes mellitus on the vascular system.
    c. the link between depression and sexual dysfunction.
    d. sexual dysfunction related to long-term use of insulin.
    B
  3. Upon entering the room of a cognitively impaired older adult patient, the nurse observes that he is exposed and rubbing his genitals. The nurse’s initial concern is to:
    a. alert staff to be aware of this new behavior.
    b. provide the patient with privacy.
    c. assess him for possible pain and fever.
    d. provide a verbal cue for him to stop the behavior.
    C
  4. To effectively assess an older adult patient’s sexual needs, the nurse must initially:
    a. reflect on personal feelings that create barriers to effective communication with the patient.
    b. be familiar with the sexual needs of the older adult population.
    c. assess the patient’s physical capacity to engage in sexual activities.
    d. inform the patient of the personal nature of the detailed questioning this assessment requires.
    A
  5. An older adult patient recovering from a radical prostatectomy is discussing his postsurgical care plan with the nurse when he expresses concern about long-term impotence. The nurse initially responds:
    a. “I’d suggest a consult with a sexuality counselor for you and your partner.”
    b. “When you’ve healed sufficiently, we can discuss prosthetic devices that help.”
    c. “There are medications called phosphodiesterase inhibitors that minimize that problem.”
    d. “While postsurgical erectile dysfunction is likely, it is generally temporary.”
    D
  6. The charge nurse on an extended care unit recognizes an immediate need for additional unit education regarding sexuality and the older adult when overhearing a staff member state:
    a. “I’ve had to tell her to stop touching my breasts twice today.”
    b. “Someone needs to tell him to keep his pants zipped.”
    c. “I realize they have needs, but I’m not sure how to handle that.”
    d. “It’s sad that Alzheimer disease causes them to become sexual perverts.”
    D
  7. An older adult female patient who has multiple sexual partners asks the nurse if the risk for contracting HIV really does increase as we age. The nurse shows the best understanding of this risk when responding:
    a. “Any time one engages in sex with multiple partners, the risk for contracting HIV increases.”
    b. “Changes in vaginal tissue and immune function increase the risk, especially if sex is unprotected.”
    c. “Unless you are engaging in unprotected oral sex, your risk does not increase substantially.”
    d. “Yes, your risk of contracting a sexually transmitted disease (STD) including HIV, dramatically increases as you age.”
    B
  8. Through the open door of the patient’s room, the nurse observes a male patient and his long-term partner in a romantic embrace. The nurse’s priority intervention is directed toward:
    a. reinforcing for the staff the patient’s intimacy needs.
    b. explaining to the patient the challenges that his relationship poses for the staff.
    c. offering to discuss the barriers to intimacy that the patient and his partner face.
    d. quietly closing the door to address the patient’s right to privacy.
    D
  9. The gerontologic nurse wants to begin assessing concerns related to sexuality among the population of patients seen in the clinic. What action by the nurse is best?
    a. Give the patients questionnaires to fill out.
    b. Get permission to discuss sexuality with them.
    c. Tell the patients you are now assessing sexuality.
    d. Ask the patients if they have concerns about sex.
    B
  10. The nursing manager feels that intimacy needs are not being assessed or addressed by the staff on the unit. What action by the manager is best?
    a. Tell the staff sexuality is expected to be assessed.
    b. Provide the staff with education on sexuality.
    c. Obtain tools for staff to use when assessing sexuality.
    d. Allow those with cultural objections to opt out.
    B
  11. The nurse notes the patient’s chart lists “dyspareunia” as a complaint. What teaching does the nurse plan to provide?
    a. Use of water-soluble lubricants
    b. Performing Kegel exercises
    c. Deep breathing and relaxation
    d. Use of antifungal medications
    A
  12. The nurse using the permission, limited information, specific suggestions, and intensive therapy (PLISSIT) model offers specific suggestions when:
    a. Referring the patient to a sex therapist.
    b. Discussing over-the-counter lubricants.
    c. Teaching safer sex practices.
    d. Discussing sexual positioning after hip surgery.
    D
  13. The patient who recently had a radical prostatectomy has the nursing diagnosis of ineffective sexuality patterns. What assessment by the nurse best indicates that the goals for this diagnosis have been met?
    a. Patient states he can live without sex.
    b. Patient says that impotence is temporary.
    c. Patient states his needs are being met.
    d. Patient asks about medication for ED.
    C
  14. An older adult has begun dating after being widowed for many years. The adult confides to the nurse about having several sexual partners. What action by the nurse is best?
    a. Warn the patient that the family may not appreciate the situation.
    b. Teach the patient about safer sexual practices including condoms.
    c. Ask the patient if there are any medical concerns related to sex.
    d. Tell the patient he or she may begin to have feelings of guilt.
    B
  15. What information about sexuality is contrary to research on sexuality in older men?
    a. Erections are not as firm.
    b. It takes longer to obtain erections.
    c. Erectile dysfunction is inevitable.
    d. Ejaculation may not be a strong.
    C
  16. A patient with arthritis has difficulty participating is sex because of joint pain and stiffness. What action by the nurse is best?
    a. Suggest a warm shower prior to sexual activity.
    b. Ask the patient if he or she needs more pain medication.
    c. Explore other ways of expressing sexuality.
    d. Refer the patient to a rheumatologist.
    A
  17. An older male patient is seen in the family practice clinic and tells the nurse he no longer takes his metoprolol (Toprol) because “it interferes with my lifestyle.” What action by the nurse is best?
    a. Warn the patient of the complications of hypertension.
    b. Ask the patient if he can afford the medication.
    c. Tell the patient this drug often causes erectile dysfunction.
    d. Take the patient’s blood pressure and record the findings.
    C
  18. A male patient takes carbamazepine (Tegretol). The spouse reports sexual dysfunction, but the patient adamantly denies he has this problem. What response by the nurse is best?
    a. Telling him, “You need to think of your wife’s needs too.”
    b. Telling him this is common and can be discussed if wanted.
    c. Questioning the patient again about sexual functioning
    d. Telling the doctor to change the patient’s prescription
    B
  19. A patient has many sexual partners but does not use condoms. What action by the nurse is best?
    a. Ask the patient what he or she knows about HIV.
    b. Assess the patient for barriers to using condoms.
    c. Give the patient statistics on HIV in older adults.
    d. Tell the patient safer sex practices should be used.
    B
  20. A recently widowed patient reports new onset of sexual dysfunction. There are no new medications or illnesses. What action by the nurse is best?
    a. Assess the patient’s alcohol intake.
    b. Refer the patient for counseling.
    c. Ask if there are new partners.
    d. Have the patient speak to the doctor.
    A
  21. A patient lives in a long-term care facility and has mild dementia. The patient has been showing interest in another resident. What action by the nurse is best?
    a. Determining if the resident has decision-making capacity.
    b. Refusing to allow the residents to be alone together.
    c. Asking the resident’s family if the relationship is okay.
    d. Providing time for the residents to be together.
    A
  22. The family of a resident in an assisted living facility contacts the director to say they are “appalled” that the resident is allowed to have pornographic magazines in the room. What response by the director is most appropriate?
    a. “We will take those away immediately.”
    b. “Your loved one has the right to have these.”
    c. “How do you know about these magazines?”
    d. “He cannot stay here if he has these in the room.”
    B
  23. The nurse working in long-term care knows there are several barriers to sexual expression for older patients. Which of the following are barriers? (Select all that apply.)
    a. Decreasing desire
    b. Medication side effects
    c. Disease processes
    d. Social circumstances
    e. Increased libido
    A, B, C, D
  24. The nurse is learning about postmenopausal changes that can affect sexuality in women. Which of the following are included? (Select all that apply.)
    a. Shortening of the vagina
    b. Need to void after intercourse
    c. Vaginal dryness
    d. Vaginal irritation needs investigation
    e. Vaginal secretions diminish
    A, B, C, E
  25. An 80-year-old patient has nausea and vomiting related to a gastrointestinal disorder. The nursing intervention most likely to help the patient is to:
    a. offer sips of soda every 15 minutes until more is tolerated.
    b. encourage the patient to lie in a prone position while nauseated.
    c. encourage the intake of high-calorie foods such as milkshakes.
    d. keep the patient on a nothing-by-mouth (NPO) order until the nausea subsides.
    A
  26. When preparing to discharge an older patient with mild dysphagia, the nurse suggests that the patient can minimize symptoms by:
    a. eating small meals every 2 to 3 hours
    b. cutting a sandwich into bite-sized peicees.
    c. eating less but choosing nutrient-dense foods.
    d. drinking thin liquids instead of eating solids.
    A
  27. An older patient is being taught about oral gingivitis. The nurse has included instruction about maintaining an oral hygiene program, signs and symptoms of oral infection, and the importance of maintaining regular professional dental care. What important teaching has been missed?
    a. Information about when to have teeth removed and dentures made
    b. The necessity of using a hard-bristled toothbrush to maintain cleanliness
    c. The importance of avoiding meat and caffeine-containing products
    d. The importance of adequate nutrition for maintaining oral health
    D
  28. An older adult patient has recently diagnosed gastritis. What statement made by this patient indicates the need for further teaching?
    a. “The abdominal pain is caused by acidity.”
    b. “I should avoid taking aspirin.”
    c. “Smoking has little effect on my stomach problem.”
    d. “I could develop pernicious anemia.”
    C
  29. The nurse is planning to teach an older patient about diverticulitis. What topic does the nurse include?
    a. Dietary fiber and fluids will reduce the symptoms.
    b. It is unusual to see diverticula in older persons.
    c. Abdominal cramping and severe diarrhea should be reported.
    d. Diverticulosis rarely reoccurs once it has been treated.
    A
  30. The nurse caring for an older adult diagnosed with hyperplastic polyps instructs him that:
    a. this type of polyp is rarely malignant and usually does not require treatment.
    b. follow-up colonoscopies should be performed every 3 to 4 years after diagnosis.
    c. stool should be guaiac tested every week for 1 year after diagnosis.
    d. the presence of blood in the stool requires a repeat sigmoidoscopy examination.
    D
  31. An older adult reports chronic constipation. When asked why this problem has gotten worse with age, the nurse responds:
    a. “As we age, our bodies require more fiber to bring about healthy bowel function.”
    b. “We need to discuss the proper use of laxatives to minimize constipation.”
    c. “It’s possible that you have lost the ability to feel when you need to move your bowels.”
    d. “Aging brings about decreased gastric motility that often results in constipation.”
    A
  32. An older adult patient reports episodes of fecal incontinence. The nurse provides appropriate emotional support when assuring the patient that:
    a. it is a common problem that occurs in response to normal aging.
    b. the incontinence is rarely a result of a serious problem.
    c. disposable absorbent underwear will help manage the problem.
    d. the problem generally responds well to bowel control programs.
    D
  33. The daughter of a dependent older patient reports to the nurse that the patient requires regular soapsud enemas to manage chronic constipation. The nurse responds that:
    a. an alternative management technique should be discussed.
    b. enemas are generally the most effective interventions for the older adult.
    c. chronic constipation is best managed with oral medications.
    d. her mother’s diet is the most likely cause of the constipation.
    A
  34. A 74-year-old adult is experiencing dumping syndrome after gastric resection surgery. The nurse caring for the patient instructs the patient to:
    a. stop smoking.
    b. abstain from beverages that contain caffeine.
    c. eat three low-carbohydrate meals daily.
    d. drink only between meals.
    D
  35. A 68-year-old patient is reporting symptoms that suggest a peptic ulcer. The nurse asks the patient if:
    a. the pain exacerbates when he eats fatty food.
    b. there is a family history of peptic ulcers.
    c. he smokes either cigars or cigarettes.
    d. he uses acetaminophen (Tylenol) for minor pain.
    B
  36. What dietary suggestion does the nurse give the older patient to manage age-related changes in taste?
    a. Add more salt to foods.
    b. Use a salt substitute.
    c. Add sugar when possible.
    d. Use a variety of herbs.
    D
  37. An older patient has been admitted with nausea and vomiting. What assessment takes priority?
    a. Respiratory system
    b. Urine output
    c. Blood pressure and pulse
    d. Skin integrity
    C
  38. A patient is admitted with copious diarrhea. The patient is dizzy when standing, and skin assessment reveals abrasions around the perianal area. What assessment finding demonstrates that goals for the priority nursing diagnosis have been met?
    a. Perianal skin abrasions are smaller in size.
    b. Patient does not fall while hospitalized.
    c. Patient sits up without dizziness.
    d. Patient is able to tolerate oral fluids.
    D
  39. A patient has pernicious anemia. What action by the patient and family indicates teaching for this condition has been effective?
    a. Proper administration of oral vitamin B12
    b. Correct technique for intramuscular (IM) injections
    c. Choosing aspirin over ibuprofen (Motrin) for pain
    d. Preparing a low-carbohydrate meal
    B
  40. A patient is admitted with infectious diarrhea. What action by the nurse is best?
    a. Place the patient in contact precautions.
    b. Place the patient on droplet precautions.
    c. Use standard precautions to care for the patient.
    d. Prepare staff to take prophylactic antibiotics.
    A
  41. An older patient with hepatitis has pruritus. What advise does the nurse provide this patient?
    a. Keep your fingernails cut short.
    b. Use diphenhydramine (Benadryl).
    c. Hot soapy showers will help.
    d. Butter is a good home remedy for itching.
    A
  42. An older patient had a stroke several months ago. The patient begins to exhibit dysphagia. What action by the nurse is best?
    a. Consult with a speech-language therapist.
    b. Discuss the need for enteral feedings.
    c. Provide the patient swallowing exercises.
    d. Arrange for a physical exam.
    D
  43. An older patient is having a colostomy as part of surgery for colon cancer. What assessment by the nurse is most important in this patient?
    a. Manual dexterity
    b. Body image
    c. Fear of dying
    d. Fluid volume status
    A
  44. A 70-year-old patient has lost 25 pounds since being diagnosed with hepatitis A. To best manage the patient’s anorexia, what does the nurse suggest? (Select all that apply.)
    a. A protein powder supplement added to liquids
    b. Several meals eaten during the day
    c. Megavitamins that include iron and folic acid
    d. A dietary assessment to identify favorite foods
    e. A high-carbohydrate, low-fat diet
    B, E
  45. Because of a knowledge of age-related changes in the gastrointestinal system, the nurse encourages regular screenings for which of the following? (Select all that apply.)
    a. Osteoporosis
    b. Vitamin B deficiency
    c. Pernicious anemia
    d. Enlarged liver
    e. Iron deficiency anemia
    A, B, C, E
  46. An older adult patient is being evaluated for a possible duodenal ulcer (DU). Which of the following assessments supports the diagnoses? (Select all that apply.)
    a. Passing a moderate amount of dark reddish-brown stool
    b. Reporting a stabbing pain in the epigastric region
    c. Asking for “some crackers to stop my stomach cramps”
    d. Reporting the need to take antacid tablets “most days”
    e. Having a rigid abdomen about 2 hours after eating
    A, C, D
  47. An 82-year-old patient with a history of chronic heart and respiratory problems asks the nurse, “What can I do to keep my hemorrhoids from acting up?” Which of the following responses made by the nurse are appropriate? (Select all that apply.)
    a. Ask if he experiences constipation with any regularity.
    b. Encourage him to increase his fluid intake to 2000 mL daily.
    c. Suggest he eat more whole grains and fresh fruits.
    d. Discuss how he should include a walk into his daily routine.
    e. Ask if he has a history of rectal bleeding.
    A, C, D, E
  48. To best identify a risk for injury in an older adult patient, the nurse assesses for:
    a. decreased muscle mass in the legs.
    b. history of falls.
    c. hyperextension of the spine.
    d. decreased bone density.
    B
  49. A nurse is caring for the older patient who had knee replacement surgery 8 days ago. What assessment by the nurse is most important?
    a. Determining whether the patient has sensation to the foot
    b. Asking the patient to rate his or her current pain.
    c. Observing the incision site for redness or drainage.
    d. Monitoring the calf circumference on the affected side
    D
  50. A patient had hip replacement surgery. What intervention is most appropriate to prevent dislocation?
    a. Instruct staff to use a fracture pan when the patient needs to toilet.
    b. Administer ordered pain medication prior to turning.
    c. Elevate the patient’s knee on the affected side with a pillow.
    d. Apply an abduction splint while the patient is in bed.
    D
  51. An older adult patient who has experienced a left knee replacement asks the nurse, “When will I be back to normal?” The nurse responds that:
    a. “What did the surgeon tell you about function after the surgery?”
    b. “Normal means different things to different people.”
    c. “You should be back to normal after 6 to 8 weeks.”
    d. “Surgery will improve your mobility but I’m not sure about being normal.”
    A
  52. The nurse caring for an older patient diagnosed with spinal stenosis encourages the patient to notify her physician if she experiences:
    a. sharp pain when turning her neck side-to-side.
    b. stabbing pain in her lower back.
    c. a cramping sensation in her feet.
    d. a burning sensation in either one or both legs.
    D
  53. The nurse is assisting a 65-year-old female patient with planning an exercise program to prevent osteoporosis. The nurse shows an understanding of appropriate exercise when stating:
    a. “The local gym offers aerobics for seniors on Tuesday and Thursday evenings.”
    b. “Bicycling along the park’s 2-mile trail twice a week would be ideal.”
    c. “Do you have a friend who would walk with you for 30 to 60 minutes?”
    d. “Are you aware that rowing is an excellent exercise for strengthening bone?”
    C
  54. An older adult patient has been casted for a fractured left wrist. Which action by the nurse takes priority?
    a. Assessing capillary refill in the nail beds of the fingers of the left hand.
    b. Instructing the patient on how to effectively rate pain on the pain scale.
    c. Teaching the patient to wrap the cast in plastic when the patient showers.
    d. Providing the patient with a protein-enriched milkshake as a bedtime snack.
    A
  55. An older adult patient has been admitted to the hospital with suspected Paget disease. What clinical manifestation will help the nurse differentiate Paget disease from other types of musculoskeletal diseases?
    a. Red, swollen upper and lower extremity joints
    b. Pain on awakening that subsides with activity
    c. Ataxia or mild hearing loss
    d. Back deformity in the absence of pain
    C
  56. An older confused patient is recovering from a stage IV sacral pressure ulcer. The nurse shows an understanding of this patient’s risk for developing osteomyelitis by:
    a. adhering to sterile technique when changing the wound’s dressing.
    b. assessing and documenting the patient’s vital signs regularly.
    c. managing the patient’s antibiotic therapy as prescribed.
    d. ensuring that the patient’s diet includes sufficient protein.
    A
  57. An older adult is diagnosed with rheumatoid arthritis. When discussing exercise with the patient, the nurse makes the greatest positive impact on the patient’s quality of life when stating:
    a. “Exercising will be important to the flexibility of your joints.”
    b. “It seems to help if you have someone to exercise with.”
    c. “I’ll provide you with a list of gyms where you can exercise.”
    d. “Let’s discuss ways for you to exercise your joints.”
    D
  58. The nurse is caring for an older adult patient prescribed allopurinol (Zyloprim). What action by the nurse is best?
    a. Offering fresh, cold water frequently during the day
    b. Monitoring temperature every 4 hours
    c. Ensuring sufficient protein intake
    d. Assessing for depression symptoms daily
    A
  59. A patient is being dismissed from the emergency department with an arm cast. What statement by the patient indicates more teaching is needed?
    a. “I will keep the cast clean and dry.”
    b. “I will wiggle my thumb and fingers often.”
    c. “I will elevate my arm on two pillows.”
    d. “I can use a hanger to scratch under the cast.”
    D
  60. An older patient is in the family practice clinic reporting increasing joint pain, anorexia, and low-grade fever. The patient has a history of osteoarthritis. What action by the nurse is best?
    a. Document the findings on the patient’s chart.
    b. Assess for joint deformities and nodules.
    c. Tell the provider the patient needs more pain medicine.
    d. Encourage the patient to ask for physical therapy.
    B
  61. A patient’s chart contains an assessment of tophi and podagra. What medication does the nurse plan to educate the patient on?
    a. Allopurinol (Zyloprim)
    b. Colchicine (Colcrys)
    c. Levadopa-carbidopa (Sinemet)
    d. Ibuprofen (Motrin)
    A
  62. A nurse works in a long-term care facility where many of the residents have osteoporosis. For which resident would alendronate (Fosamax) be contraindicated?
    a. A patient on a continuous tube feeding
    b. A wheelchair-bound patient
    c. A patient over the age of 85
    d. A male patient
    A
  63. A patient is scheduled to have a lower extremity amputation. What action by the nurse takes priority?
    a. Discuss stump management and prostheses.
    b. Ensure informed consent is on the chart.
    c. Determine the patient’s goal for pain control.
    d. Administer the preoperative antibiotic.
    B
  64. A patient has just arrived in the postanesthesia care unit after a below-the-knee amputation. What assessment takes priority?
    a. Surgical dressing
    b. Level of pain
    c. Pulse and blood pressure
    d. Airway
    D
  65. After a below-the-knee amputation, a patient has disturbed body image. What action by the patient indicates movement toward resolution of this diagnosis?
    a. The patient names his stump “Pete.”
    b. The patient attends physical therapy.
    c. The patient begins to change dressings.
    d. The patient asks questions about prosthetics.
    C
  66. A patient has polymyalgia rheumatica. When teaching about medications, what information does the nurse provide?
    a. “Take the full dose of antibiotics even if you are feeling better.”
    b. “You need to remain upright 1 hour after taking the medication.”
    c. “Stay away from large crowds and avoid people who are sick.”
    d. “Do not drink alcohol while taking this medication.”
    C
  67. A patient has onychomycosis. The nurse should anticipate educating the patient on which of the following drugs?
    a. Clotrimazole (Lotrimin)
    b. Terbinafine (Lamisil)
    c. Itraconazole (Sporanox)
    d. Methylprednisolone (Solu-Medrol)
    A
  68. The nurse conducting a community-screening event for osteoporosis knows that which woman is at highest risk?
    a. A slender 84-year-old Asian who smokes
    b. A heavy set 65-year-old Caucasian
    c. A 75-year-old taking a steroid “burst”
    d. A 68-year-old African American who consumes one drink a day
    A
  69. To assess for osteoarthritis in an older adult patient, the nurse asks which of the following questions? (Select all that apply.)
    a. “Do you have pain in your finger joints?”
    b. “Do your knees crackle when you bend down?”
    c. “Does you get dizzy when you turn your head?”
    d. “Does it hurt when you get up from a chair?”
    e. “Does your back creak when you bend over?”
    A, B, D, E
  70. The nurse working with older adults knows which facts about age-related musculoskeletal changes? (Select all that apply.)
    a. Muscle mass decreases, causing atrophy.
    b. Myocytes are replaced by fibrous tissue.
    c. Vertebral spaces enlarge with fluid retention.
    d. Posture and gait change, leading to fall risk.
    e. Men become bowlegged and waddle.
    A, B, D
  71. When caring for older adults, the nurse expects to encounter the normal urinary age-related outcome of:
    a. urinary incontinence.
    b. low-grade bladder infection.
    c. nocturia.
    d. urinary residual volume.
    C
  72. An 87-year-old patient has suddenly become incontinent. What should the nurse’s first action be?
    a. Review the patient’s record for medications that may be causing urinary incontinence.
    b. Seek an order for an indwelling urinary catheter to prevent skin breakdown.
    c. Limit the patient’s fluid intake to reduce the feeling of having to void so often.
    d. Teach the patient to void every 2 hours when awake during the day or night.
    A
  73. An older adult patient reports “losing urine” when she bends over or gets out of a chair. What type of incontinence does the nurse plan interventions for?
    a. Overflow
    b. Urge
    c. Functional
    d. Stress
    D
  74. When assessing the patient for urinary incontinence, which patient symptom best supports the nursing diagnosis of overflow incontinence?
    a. “I have small accidents ever since I developed a cystocele.”
    b. “It burns so badly after I urinate that I hold it as long as I can.”
    c. “I can’t make it to the toilet when I feel the need to urinate.”
    d. “I lose small amounts of urine when I sneeze or laugh hard.”
    A
  75. An older cognitively impaired adult patient is being discharged to a daughter’s home. The nurse knows continued success of the patient’s bladder training for urinary incontinence primarily rests on the:
    a. patient’s ability to follow instructions.
    b. severity of the impairment of the urinary sphincter.
    c. patient’s ability to sense the need to urinate.
    d. daughter’s ability to support the training.
    D
  76. An older adult patient is hospitalized for after an automobile crash. The nurse recognizes symptoms suggestive of an upper urinary tract (UTI) infection when the patient:
    a. voids 100 mL of urine over a 3-hour period of time.
    b. is not able to state where he is or what day it is.
    c. has an elevated red blood cell (RBC) count.
    d. reports burning when he urinates.
    B
  77. An older adult woman has a resistant strain of pneumonia. To best minimize her risk of developing acute renal failure, the nurse:
    a. monitors the patient’s serum blood urea nitrogen (BUN) levels via diagnostic laboratory work.
    b. helps the patient select low-sodium foods from her daily menu.
    c. measures and records the patient’s urinary output.
    d. chooses an analgesic other than ibuprofen (Motrin).
    D
  78. An older patient is admitted with possible chronic renal failure (CRF). Which lab value does the nurse notify the physician about as a priority?
    a. Increased calcium level
    b. Increased red blood cells
    c. Decreased BUN level
    d. Decreased creatinine clearance level
    D
  79. The nurse is admitting an older patient with benign prostate hyperplasia (BPH). The nurse’s priority questioning focuses on:
    a. family history of prostate disorders.
    b. onset of symptoms.
    c. psychosocial impact of the diagnosis.
    d. typical urinary voiding patterns.
    D
  80. A patient in a long-term care facility has incontinence. What assessment by the nurse is most important before designing interventions for this problem?
    a. Cognitive status
    b. Ambulatory status
    c. Cardiovascular status
    d. History of childbirth
    A
  81. A male patient has benign prostatic enlargement. He is at risk for what type of acute kidney injury?
    a. Prerenal
    b. Intrarenal
    c. Postrenal
    d. Combined form
    C
  82. A patient has a history of smoking and now has painless hematuria. After a workup, the patient is told the diagnosis of bladder cancer. What action by the nurse is most important?
    a. Allow the patient to verbalize feelings.
    b. Educate the patient on care of an ileal conduit.
    c. Teach the patient how to manage nausea.
    d. Offer a social work referral to complete a living will.
    A
  83. A patient being treated for prostate cancer calls the clinic to report severe back pain. What action by the nurse is best?
    a. Advise the patient to take his pain medication.
    b. Tell the patient to come in to the clinic today.
    c. Make an appointment for the patient next week.
    d. Encourage the patient to rest and use moist heat.
    B
  84. A male patient reports difficulty starting a urine stream and a weak urine flow. When prompted to seek medical attention, the patient asks why, as it’s “obviously” benign prostatic hypertrophy. What response by the nurse is best?
    a. “You never know; it could be cancer.”
    b. “You should have any change checked out.”
    c. “Only the physician can make a diagnosis,”
    d. “BPH and prostate cancer have similar symptoms.”
    D
  85. A patient treats chronic kidney failure with peritoneal dialysis. The patient notes the fluid draining out of the abdomen is cloudy and foul smelling. What action by the nurse is best?
    a. Assess the patient for other signs of infection.
    b. Document the findings in the patient’s chart.
    c. Call the rapid response team immediately.
    d. Request a prescription for an antibiotic.
    A
  86. A patient is scheduled to have surgery for prostate cancer in a few weeks. What action by the nurse is most important?
    a. Discuss options and their effect on sexuality.
    b. Ensure the patient has advance directives.
    c. Offer the patient a tour of the operating room.
    d. Determine if the patient prefers outpatient surgery.
    A
  87. A patient asks how elevating the legs at night will decrease nocturia. What is the nurse’s best response?
    a. All that fluid gets into circulation before you go to bed.
    b. Decreased swelling makes it easier to ambulate at night.
    c. It won’t help; that’s an old wives’ tale you heard.
    d. This measure helps dehydrate you before bedtime.
    A
  88. What information does the nurse share with the student about normal age-related changes in the kidneys?
    a. Renal mass increases.
    b. The glomerular filtration rate decreases.
    c. Poor renal function occurs after age 65.
    d. There are no real age-related changes.
    B
  89. A nurse is assessing an older patient for the possible cause of his acute urinary incontinence. Which actions by the nurse are most important? (Select all that apply.)
    a. Asking when his last normal bowel movement was
    b. Monitoring his intake and output
    c. Determining if he has been screened for prostatic hypertrophy
    d. Asking him if he awakens during the night to urinate
    e. Measuring his abdominal girth
    A, C, D
  90. When preparing educational information regarding benign prostatic hyperplasia (BPH) for a group of older male patients, the nurse includes which of the following? (Select all that apply.)
    a. Eighty percent of males experience the symptoms by age 80.
    b. Diabetes mellitus is a risk factor.
    c. It is only as the prostate enlarges that symptoms occur.
    d. The resulting urinary retention can cause urinary tract infections.
    e. Symptoms are a result of urethral obstruction.
    A, C, D, E
  91. An older adult patient’s urinary incontinence is being addressed by prompted voiding. The nurse instructs all ancillary staff to do which of the following? (Select all that apply.)
    a. Provide only minimal fluids after 7 PM.
    b. Keep the patient on the toilet until voiding occurs.
    c. Allow the patient to void at times other than those scheduled.
    d. Offer toileting during the night only when the patient is awake.
    e. Encourage the patient to toilet himself.
    C, D
  92. A patient has a glomerular filtration rate (GFR) of 19 mL/min/1.73m2. What assessment findings correlate with this condition? (Select all that apply.)
    a. Fatigue
    b. Weakness
    c. Edema
    d. No specific symptoms
    e. Headaches
    A, B, C
  93. The nurse working in the gerontology clinic understands which facts related to incontinence? (Select all that apply.)
    a. It is a normal age-related change.
    b. It is an independent predictor of nursing home admission.
    c. It contributes to falls and injuries.
    d. It can disrupt sleep.
    e. It can lead to urinary tract infections.
    B, C, D, E
  94. An older adult is experiencing age-related postural hypotension and he fears “something is really wrong” because he is the only one in his social group experiencing the problems. The nurse responds:
    a. “Don’t be concerned; just be very careful about your risk for falling.”
    b. “You have had very thorough testing, so don’t worry about it being serious.”
    c. “It’s just a matter of time before they too have to watch not to get up too quickly.”
    d. “You just don’t have the compensating mechanisms of your friends.”
    D
  95. What education by the nurse is most important to address age-related changes to the senses?
    a. Installing auditory smoke alarms
    b. Having regular eye checkups
    c. Being aware that hearing acuity decreases with age
    d. Checking the expiration dates on foods such as dairy
    A
  96. The nurse is conducting an admission assessment on a mildly confused older patient. The nurse best assures an accurate history by first:
    a. scoring the client’s cognitive responses.
    b. focusing on the client to respond.
    c. directing the questions to both patient and family.
    d. arranging a Mini-Mental State Examination (MMSE).
    C
  97. A nurse is caring for an older patient diagnosed with acute depression. What action by the nurse is most important to help prevent delirium in this patient?
    a. Reorienting the patient to the day, time and place frequently
    b. Being physically present to help the patient with eating meals
    c. Providing the patient with opportunities to discuss depression
    d. Administering antidepressive medication as prescribed
    B
  98. When assessing an older patient displaying symptoms of delirium, the nurse focuses the assessment on:
    a. the degree and duration of the symptoms.
    b. the amount of self-care deficiency the symptoms cause.
    c. identifying processes that commonly result in the symptoms.
    d. physiologic dysfunction resulting from the symptoms.
    C
  99. An 80-year-old patient is exhibiting signs of dementia representative of Alzheimer disease (AD). The nurse supports that possibility when determining that the patient:
    a. experienced a gastric resection several years ago.
    b. traveled often to third world countries.
    c. was employed as a steelworker for 40 years.
    d. has a history of viral encephalitis.
    D
  100. When planning care for the older adult with advanced dementia, the nurse recognizes that the best way to implement reality orientation is to:
    a. place printed labels on important items, such as the telephone.
    b. place a clock and calendar in the patient’s immediate environment.
    c. use hand gestures instead of verbal communications to demonstrate meaning.
    d. show the patient a picture of a toothbrush when it is time for oral hygiene.
    D
  101. A 73-year-old patient diagnosed with vascular dementia is admitted for exacerbation of asthma. The patient has been treated for 2 years with benzodiazepines to manage her increasingly aggressive behavior. The nurse’s initial response is to:
    a. identify the patient as being at high risk for falls.
    b. monitor the patient for signs of benzodiazepine withdrawal.
    c. notify the admitting physician immediately.
    d. place the patient on strict intake and output.
    C
  102. Which of the following statements, when made by family members caring for an older patient with dementia, indicates peaceful acceptance of the situation?
    a. “I’m so pleased that Mother had a good day today. I’m really very hopeful.”
    b. “The hospice nurses are so helpful when I need time for myself.”
    c. “I promised Mother I would take care of her and I’ll never leave her.”
    d. “It’s the least I can do for Mother since she cared for us all these years.”
    B
  103. The son of a patient with possible Alzheimer disease (AD) asks the nurse if there is a diagnostic test that can confirm the diagnosis. The nurse responds that:
    a. an electroencephalogram is often very useful in diagnosing AD.
    b. a positron emission tomography (PET) scan is a cheap but dependable tool.
    c. magnetic resonance imaging (MRI) is often ordered for that purpose.
    d. postmortem autopsy is the only definitive diagnostic tool.
    D
  104. An 89-year-old diagnosed with dementia was until recently responding well to cognitive cueing techniques. The nurse shows an understanding of dementia when sharing with staff that:
    a. “We will implement new interventions that address the disease’s progression.”
    b. “It’s important that we frequently recue the patient to improve her quality of life.”
    c. “The patient’s family needs to be made aware of this decline.”
    d. “This poor response to cueing is likely a result of advanced aging.”
    A
  105. An 80-year-old patient who is experiencing symptoms of depression and anxiety is reluctant to comply with the prescribed treatment plan. The nurse initially addresses the issue with the patient by asking:
    a. “How do you feel about how others view your mental health problem?”
    b. “Are you concerned about paying for your psychiatric medications?”
    c. “Did you know that depression is common among people your age?”
    d. “Do you have any questions about your the mental health treatment plan?”
    A
  106. An older adult patient being treated for chronic obstructive pulmonary disease (COPD) is exhibiting signs of memory loss and confusion. In planning his care, the nurse should give priority to:
    a. obtaining an order for a pulmonary function test (PFT).
    b. determining the potential of a possible adverse drug reaction.
    c. reorienting the patient to time, place, and person frequently.
    d. assessing for a family history of dementia.
    B
  107. The nurse caring for an older adult patient recovering from cardiac surgery recognizes that it is most appropriate to assess this patient for mental health problems because:
    a. cardiac surgery often results in anxiety-related issues.
    b. untreated depression can contribute to the patient’s morbidity risks.
    c. many in this age cohort have undiagnosed depression.
    d. hospitalization is both anxiety and depression inducing.

ANS: B

  1. When planning care for the older adult being treated for depression, the nurse addresses the patient’s tertiary intervention needs best by:
    a. helping the patient to identify the early symptoms of depression.
    b. helping the patient deal with the physical symptoms of depression.
    c. discussing with the patient how to implement new coping skills.
    d. educating the patient about the importance of being drug compliant.
    C
  2. To best advocate for an older adult patient being prescribed medication to control newly observed signs of confusion and aggressive behavior, the nurse:
    a. initiates an assessment to determine possible underlying causes of the behavior.
    b. contacts family to inform them of the new medication therapy being planned.
    c. discusses possible nonpharmaceutical treatments with the physician.
    d. documents a detailed description of the behaviors before administering the drugs.
    A
  3. An older adult has been recently diagnosed with type 2 diabetes and mild retinal deterioration. To best address the patient’s potential for developing situation depression, the nurse:
    a. assesses the patient’s coping skills.
    b. Encourages the patient to participate in a depression support group.
    c. assesses the patient’s ability to manage the symptoms.
    d. educates the family on early signs of depression.
    A
  4. An older adult has a medical condition that has required hospitalization at a facility far from home and family. To best minimize the patient’s risk for depression, the nurse:
    a. keeps the patient informed of the expected discharge date.
    b. offers to help the patient telephone family members each evening.
    c. reassures the patient that early discharge is a nursing goal.
    d. encourages the patient to place family photographs around the room.
    B
  5. An older patient is anxious about an upcoming diagnostic test and requests something to calm the nerves. To best address the patient’s need, the nurse prepares to administer a PRN dose of:
    a. clonazepam (Klonopin).
    b. diazepam (Valium).
    c. chlordiazepoxide (Librium).
    d. lorazepam (Ativan).
    D
  6. While collecting a health history for an older adult patient, the nurse learns that the patient had been prescribed Elavil 3 weeks ago and wants to stop taking it because “It didn’t make me feel any better.” In response to this information, the nurse shares with the patient that:
    a. sudden withdrawal is likely to cause a hypertensive crisis.
    b. depression seldom improves without medication.
    c. realistically it will take longer for the patient to feel an improvement.
    d. in time, people adjust to the side effects.
    C
  7. The nurse familiar with the old adult population recognizes that the patient who has the greatest potential for successfully committing suicide is the:
    a. 63-year-old Asian female.
    b. 86-year-old Caucasian male.
    c. 76-year-old Hispanic female.
    d. 67-year-old African-American male.
    B
  8. A 65-year-old adult who recently lost his spouse is admitted to the hospital after a failed suicide attempt. He presents with a sad affect and is reluctant to interact within the milieu. The nursing diagnosis with priority is:
    a. ineffective coping related to recent loss.
    b. hopelessness related to death of spouse.
    c. risk for loneliness related to loss of spouse.
    d. risk for self-directed violence related to depression.
    D
  9. The nurse is caring for a severely depressed older patient. To best effect change in the patient’s emotional state, the nurse’s initial goal is to:
    a. plan interventions that will enhance the patient’s self-esteem.
    b. introduce the patient to new coping skills.
    c. assess the patient’s potential to self-harm.
    d. develop a therapeutic nurse-patient relationship.
    D
  10. An older adult is hospitalized for treatment of a mental health disorder and is prescribed clomipramine (Anafranil). The nurse documents that the medication is having the desired effect when the patient:
    a. begins sleeping 8 hours per night.
    b. engages in fewer ritualistic behaviors.
    c. reports fewer episodes of nervousness.
    d. exhibits no delusionary thinking.
    B
  11. To help manage the potential side effects of prescribed antipsychotic medications, amantadine (Symmetrel) may be prescribed. Which statement best indicates that the nurse understands the appropriateness of this medication for the older adult patient?
    a. “This medication produces few anticholinergic effects.”
    b. “Symmetrel is an effective dopamine agonist.”
    c. “Extrapyramidal symptoms are best controlled by Symmetrel.”
    d. “Older patients seem to have the fewest side effects on this medication.”
    A
  12. A 72-year-old is prescribed lithium. The nurse educates the patient on the importance of biannual evaluation of which of the following? (Select all that apply.)
    a. Renal function
    b. Serum glucose level
    c. Liver function
    d. Thyroid function
    e. Red blood cell count
    A, C, D
  13. A 78-year-old patient was admitted with dehydration. The nurse assesses and documents observations that support a finding of dementia. Which of the following observations are related to dementia? (Select all that apply.)
    a. Forgetting what she ate for lunch today
    b. Crying frequently when alone
    c. Inability to find her way back to her room from the dayroom
    d. Being impatient with the nursing staff for not closing her door
    e. Repeatedly asking to call her son
    A, C, D, E
  14. The nurse is caring for an older adult patient admitted to the hospital. What assessment findings place the patient at risk for developing delirium during the hospitalization? (Select all that apply.)
    a. The patent takes medications to manage several chronic illnesses.
    b. The patent has a history of urinary tract infections.
    c. The patent is in cancer remission.
    d. The patent has recently been eating poorly.
    e. The patent experienced a mild heart attack 2 years ago.
    A, B, D
  15. A home care nurse is visiting a patient with moderate cognitive impairment from Alzheimer disease. The patient’s partner expresses concern about difficulty getting the patient “to eat properly.” The nurse suggests which of the following? (Select all that apply.)
    a. Serving meals at the same time each day
    b. Offering liquids in place of solid foods when possible
    c. Offering a calorie-dense snack at bedtime
    d. Cutting food into bite-sized pieces that will fit into the patient’s hand
    e. Asking the patent to identify favorite foods
    A, C, D
  16. The nurse explains that the plan of care for an older adult patient with seborrheic dermatitis of the scalp should include:
    a. cleaning lesions with a weak hydrogen peroxide solution daily.
    b. cleaning the scalp with a low-dose steroidal shampoo.
    c. applying hydrocortisone 10% to scalp lesions.
    d. applying selenium shampoo to the scalp.
    D
  17. An older adult patient reports simple xerosis with mild pruritus. The nurse educates her on the importance of:
    a. applying a lanolin-rich cream and avoiding scratching the areas.
    b. taking warm baths and gently rubbing of affected areas with a terrycloth towel.
    c. minimizing ingestion of fried foods and use of an antihistamine cream.
    d. avoiding bath oils and allowing the skin to air-dry after bathing.
    A
  18. The nurse plans to assess for candidiasis as a priority intervention for a:
    a. 60-year-old with a history of bacterial pneumonia.
    b. 72-year-old incontinence of urine and feces.
    c. 58-year-old with a casted left foot.
    d. 90-year-old receiving antihypertensives.
    B
  19. An 87-year-old patient developed herpes zoster after surgical repair of a hip fracture. The priority nursing diagnosis is:
    a. impaired skin integrity related to immunologic deficit.
    b. self-care deficit related to severe pain and fatigue.
    c. risk for infection related to impaired skin integrity.
    d. pain related to inadequate pain relief from analgesia.
    C
  20. The presence of which skin assessment finding, if noted on an older adult patient, should cause the nurse to suspect a premalignancy?
    a. Numerous small red papules on the chest and back
    b. An oozing, rough, reddish macule on the ear
    c. An irregularly shaped mole on the face or shoulders
    d. Brown, greasy lesions on the neck
    B
  21. An older adult patient has been taught measures to prevent the development of skin cancer. Which statement, if made by the patient, indicates the need for more teaching?
    a. “I will certainly miss my vegetable and flower gardening.”
    b. “I should buy a sunscreen with an SPF of 15 or higher.”
    c. “Now I have a good excuse to wear the straw hat my spouse hates.”
    d. “My cool long-sleeved shirts will work just fine while I’m golfing.”
    A
  22. When assessing the older adult patient’s skin for indications of melanoma, the nurse should inspect for a(n):
    a. thick, adherent scale with a soft center.
    b. small, inflamed lesion that bleeds easily.
    c. irregularly shaped multicolored mole.
    d. small, purple, hard nodule beneath the skin surface.
    C
  23. An older adult patient newly diagnosed with peripheral vascular disease (PVD) is being educated on the possibility of developing a foot ulcer. What assessment finding indicates the patient may have an ulcer resulting from this disease?
    a. Deep, necrotic, and painless sore
    b. Shiny, dry, cyanotic skin surrounding the ulcer
    c. Ulcer appears shallow, crusty with warm skin
    d. Sore that has dull pain and is oozing
    B
  24. The patient has an open, draining wound on the lower medial aspect of the right leg. The skin surrounding the wound is reddish brown with surrounding erythema and edema. Based on this information, the nurse edits the patient’s care plan to include impaired skin integrity:
    a. related to altered venous circulation.
    b. peripheral related to arterial insufficiency.
    c. related to diabetic neuropathy.
    d. open wound related to pressure ulcer.
    A
  25. When assessing for squamous cell cancer (SCC), a home health nurse is particularly concerned about a suspicious lesion on the:
    a. leg of a 60-year-old Asian female.
    b. neck of a 73-year-old Hispanic female.
    c. Lower lip of a 70-year-old African American male.
    d. back of a 90-year-old Caucasian male.
    C
  26. A 65-year-old man is seen in the outpatient clinic for treatment of psoriasis. The nurse educates the patient to the possibility of developing:
    a. alopecia.
    b. orange-tinged urine.
    c. yellow-brown nails.
    d. cherry angiomas.
    C
  27. The nurse of a bedridden 74-year-old woman is evaluating whether the family members understand how to position the patient correctly. The nurse is confident the family is capable of effective positioning when it is observed that the patient’s:
    a. arms and legs are supported on two pillows.
    b. position is changed at least every 2 hours.
    c. neck is hyperflexed.
    d. elbows rest on the bed.
    B
  28. An older diabetic patient reports a candidiasis infection. When asked, the patient states all blood sugars have been within the target range. What action by the nurse is best?
    a. Facilitate having a hemoglobin A1c drawn.
    b. Teach the patient preventive measures.
    c. Teach the patient about the side effects of medications.
    d. Review the patient’s medication history.
    A
  29. An older patient has been treated for a small basal cell carcinoma on the face. What assessment finding indicates to the nurse that the goals for a priority diagnosis have been met?
    a. The patient verbalizes relief there is no metastasis.
    b. Wound edges are approximated without redness.
    c. The patient expresses satisfaction with the cosmetic outcome.
    d. The patient relates the need for proper sun protection.
    B
  30. In creating community education on various types of skin cancer, the nurse places the highest priority on early diagnosis of melanoma because:
    a. it accounts for the largest number of mortalities.
    b. extensive surgery can be avoided if caught early.
    c. once it has spread there is no chance of curing it.
    d. it is the most commonly occurring skin cancer.
    A
  31. An older diabetic patient has impaired mobility and decreased vision. The nurse examines the patient’s feet at each clinical visit. The patient asks why this is necessary. What response by the nurse is best?
    a. “It’s part of our diabetic clinic visit protocol.”
    b. “You may not be able to see a sore on your feet.”
    c. “Limited mobility may keep you from checking your feet.
    d. “You may get an ulcer and not be able to feel it.”
    D
  32. For which patient does the nurse add compression therapy to the nursing care plan?
    a. Taut, white, shiny skin
    b. Faint pedal pulses
    c. Brownish skin and edema
    d. Large ulcer with skin graft
    C
  33. The nurse assesses a patient using the Braden scale. The patient scores a 13. What action is most important to add to the patient’s care plan?
    a. Encourage high-protein meals and snacks
    b. Turn the patient every to 2 hours
    c. Assess the patient’s skin daily
    d. Monitor patient’s prealbumin weekly
    B
  34. A patient has a purulent, foul-smelling leg wound. What wound care practice is most appropriate?
    a. Leave the wound open to the air.
    b. Administer systemic antibiotics.
    c. Cleanse the wound with diluted povidone iodine.
    d. Prepare the patient for operative débridement.
    C
  35. A patient has a wound that is a shallow crater with surrounding erythema and warmth. What stage pressure ulcer does the nurse chart?
    a. Stage I
    b. Stage II
    c. Stage III
    d. Stage IV
    B
  36. The nurse knows that several age-related changes in the integumentary system increase older adults’ risk for pressure ulcers. Which factors does this include? (Select all that apply.)
    a. Poor nutrition
    b. Living in a nursing home
    c. Thinning epidermis
    d. Decreased skin elasticity
    e. Vessel degeneration
    C, D, E
  37. An older adult is experiencing age-related postural hypotension and he fears “something is really wrong” because he is the only one in his social group experiencing the problems. The nurse responds:
    a. “Don’t be concerned; just be very careful about your risk for falling.”
    b. “You have had very thorough testing, so don’t worry about it being serious.”
    c. “It’s just a matter of time before they too have to watch not to get up too quickly.”
    d. “You just don’t have the compensating mechanisms of your friends.”
    D
    The age-related symptoms of postural hypotension are dizziness or lightheadedness when changing positions rapidly. However, compensatory processes in the cortex and subcortical areas of the brain help aging individuals maintain relatively normal motor performance.

DIF: Understanding (Comprehension) REF: Page 565 OBJ: 27-2
TOP: Teaching-Learning

  1. What education by the nurse is most important to address age-related changes to the senses?
    a. Installing auditory smoke alarms
    b. Having regular eye checkups
    c. Being aware that hearing acuity decreases with age
    d. Checking the expiration dates on foods such as dairy
    A
    An age-related reduction in the senses makes it less likely that an older person will smell smoke from a fire. Loud fire alarms are important for home safety. The other factors are not as directly related to safety.

DIF: Understanding (Comprehension) REF: Page 566 OBJ: 27-2
TOP: Teaching-Learning

  1. The nurse is conducting an admission assessment on a mildly confused older patient. The nurse best assures an accurate history by first:
    a. scoring the client’s cognitive responses.
    b. focusing on the client to respond.
    c. directing the questions to both patient and family.
    d. arranging a Mini-Mental State Examination (MMSE).
    C
    An interview with the friend or family member is an appropriate method to first implement when a patient is exhibiting confused behavior. The other options will not get accurate information for the assessment.

DIF: Understanding (Comprehension) REF: Page 566 OBJ: 27-2
TOP: Nursing Process: Assessment

  1. A nurse is caring for an older patient diagnosed with acute depression. What action by the nurse is most important to help prevent delirium in this patient?
    a. Reorienting the patient to the day, time and place frequently
    b. Being physically present to help the patient with eating meals
    c. Providing the patient with opportunities to discuss depression
    d. Administering antidepressive medication as prescribed
    B
    Depressed older adults may neglect eating or caring for a chronic medical condition, predisposing them to the development of delirium resulting from hypoalbuminemia and possibly electrolyte imbalances. The other actions will not prevent delirium.

DIF: Applying (Application) REF: N/A OBJ: 27-4
TOP: Nursing Process: Implementation

  1. When assessing an older patient displaying symptoms of delirium, the nurse focuses the assessment on:
    a. the degree and duration of the symptoms.
    b. the amount of self-care deficiency the symptoms cause.
    c. identifying processes that commonly result in the symptoms.
    d. physiologic dysfunction resulting from the symptoms.
    C
    The treatment of delirium entails the identification and treatment of the underlying cause. The nurse should assess this factor as the priority. The other assessments are of lesser priority.

DIF: Applying (Application) REF: N/A OBJ: 27-4
TOP: Nursing Process: Assessment

  1. An 80-year-old patient is exhibiting signs of dementia representative of Alzheimer disease (AD). The nurse supports that possibility when determining that the patient:
    a. experienced a gastric resection several years ago.
    b. traveled often to third world countries.
    c. was employed as a steelworker for 40 years.
    d. has a history of viral encephalitis.
    D
    Viral illness such as herpes zoster, herpes simplex, or viral encephalitis is believed to be a possible risk factor for AD. However, advancing age is the primary risk factor. The other options are not related.

DIF: Remembering (Knowledge) REF: Page 571 OBJ: 27-5
TOP: Nursing Process: Assessment

  1. When planning care for the older adult with advanced dementia, the nurse recognizes that the best way to implement reality orientation is to:
    a. place printed labels on important items, such as the telephone.
    b. place a clock and calendar in the patient’s immediate environment.
    c. use hand gestures instead of verbal communications to demonstrate meaning.
    d. show the patient a picture of a toothbrush when it is time for oral hygiene.
    D
    Reality orientation supports failing memory in early stages of dementia and preserves independent functioning for a longer duration. Although written messages and signs may become meaningless to individuals with advancing dementia, pictures often evoke a response. The other options are not part of this strategy.

DIF: Remembering (Knowledge) REF: Page 579 OBJ: 27-4
TOP: Nursing Process: Implementation

  1. A 73-year-old patient diagnosed with vascular dementia is admitted for exacerbation of asthma. The patient has been treated for 2 years with benzodiazepines to manage her increasingly aggressive behavior. The nurse’s initial response is to:
    a. identify the patient as being at high risk for falls.
    b. monitor the patient for signs of benzodiazepine withdrawal.
    c. notify the admitting physician immediately.
    d. place the patient on strict intake and output.
    C
    Benzodiazepines should be reserved for acute situations and not used for the long-term management of troubling behaviors. Long-term use can precipitate withdrawal if use is stopped and can possibly cause seizures. The nurse should notify the physician immediately so that plans for safely discontinuing the drug can be made.

DIF: Applying (Application) REF: N/A OBJ: 27-6
TOP: Communication and Documentation

  1. Which of the following statements, when made by family members caring for an older patient with dementia, indicates peaceful acceptance of the situation?
    a. “I’m so pleased that Mother had a good day today. I’m really very hopeful.”
    b. “The hospice nurses are so helpful when I need time for myself.”
    c. “I promised Mother I would take care of her and I’ll never leave her.”
    d. “It’s the least I can do for Mother since she cared for us all these years.”
    B
    Adjusting to the fact that dementia is irreversible and prolonged places families in situations of dealing with grief over a long period. Nurses need to encourage caregivers to take time out from their task and participate in self-care and health promotion activities. The other statements do not show this acceptance as clearly.

DIF: Evaluating (Evaluation) REF: N/A OBJ: 27-6
TOP: Nursing Process: Evaluation

  1. The son of a patient with possible Alzheimer disease (AD) asks the nurse if there is a diagnostic test that can confirm the diagnosis. The nurse responds that:
    a. an electroencephalogram is often very useful in diagnosing AD.
    b. a positron emission tomography (PET) scan is a cheap but dependable tool.
    c. magnetic resonance imaging (MRI) is often ordered for that purpose.
    d. postmortem autopsy is the only definitive diagnostic tool.
    D
    Autopsy remains the gold standard and only definitive method for the diagnosis of AD.

DIF: Understanding (Comprehension) REF: Page 572 OBJ: 27-5
TOP: Teaching-Learning

  1. An 89-year-old diagnosed with dementia was until recently responding well to cognitive cueing techniques. The nurse shows an understanding of dementia when sharing with staff that:
    a. “We will implement new interventions that address the disease’s progression.”
    b. “It’s important that we frequently recue the patient to improve her quality of life.”
    c. “The patient’s family needs to be made aware of this decline.”
    d. “This poor response to cueing is likely a result of advanced aging.”
    A
    Positive responses to selected interventions may continue for a time but may decline as the disease progresses, which results in the need to reevaluate strategies. The nursing staff cannot evaluate the patient’s quality of life; only the patient can, and this patient is not capable. The family should be informed but that is not related to understanding dementia. The change in response is the result of advancing disease, not age.

DIF: Applying (Application) REF: N/A OBJ: 27-6
TOP: Teaching-Learning

  1. An 80-year-old patient who is experiencing symptoms of depression and anxiety is reluctant to comply with the prescribed treatment plan. The nurse initially addresses the issue with the patient by asking:
    a. “How do you feel about how others view your mental health problem?”
    b. “Are you concerned about paying for your psychiatric medications?”
    c. “Did you know that depression is common among people your age?”
    d. “Do you have any questions about your the mental health treatment plan?”
    A
    Older adults are often reluctant to seek care from a mental health professional because they grew up during a period when a strong stigma was attached to mental illness, mental hospitals, and mental treatment. The other questions do not open a discussion.

DIF: Applying (Application) REF: N/A OBJ: 27-8
TOP: Nursing Process: Assessment

  1. An older adult patient being treated for chronic obstructive pulmonary disease (COPD) is exhibiting signs of memory loss and confusion. In planning his care, the nurse should give priority to:
    a. obtaining an order for a pulmonary function test (PFT).
    b. determining the potential of a possible adverse drug reaction.
    c. reorienting the patient to time, place, and person frequently.
    d. assessing for a family history of dementia.
    B
    Two of the most common side effects of many medications taken by older adults are mental confusion and disorientation. The initial action should be to determine the possible cause of the symptoms. If a cause can be found, a change might be possible. There is no indication the patient needs a PFT. Reorienting the patient is a good intervention, but it would be better to identify and eliminate the causative factor. Assessing a family history is a potential intervention as well.

DIF: Applying (Application) REF: N/A OBJ: 27-3
TOP: Nursing Process: Assessment

  1. The nurse caring for an older adult patient recovering from cardiac surgery recognizes that it is most appropriate to assess this patient for mental health problems because:
    a. cardiac surgery often results in anxiety-related issues.
    b. untreated depression can contribute to the patient’s morbidity risks.
    c. many in this age cohort have undiagnosed depression.
    d. hospitalization is both anxiety and depression inducing.
    B
    Depression can and should be treated when it occurs with other illnesses because untreated depression can delay recovery from or worsen the outcome of the other illnesses. Cardiac illness is associated with depression, but not necessarily with anxiety issues. It is true that depression in the older population is underdiagnosed. Hospitalization can lead to depression. But the main reason to assess for depression is because of its effects on other health conditions.

DIF: Applying (Application) REF: N/A OBJ: 27-8
TOP: Nursing Process: Assessment

  1. When planning care for the older adult being treated for depression, the nurse addresses the patient’s tertiary intervention needs best by:
    a. helping the patient to identify the early symptoms of depression.
    b. helping the patient deal with the physical symptoms of depression.
    c. discussing with the patient how to implement new coping skills.
    d. educating the patient about the importance of being drug compliant.
    C
    Tertiary intervention refers to the restorative or rehabilitative functions that the nurse performs to assist patients in the recovery process. An important aspect of tertiary intervention involving patients with depression is teaching new coping skills to lessen the likelihood of recurring depression. Identifying early symptoms is a secondary prevention. Treating the depression will limit the physical symptoms. Education is generally considered primary prevention. However, in this case it is education on part of treatment. This is not the best answer because the nurse is not teaching about the drugs, only about the importance of being compliant.

DIF: Applying (Application) REF: N/A OBJ: 27-8
TOP: Nursing Process: Implementation

  1. To best advocate for an older adult patient being prescribed medication to control newly observed signs of confusion and aggressive behavior, the nurse:
    a. initiates an assessment to determine possible underlying causes of the behavior.
    b. contacts family to inform them of the new medication therapy being planned.
    c. discusses possible nonpharmaceutical treatments with the physician.
    d. documents a detailed description of the behaviors before administering the drugs.
    A
    In this population, such symptoms may be mistakenly assumed to be a result of normal aging, so prescription medications may be ordered for anxiety, depression, aggressive and disruptive behavior, or paranoid-type behavior, without assessing the reasons for the behavior. If an underlying cause of the behavior is found, it can be treated, thereby eliminating the problem. The other actions do not demonstrate advocacy.

DIF: Applying (Application) REF: N/A OBJ: 27-9
TOP: Nursing Process: Assessment

  1. An older adult has been recently diagnosed with type 2 diabetes and mild retinal deterioration. To best address the patient’s potential for developing situation depression, the nurse:
    a. assesses the patient’s coping skills.
    b. Encourages the patient to participate in a depression support group.
    c. assesses the patient’s ability to manage the symptoms.
    d. educates the family on early signs of depression.
    A
    One of the keys to successful aging is adjusting to or, perhaps more accurately, adapting to, although not necessarily accepting, changes that occur in one’s life. The nurse assesses the patient’s coping skills and methods. The patient does not need a support group before developing depression. Managing symptoms is part of coping. Educating the family is an appropriate intervention but is not the priority.
    DIF: Applying (Application) REF: N/A OBJ: 27-8
    TOP: Nursing Process: Assessment
  2. An older adult has a medical condition that has required hospitalization at a facility far from home and family. To best minimize the patient’s risk for depression, the nurse:
    a. keeps the patient informed of the expected discharge date.
    b. offers to help the patient telephone family members each evening.
    c. reassures the patient that early discharge is a nursing goal.
    d. encourages the patient to place family photographs around the room.
    B
    The family continues to be the first source of support for older adults. This support is best achieved by regular contact through visiting or telephoning. The other options may be helpful, but they are not the best choice.
    DIF: Understanding (Comprehension) REF: Page 599 OBJ: 27-8
    TOP: Caring MSC:
  3. An older patient is anxious about an upcoming diagnostic test and requests something to calm the nerves. To best address the patient’s need, the nurse prepares to administer a PRN dose of:
    a. clonazepam (Klonopin).
    b. diazepam (Valium).
    c. chlordiazepoxide (Librium).
    d. lorazepam (Ativan).
    D
    There are two broad categories of benzodiazepines: short-acting (e.g., alprazolam [Xanax], lorazepam [Ativan], and oxazepam [Serax]) and long-acting (e.g., diazepam [Valium], chlordiazepoxide [Librium], and clonazepam [Klonopin]). The short-acting agents are preferred for older adults because of their lower potential for buildup leading to sedation and depression.

DIF: Applying (Application) REF: N/A OBJ: 27-9
TOP: Nursing Process: Implementation

  1. While collecting a health history for an older adult patient, the nurse learns that the patient had been prescribed Elavil 3 weeks ago and wants to stop taking it because “It didn’t make me feel any better.” In response to this information, the nurse shares with the patient that:
    a. sudden withdrawal is likely to cause a hypertensive crisis.
    b. depression seldom improves without medication.
    c. realistically it will take longer for the patient to feel an improvement.
    d. in time, people adjust to the side effects.
    C
    Older patients may need up to 12 weeks of this medication for evaluation of a full response. Psychotropic medications need to be started low and increases should be done slowly. The other options are not correct.

DIF: Understanding (Comprehension) REF: Page 597 OBJ: 27-9
TOP: Teaching-Learning

  1. The nurse familiar with the old adult population recognizes that the patient who has the greatest potential for successfully committing suicide is the:
    a. 63-year-old Asian female.
    b. 86-year-old Caucasian male.
    c. 76-year-old Hispanic female.
    d. 67-year-old African-American male.
    B
    The highest rates of suicide are among men over the age of 85.

DIF: Remembering (Knowledge) REF: Page 567 OBJ: 27-9
TOP: Nursing Process: Assessment

  1. A 65-year-old adult who recently lost his spouse is admitted to the hospital after a failed suicide attempt. He presents with a sad affect and is reluctant to interact within the milieu. The nursing diagnosis with priority is:
    a. ineffective coping related to recent loss.
    b. hopelessness related to death of spouse.
    c. risk for loneliness related to loss of spouse.
    d. risk for self-directed violence related to depression.
    D
    This patient is at risk for another attempt at suicide, so safety is the primary concern.

DIF: Applying (Application) REF: N/A OBJ: 27-8
TOP: Nursing Process: Analysis

  1. The nurse is caring for a severely depressed older patient. To best effect change in the patient’s emotional state, the nurse’s initial goal is to:
    a. plan interventions that will enhance the patient’s self-esteem.
    b. introduce the patient to new coping skills.
    c. assess the patient’s potential to self-harm.
    d. develop a therapeutic nurse-patient relationship.
    D
    The nurse’s ability to positively effect change in older adults’ responses to depression lies in the development of therapeutic relationships. Assessing risk for harm is an important safety issue but does not help the patient’s emotional state. The other two options come later after the relationship has entered its working phase.

DIF: Applying (Application) REF: N/A OBJ: 27-8
TOP: Caring

  1. An older adult is hospitalized for treatment of a mental health disorder and is prescribed clomipramine (Anafranil). The nurse documents that the medication is having the desired effect when the patient:
    a. begins sleeping 8 hours per night.
    b. engages in fewer ritualistic behaviors.
    c. reports fewer episodes of nervousness.
    d. exhibits no delusionary thinking.
    B
    This medication is a tricyclic antidepressant that is specifically helpful for obsessive-compulsive disorder (OCD). The other assessments are not specific indicators of the effectiveness of this medication.

DIF: Evaluating (Evaluation) REF: N/A OBJ: 27-8
TOP: Nursing Process: Evaluation

  1. To help manage the potential side effects of prescribed antipsychotic medications, amantadine (Symmetrel) may be prescribed. Which statement best indicates that the nurse understands the appropriateness of this medication for the older adult patient?
    a. “This medication produces few anticholinergic effects.”
    b. “Symmetrel is an effective dopamine agonist.”
    c. “Extrapyramidal symptoms are best controlled by Symmetrel.”
    d. “Older patients seem to have the fewest side effects on this medication.”
    A
    Amantadine (Symmetrel), a dopamine agonist prescribed to manage EPS, may be used, especially in older patients and in those with cardiovascular dysfunction, because of its reduced anticholinergic effects. The other statements are not accurate.

DIF: Remembering (Knowledge) REF: Page 598 OBJ: 27-8
TOP: Nursing Process: Assessment

  1. A 72-year-old is prescribed lithium. The nurse educates the patient on the importance of biannual evaluation of which of the following? (Select all that apply.)
    a. Renal function
    b. Serum glucose level
    c. Liver function
    d. Thyroid function
    e. Red blood cell count
    A, C, D
    Renal, liver, and thyroid studies should be evaluated every 6 months because of the drug’s potential toxicity. Glucose and red blood cell count are not affected.

DIF: Remembering (Knowledge) REF: Page 597 OBJ: 27-8
TOP: Teaching-Learning

  1. A 78-year-old patient was admitted with dehydration. The nurse assesses and documents observations that support a finding of dementia. Which of the following observations are related to dementia? (Select all that apply.)
    a. Forgetting what she ate for lunch today
    b. Crying frequently when alone
    c. Inability to find her way back to her room from the dayroom
    d. Being impatient with the nursing staff for not closing her door
    e. Repeatedly asking to call her son
    A, C, D, E
    Common manifestations of dementia include repeated questions and statements, forgetting to pay bills or take medications, increasing problems with orientation, and geographic disorientation. Other symptoms of AD include pervasive forgetfulness and memory loss, language deterioration, impaired ability to mentally manipulate visual information, poor judgment, confusion, restlessness, and mood swings. Personality changes may include apathy or loss of interest in previously enjoyed activities. Crying is not a classic sign of dementia, although depression often accompanies dementia and this could be a sign of depression.

DIF: Remembering (Knowledge) REF: Page 569 OBJ: 27-3
TOP: Nursing Process: Assessment

  1. The nurse is caring for an older adult patient admitted to the hospital. What assessment findings place the patient at risk for developing delirium during the hospitalization? (Select all that apply.)
    a. The patent takes medications to manage several chronic illnesses.
    b. The patent has a history of urinary tract infections.
    c. The patent is in cancer remission.
    d. The patent has recently been eating poorly.
    e. The patent experienced a mild heart attack 2 years ago.
    A, B, D
    The risk factors for delirium include advanced age, central nervous system diseases, infection, polypharmacy, hypoalbuminemia, electrolyte imbalances, trauma history, gastrointestinal or genitourinary disorders, cardiopulmonary disorders, and sensory changes. These factors can lead to physiologic imbalances increasing the risk for confusion. Cancer remission and a heart attack 2 years prior do not increase the patient’s risk.

DIF: Remembering (Knowledge) REF: Page 569 OBJ: 27-3
TOP: Nursing Process: Assessment

  1. A home care nurse is visiting a patient with moderate cognitive impairment from Alzheimer disease. The patient’s partner expresses concern about difficulty getting the patient “to eat properly.” The nurse suggests which of the following? (Select all that apply.)
    a. Serving meals at the same time each day
    b. Offering liquids in place of solid foods when possible
    c. Offering a calorie-dense snack at bedtime
    d. Cutting food into bite-sized pieces that will fit into the patient’s hand
    e. Asking the patent to identify favorite foods
    A, C, D
    It is important to support the ongoing nutrition of individuals with dementia because they may experience decreased hunger and ability to taste food. People who demonstrate symptoms of moderate to severe cognitive impairment may benefit from having meals in the same place at the same time each day. Small, frequent, nutritionally dense meals and snacks should be provided. During later stages of dementia, individuals may need to be reminded to open the mouth and chew. Food should be soft and cut in small pieces. Liquids do not need to be substituted for solid food. The patient may not be able to identify favorite foods, and asking may cause frustration.

DIF: Applying (Application) REF: N/A OBJ: 27-6
TOP: Teaching-Learning

  1. The nurse is preparing an older widowed patient with several chronic illnesses for discharge to home. The nurse addresses the primary nursing outcome for this patient when:
    a. assuring the patient that social services will arrange for help with medical expenses.
    b. arranging for in-home assistance in areas of activities of daily living (ADLs) and nursing care as needed.
    c. educating the patient regarding the safety risks caused by these conditions.
    d. identifying barriers to ensure adherence to the prescribed drug therapies.
    D
    A key role for the nurse caring for an older adult with a chronic condition is to help the patient achieve optimal physical and psychosocial health. Staying adherent with drug therapy can help achieve this outcome. Payment through a third party is not guaranteed. In-home assistance may or may not be needed. Education is always needed but is not the priority for achieving optimal wellness.
  2. The nurse is assessing patients diagnosed with chronic disease processes for the probability of hospitalization because of the exacerbation of related symptoms. The nurse recognizes that the patient with the highest probability is a(n):
    a. 72-year-old male with congestive heart failure (CHF).
    b. 82-year-old male with type 2 diabetes.
    c. 72-year-old female with chronic bronchitis.
    d. 82-year-old female with osteoporosis.
    A
    Individuals with chronic conditions typically have repeated hospitalizations to treat exacerbations of their illness. The most common reasons for hospitalization in older patients are heart disease, cancer, pneumonia, and stroke. The 72-year-old with CHF is at highest risk.
  3. An older patient has developed moderate muscle weakness on the left side as a result of a cerebral vascular accident (CVA, stroke). The nurse determines the patient possesses the healthiest view of self-wellness when heard stating:
    a. “I’ll certainly miss hiking, but I guess I’ll find something else to do outdoors.”
    b. “I was getting too old to safely practice karate.”
    c. “I’ve decided to take up oil painting because it’s difficult for me to knit.”
    d. “It was getting difficult to work in the garden anyway.”
    C
    After learning and mastering the requirements imposed by the condition, older adults often view themselves as “well.” With a wellness-in the-foreground perspective, the disease is only one component of their life and is not their identity, so they substitute lost abilities and resulting pleasures with others.
  4. A 73-year-old patient has been diagnosed with congested heart failure (CHF). The nurse provides the greatest support for this patient’s positive view of self-wellness by presenting information regarding:
    a. how to minimize the exacerbation of symptoms.
    b. locally available supportive services.
    c. the importance of adherence to medical treatment.
    d. the need to report symptoms promptly.
    A
    Many older adults now seek education about health promotion and management of their illness. The nurse can support older adults by teaching self-care management in these areas. The other actions are also valued, but learning how to control symptoms gives patients the feeling of accomplishment.
  5. The nurse is caring for an older patient who recently immigrated to the United States from Asia. To best address the patient’s apparent resistance to the medical and nursing plan of care, the nurse:
    a. discusses the patient’s behavior with Asian staff members.
    b. researches the patient’s cultural views on health care.
    c. requests a cultural consultation from social services.
    d. asks family members to discuss the patient’s views on health care.
    D
    Concepts of health and illness are deeply rooted in culture, race, and ethnicity and influence an individual’s (and family’s) illness perceptions and health and illness behavior. The patient’s family should have the best insight into the patient’s culturally biased beliefs. Discussing behavior with other staff members might be a privacy violation. Researching culture may be helpful, but each patient is an individual and should not be stereotyped. Social services may or may not be able to provide cultural services.
  6. The nurse feels most confident that an older patient is prepared to assume self-management of new type 2 insulin-dependent diabetes when the patient:
    a. is heard asking her son to check the insulin’s expiration date.
    b. is able to identify the symptoms of hypoglycemia.
    c. asks why she needs to test her glucose levels so frequently.
    d. inquires why she needs to have an A1C test every 3 months.
    A
    Adherence is greatly improved when the patient is in agreement with the treatment plan and shows a willingness to follow it. The patient is requesting help ensuring that the insulin is not expired. Knowledge about symptoms does not equate with adherence. Asking questions does not indicate adherence or not.
  7. The nurse impacts the trajectory of a patient’s type 2 insulin-dependent diabetes best by:
    a. evaluating the patient’s ability to administer insulin appropriately.
    b. providing the patient with a written copy of the treatment plan.
    c. explaining to the patient the importance of serum glucose control.
    d. providing the patient with research-based nursing care.
    A
    The illness trajectory can be modified by actions taken by the health care provider that directly affect the patient’s ability

interest to adhere to the treatment plan prescribed. The other actions are important but do not directly affect the illness trajectory.

  1. An older patient diagnosed with severe osteoarthritis has recently moved in with his son ecause of a history of falls. The son describes how he and his family have eagerly assumed responsibility for “meeting all Dad’s needs.” The nurse is most concerned that this environment will result in the patient:
    a. developing a sense of powerlessness and possibly a loss of hope.
    b. becoming unnecessarily physically and emotionally dependent.
    c. losing his will to “get better” and become independent again.
    d. becoming resentful and argumentative with his son’s family.
    A
    With no control over the meeting of his own needs, the patient may develop a sense of powerlessness, which can result in a loss of hope. The other concerns might be a problem for some patients, but powerlessness and loss of hope remain the priority.
  2. To best assist an older adult patient to cope with a new diagnosis of chronic renal failure, the nurse:
    a. asks the patient to describe her usual coping strategies.
    b. provides the patient with descriptions of new coping strategies.
    c. initiates discussions with the patient to explain the disease.
    d. offers to arrange a meeting with another patient with the diagnosis.
    C
    Understanding the illness and what to expect is directly related to the ability to cope. After the patient has information, the nurse can then assess psychosocial systems.
  3. To best help manage health care costs in older adults, the nurse entrepreneur would do which of the following?
    a. Create a telehealth system where nurses could check on patients daily.
    b. Provide local transportation services for older people to keep appointments.
    c. Create educational videos in multiple languages seen in the community.
    d. Build a nurse-run clinic to serve the homeless and underinsured population.
    A
    The majority of health care expenditure is spent on chronic illness. Patients with chronic illnesses have multiple hospitalizations for exacerbations of their conditions. Keeping chronic conditions under control would make a difference in health care cost. A telehealth service in which nurses could assess and counsel patients daily could help accomplish that goal. The other ideas are good too, but tight control of chronic conditions is a priority.
  4. A nurse is working with a patient who was diagnosed with type 2 diabetes 4 months ago. The patient’s blood sugars have stayed under control. What action by the nurse is best?
    a. Ask the patient what barriers to wellness still exist.
    b. Remind the patient about the A1C in 2 months.
    c. Review side effects of medications with the patient.
    d. Ask the patient how she or he feels about diabetes.
    A
    Older patients typically see chronic illness as one part of their lives. The nurse can support older adults by working with them to identify areas that may hinder progress along the wellness continuum and by teaching self-care management in these areas. The nurse should assess the patient’s needs from his or her point of view.
  5. A nurse is assessing quality of life (QOL) in older individuals with chronic illnesses who attend a community center. What information is most important to assess?
    a. How many days were lost to exacerbations in the last year
    b. How good each individual perceives his or her QOL to be
    c. How burdensome the patient’s treatment regime is daily
    d. How often the patient needs to see a health care provider
    B
    QOL is individualized for each patient, and each person is the only one who can rate his or her quality of life. It is not dependent on objective measures such as number of health care visits or how many days were spent sick.
  6. Nurses should evaluate health programs based on what data?
    a. Effect on quality of life
    b. Cost-benefit ratio of service
    c. Adherence statistics
    d. Ease of following through
    A
    Quality of life should drive treatment decisions. The patient and health care provider should set goals that are mutually acceptable and promote independence and quality of life. The other factors are considerations, but quality of life is most important.
  7. A student learning about the early AIDS epidemic wonders why the patients were stigmatized. What response by the nurse was best?
    a. Fear of the unknown etiology
    b. Expense required government assistance
    c. Patients being ashamed of their illness
    d. Younger patients not having accomplishments
    A
    Stigma arises out of specific characteristics of a disease or an unknown etiology, which causes fear. In the early days of the AIDS epidemic, not much was known about transmission, which generated fear in health care workers and the general population.
  8. When working with older adults with chronic illness and exacerbations, what action by the nurse is most appropriate?
    a. Continually assess the patients for adherence to the regime.
    b. Assess the patients for ways they can remain in control.
    c. Teach the patients about the illness trajectory.
    d. Routinely review all medications the patients are taking.
    B
    With exacerbations, the patient loses some control over an acute phase of the illness. The patient can be helped to maintain independence, control, and dignity by reassessing what is still within the patient’s ability and desire to control. The patient may or may not be adherent, but the nurse should not assume he or she is not. Teaching about the illness trajectory is one tool for giving control to patients. Medication reviews should be done but are not the best action.
  9. An older patient has moved into an adult child’s home after an extended stay at a rehabilitation facility. The patient complains the child is now “the boss” and the child complains about caregiving duties. What action by the nurse is best?
    a. Help the older patient find another place to live.
    b. Suggest that it is time for assisted living.
    c. Mediate a family meeting to discuss roles.
    d. Listen empathetically but let them work it out.
    C
    Role reversals and role changes are common in families where an older adult has chronic illnesses. These lost roles need to be mourned by all involved. The nurse helps most in this situation by mediating a family meeting where roles, coping, and feelings can be discussed. The nurse can help problem solve by assisting the individuals to identify ways in which they can keep their traditional roles, if even only for a short time.
  10. An older adult has chronic fatigue from several illnesses. The patient is frustrated at this symptom. What action by the nurse is best?
    a. Ask the patient to prioritize activities.
    b. Have the patient keep a fatigue diary.
    c. Encourage the patient to rest in the day.
    d. Instruct the patient on good sleep hygiene.
    A
    Helping the patient cope with fatigue is an important nursing intervention. The nurse should first ask the patient to prioritize the activities she or he most wants to do. Then the nurse and patient can plan strategies that will allow the patient to participate in these activities. Keeping a diary is helpful, but knowing what interests the patient most is more important. Rest and sleep are important, but they are not the priorities.
  11. An older patient has been admitted to the nursing unit after a car crash and surgery. When does the nurse begin planning for rehabilitation?
    a. On admission
    b. When the patient is awake
    c. When the patient is stable
    d. When the family requests it
    A
    Planning for rehabilitation, like discharge planning, begins on admission.
  12. A home health care nurse is conducting a functional assessment on an older woman who lives alone. What assessment question is likely to get the best information?
    a. How do you manage all your medications?
    b. Who shops and cleans your house for you?
    c. Can you show me how you prepare a meal?
    d. What parts of your body can’t you wash?
    C
    Older patients may downplay or deny functional limitations, so the nurse gets more accurate data asking what the patient is able to do, rather than what she or he is not able to do.
  13. A nurse assesses a newly admitted patient to a nursing home using the Functional Independence Measure (FIM) and rates the patient at 20. What action by the nurse is best?
    a. Arrange admission to a rehabilitation center.
    b. Plan care for a nearly dependent person.
    c. Plan care for a nearly independent person.
    d. Tells the family the patient is cognitively impaired.
    B
    Eighteen measures are accounted for in the FIM with scores ranging from 1 (dependent) to 7 (independent). A score of 20 indicates near total dependence. The FIM does not measure cognitive status.
  14. The nurse is preparing to apply a topical cream on the arm of a cognitively impaired, anorexic older adult patient in the terminal stage of lung cancer. The nurse carefully monitors the effectiveness of the medication because its effectiveness will be most negatively impacted by the patient’s:
    a. age.
    b. cognitive limitations.
    c. nutritional status.
    d. cancer diagnosis.
    A
    Topical drugs face barriers to absorption because the aged skin has decreased water content, a relative decrease in lipid content, and a decrease in tissue perfusion. These changes may result in impaired absorption of some medications that are administered via lotions, creams, ointments, and patches. The other options are not related to medication effectiveness in this situation.
  15. When administering medications to older adults, the nurse shows an understanding of the effect of aging on drug distribution by monitoring the patient’s:
    a. cardiac function.
    b. liver function.
    c. red blood cell count.
    d. plasma albumin levels.
    D
    With age, particularly for malnourished or frail adults, plasma albumin levels may drop and therefore should be monitored. As a result of decreased sites for protein binding, the activity of highly protein bound drugs, and any side effects caused by these drugs may be increased. The other options may be appropriate for specific drugs, but not in general.
  16. An older adult patient has been prescribed warfarin (coumadin). The nurse’s primary intervention involves daily review of the patient’s:
    a. prothrombin time.
    b. body for bruising.
    c. serum creatinine level.
    d. reflex tone.
    A
    Warfarin therapy is monitored by the international normalized ratio (INR) or INR with prothrombin time.
  17. A patient with diabetes and hypothyroidism is being admitted to an assisted living facility. During the admission assessment, the patient reports difficulty falling asleep. The nurse shows an understanding of sleep dysfunction and the older patient when asking:
    a. “Have you ever been prescribed a sleeping medication?”
    b. “How do you feel about leaving your home to live here?”
    c. “How long have you been a diabetic?”
    d. “Are you taking medication for your thyroid problem?
    D
    Insomnia and anxiety are problems that commonly plague older adults. Because insomnia and anxiety often occur secondary to medication side effects or secondary to medical conditions such as dementia, thyroid abnormalities, or depression, proper diagnosis and treatment of any underlying causes of insomnia or anxiety can help this condition. The other questions are appropriate for an intake interview, but not specifically related to the insomnia.
  18. A patient is receiving propranolol (Inderal) for hypertension. Which outcome is the best indicator of goal success when considering the drug’s potential effect on the patient’s quality of life?
    a. The patient verbalizes the importance of moderate exercise.
    b. The patient experiences no injuries as a result of dizziness.
    c. The patient’s blood pressure stays within normal limits.
    d. The patient describes symptoms indicative of an adverse drug reaction.
    B
    The main concerns with the use of antihypertensive medications in older adults are an increased risk of orthostatic hypotension and dehydration. Exercising and maintaining the blood pressure within normal limits are treatment goals but do not impact quality of life like dizziness or fainting. Having an adverse drug reaction would not improve quality of life.
  19. The nurse responsible for administering medications to the residents of a long-term care facility shows an understanding of the risk of injury this population experiences when:
    a. confirming the patient’s identity prior to providing the medication.
    b. assessing the patient for a history of drug-related allergies.
    c. implementing the 5 rights of medication administration routinely.
    d. educating patients about the purpose and side efforts of their medications.
    C
    The Institute of Medicine (IOM) estimates that 1.5 million ADEs and 7000 deaths occur in the United States each year secondary to medication errors. Older adults are disproportionately affected; more than half of the medication errors occur in long-term care facilities and more than 500,000 occur among ambulatory Medicare patients. Some references use the 6 rights of medication administration.
  20. An older adult diabetic patient is mildly hypertensive. The nurse prepares to educate the patient regarding angiotensin II-blocking agents. These drugs are especially useful in older adults because they:
    a. protect the kidney’s function.
    b. have a well-defined therapeutic window.
    c. are more effective than other drugs in the same class.
    d. can be given when liver function is compromised.
    A
    The ACEIs and ARBs also have demonstrated value in decreasing the chance of cardiac mortality in patients with heart failure. They also confer renal protection, which is particularly beneficial for patients with diabetes. The other statements are not related to both the patient’s conditions.
  21. The nurse shows an understanding of medication-related risk factors common to older adults when asking:
    a. “Are you aware of the possible side effects of your medications?”
    b. “Do you regularly take any dietary supplements?”
    c. “How do you keep track of when your medications are due?”
    d. “How many different physicians are prescribing medications for you?”
    B
    About 52% of older adults living in the United States take some sort of dietary supplement on a regular basis in addition to prescription medications. This increases the potential for drug-drug interactions. The other questions are important assessment questions to include in a medication review.
  22. An older adult patient is having difficulty remembering when to take several of the prescribed medications. To improve the patient’s compliance with the medication regimen, the nurse:
    a. asks the patient’s spouse to consistently administer the drugs.
    b. checks the drug guide to see if decreasing the frequency if the drugs is possible.
    c. informs the patient’s physician about the drug noncompliance.
    d. teaches the patient to administer daily pills with a pill dispenser.
    D
    The regimen should be simplified as much as possible; using a drug dispenser could make the daily process less complicated. If the patient is still unable to manage this task, the nurse could consult with the provider about decreasing frequency or changing medications, or the nurse could ask the spouse to administer the medications if this were acceptable to the patient. But the easiest and most cost-effective action is to try a pill dispenser.
  23. The nurse is caring for an older adult who reports severe chronic pain. To best assess age-related physiologic changes that could influence plans for initiating an appropriate drug regimen, the nurse prepares the patient for which laboratory evaluation?
    a. White blood count
    b. Glomerular filtration rate
    c. Serum complement level
    d. Electroencephalogram
    B
    Many drugs are renally excreted, and there are age-related reductions in renal function. The nurse wanting to assess for such factors that influence the selection of drugs would most likely anticipate the patient having renal function studies done, including an evaluation of the patient’s glomerular filtration rate.
  24. An older adult patient is being assessed for possible alcohol abuse. To best assess the patient’s risk potential, the nurse asks:
    a. “Have you ever experienced a memory loss as a result of consuming alcohol?”
    b. “Would you drink to relax after a particularly stressful day?”
    c. “Do you ever drink when you are alone?”
    d. “How many alcoholic drinks do you consume each week?”
    D
    The nurse should start the assessment for alcohol abuse by inquiring as to the number of drinks the patient consumes each week. The other questions can be part of an abuse assessment, but it is easiest to start with a simple, quantitative question to open the discussion.
  25. An older adult patient is currently undergoing detoxification for alcohol at a rehabilitation center. When assessing the patient using the Clinical Institute Withdrawal Assessment tool, the nurse determines the patient’s current score to be 23. The nurse:
    a. immediately institutes seizure precautions.
    b. monitors the patient’s vital signs every 2 hours.
    c. arranges for the patient to be transferred to an acute care hospital.
    d. shares with the patient that the detoxification process is almost complete.
    C
    The maximum score on this tool is 67, and patients who score higher than 20 should be admitted to a hospital. The other options are incorrect.
  26. A 68-year-old man with a history of alcohol abuse is admitted to the acute care facility for reports of abdominal pain. Based on your understanding of alcohol withdrawal, the nurse knows that if patient is currently abusing alcohol, he will most likely:
    a. experience delirium tremors within 4 hours of hospitalization.
    b. develop withdrawal symptoms 48 to 72 hours after the last intake of alcohol.
    c. receive 1 ounce of alcohol every 4 hours while awake.
    d. be prescribed oxazepam (Serax).
    B
    Symptoms tend to peak 48 to 72 hours after a patient’s last drink, although they may occur within 4 to 12 hours. The patient may or may not have DTs. The patient should not receive alcohol and may or may not need medication.
  27. When working with a patient suspected of substance abuse, the nurse is particularly interested in determining the cause of a patient’s:
    a. acute abdominal pain.
    b. recurring insomnia.
    c. extensive history of falls.
    d. chlordiazepoxide (Librium) prescription.
    C
    Frequently, the symptoms of substance abuse are subtle or atypical, or they mimic symptoms of other age-related illnesses and remain undiagnosed. Patients’ presenting symptoms may be erratic changes in affect, mood, or behavior; malnutrition; bladder and bowel incontinence; gait disturbances; and recurring falls, burns, and head trauma. Acute abdominal pain, insomnia, and prescriptions for Librium may or may not be related to substance abuse, but falling is.
  28. A 67-year-old woman presents at the emergency department with symptoms that suggest possible abuse of a narcotic analgesic. To best assure the patient’s safe care, the nurse asks:
    a. “When did you first start using the analgesic?”
    b. “Have you experienced withdrawal symptoms before?”
    c. “Why did you initially need an analgesic?”
    d. “What prescribed drugs are you currently taking?”
    D
    First, if prescription drug abuse is suspected, the nurse should ask the patient or a family member to identify all medications that the patient is currently using. The nurse and physician can then plan for safe detoxification. In addition, the physician can try to prevent any untoward drug interactions resulting from prescribing a new medication that is contraindicated because of an existing prescription.
  29. Your patient reports frequent constipation as a result of prescription medications and asks the nurse for advice about using a daily over-the-counter laxative. The most appropriate response by the nurse is to:
    a. tell the patient to consult the health practitioner before using nonprescription drugs.
    b. educate the patient about the side effects of regular laxative use.
    c. tell the patient to avoid laxatives because they can interfere with medications already being taken.
    d. tell the patient to consult a dietician about ways to correct chronic constipation.
    A
    Education regarding the importance of contacting the health practitioner (physician or pharmacist) before taking nonprescription medication is essential for reducing the number of unintentional medication interactions. Educating the patient on side effects and teaching the patient nonpharmaceutical ways to manage constipation are also appropriate.
  30. When initially planning care for the older adult patient who is prescribed clonidine patches as part of a smoking cessation program, the nurse:
    a. assesses the patient for any skin disorders on the upper arms and back.
    b. determines how many cigarettes or cigars the patient smokes per day.
    c. asks if the patient is currently taking any antihypertensive medications.
    d. educates the patient to the possible side effects of clonidine therapy.
    C
    Clonidine is an antihypertensive, so knowledge of the patient’s medication history is vital to avoid inducing hypotension. The other assessments are not related to patient safety.
  31. The nurse explains to ancillary staff that caffeine abuse is difficult to diagnose in the older adult patient because caffeine intoxication symptoms:
    a. can be confused with normal effects of aging.
    b. often mimic those of some cardiac disorders.
    c. produce fewer symptoms in older adults than in younger adults.
    d. resemble the side effects of several antihypertensive drugs.
    B
    Caffeine stimulates the sympathetic nervous system, often producing the rapid pulse associated with cardiac disorders. Caffeine effects are not mistaken for normal signs of aging, produce fewer symptoms in older adults, or resemble side effects of antihypertensives.
  32. An older adult patient shares with the admitting nurse that she drinks “one shot” of whiskey nightly to help her sleep. The nurse documents the need to:
    a. assess the patient for slurred speech, lack of coordination, and nystagmus.
    b. address the effects of alcohol abuse with the patient.
    c. provide the patient with an alcohol substitute.
    d. assess the patient for signs of agitation, as well as anxiety and seizures.
    D
    It is important to assess older patients for the possibility of alcohol withdrawal if agitation, hallucinations, anxiety, or seizures develop. Because the patient admits to a shot a day, it is possible she drinks more or uses alcohol to self-medicate for problems other than insomnia. The nurse should monitor the patient for signs of withdrawal as a priority, because this is a medical emergency. Slurred speech, lack of coordination, and nystagmus are signs of overindulging. The nurse should not provide an alcohol substitute. It is appropriate to discuss the effects of alcohol, but safety comes first.
  33. The nurse is assessing patients for impending alcohol withdrawal. The nurse assesses the patient with which of the following conditions as a priority?
    a. Pulse, 58 beats

min; and BP 100

60
b. Pulse, 118 beats

min; and BP 160

90
c. Dozing off in chair and not recognizing staff
d. Reporting muscle aches and frequent stumbling
B
Manifestations of alcohol withdrawal are elevated blood pressure, elevated pulse, and autonomic hyperactivity. In addition, fever; increased hand tremors; insomnia; nausea and vomiting; transient visual, tactile, or auditory hallucinations or illusions; psychomotor agitation; anxiety; and grand mal seizures may occur. The nurse should see the hypertensive, tachycardic patient as the priority.

  1. The nurse is teaching cardiovascular risk factors to a group of older adults. The nurse stresses that cigarette smokers are four times more likely to die of sudden cardiac death than nonsmokers because smoking:
    a. interferes with the development of collateral coronary vessels.
    b. produces coronary artery stricture.
    c. results in carbon monoxide poisoning.
    d. increases platelet aggregation.
    D
    Smoking increases platelet aggregation and causes coronary artery spasms. Nicotine increases blood pressure and cardiac demands. Carbon monoxide in tobacco smoke decreases the oxygen-carrying capacity of the blood. Smoking does not interfere with collateral circulation or produce strictures, but it may contribute to higher levels of carbon monoxide in the blood.
  2. When assessing an older, female, African American adult, the nurse notes that she has been a type 2 insulin-dependent diabetic 10 years. The nurse notes that the patient’s greatest risk for developing secondary hypertension is her:
    a. gender.
    b. ethnic origin.
    c. vascular system status.
    d. insulin therapy.
    C
    Secondary hypertension identified in the vascular system refers to elevated blood pressure caused by underlying disease such as renal artery disease, renal parenchymal disorders, endocrine and metabolic disorders, central nervous system (CNS) disorders, coarctation of the aorta, and increased intravascular volume. Gender, ethnic origin, and insulin are not diseases that cause hypertension.
  3. When administering Lopressor to an older adult patient with hypertension, the nurse is careful to have the patient’s care plan include
    a. frequent assessment for dizziness or syncope.
    b. education of the signs and symptoms of thromboembolism.
    c. regular evaluation of the patient’s muscle strength.
    d. regularly scheduled serum potassium levels.
    A
    Dizziness is an adverse reaction to beta-blockers such as Lopressor.
  4. The nurse educates the obese older adult patient that the single most important outcome that will affect cardiac health is:
    a. compliance with drug therapy.
    b. adherence to the DASH diet.
    c. 20 minutes of exercise daily.
    d. a 10% reduction in weight.
    D
    A 10% reduction of total weight will decrease blood pressure in many overweight individuals. This factor has significance because it underscores the importance of weight reduction in the older adult population. The other factors are important but not as significant to overall cardiac health as is weight loss.
  5. To evaluate an older patient for possible renal failure as a result of chronic untreated hypertension, nurse prepares to:
    a. schedule an ultrasound.
    b. collect a urine sample.
    c. monitor intake and output.
    d. transport the patient to radiology.
    B
    A urinalysis will investigate for proteinuria or other signs of renal failure.
  6. The nurse should assess which patient first?
    a. The patient with acute shortness of breath
    b. The patient with epigastric pain
    c. The patient with right arm pain
    d. The patient with persistent indigestion
    ANS: A
    Older patients often have atypical signs and symptoms of cardiac ischemia, including shortness of breath, fatigue, syncope, confusion, and abdominal or back pain. Shortness of breath requires the most immediate assessment.
  7. It is suspected that an older adult patient is experiencing severe hypertension. The nurse documents symptoms that support this diagnosis when the patient reports:
    a. difficulty reading the newspaper’s print.
    b. being fatigued after walking around the block.
    c. noticing that his heart “skips a beat” frequently.
    d. getting up from a chair too quickly makes him dizzy.
    A
    The patient with severe hypertension may experience throbbing occipital headaches, confusion, visual loss, focal deficits, epistaxis, and coma. The other manifestations are not associated with hypertension.
  8. A novice nurse requires additional education on arterial vascular deficiency when suggesting the condition’s symptoms include:
    a. 2+ edema in calf and foot of left leg.
    b. a 2-cm ulcer between the great and second toe on the left foot.
    c. skin on the left leg is cool to the touch.
    d. toenails on the left foot are thick and brittle.
    A
    Edema is not generally observed in cases of arterial deficiency, but rather in venous insufficiency. The other options are manifestations of arterial vascular deficiency.
  9. The nurse shows an understanding of how anemia symptoms present in the older population when:
    a. questioning the patient about dizziness when turning from side to side in bed.
    b. assessing the patient for pale oral mucous membranes.
    c. asking whether the patient takes supplementary iron tablets.
    d. assessing the patient’s weekly intake of red meat.
    B
    Skin color is not a good indicator of pallor because of varying pigmentation. Oral mucous membranes, as well as conjunctivae and nail beds, are better indicators. The other options are not related to symptoms.
  10. A nurse has provided discharge teaching for an older adult patient who had a pacemaker implanted. Which statement by the patient indicates appropriate understanding of the device?
    a. “The battery will need charging every 2 years or so.”
    b. “I’m supposed to call my doctor if my pulse is within 10 beats of my preset rate.”
    c. “My wife will have to be the one who makes the microwave popcorn.”
    d. “I’ll take my pulse each morning before my first cup of coffee.”
    D
    The radial pulse should be taken at the same time daily and recorded. The patient should notify the provider if the pulse is lower than the preset lower limit on the pacemaker. Battery life is longer than 2 years. Microwaves are safe to use.
  11. A 76-year-old patient has been recently diagnosed with cardiac valvular disease. The nurse assesses the patient and recognizes that the medical diagnosis is supported by:
    a. cyanotic fingertips.
    b. weight loss of 10 pounds in 3 months.
    c. angina pain.
    d. shortness of breath with activity.
    D
    Individuals with valvular disease may be asymptomatic for many years, but with the deterioration of the valves and hypertrophic changes in the atria or ventricles, symptoms become evident. Exertional dyspnea is frequently the initial symptom. Other symptoms include dizziness, fatigue, weakness, and palpitations. The other signs are not manifestations of valve disease.
  12. An older patient is upset with a blood pressure reading of 180

78 mmHg. What response by the nurse is best?
a. “It looks like you need blood pressure medicine now.”
b. “Most people get hypertension when they get older.”
c. “With age, blood vessels stiffen, raising blood pressure.”
d. “Don’t worry, there are lots of good medications for this.”
C
With age, elastin in vessel walls decreases, making them stiffer. Systolic blood pressure (SBP) is increased in older adults because of a loss of arterial distensibility resulting from arterial stiffening. The other responses do not offer any useful information on the cause of the patient’s condition.

  1. An older patient is overwhelmed at the number of lifestyle changes needed to manage newly diagnosed cardiovascular disease. What action by the nurse will reduce this barrier to teaching?
    a. Tell the patient even tiny changes over time make a big difference.
    b. Tell the patient that smoking is the biggest risk factor and needs to stop.
    c. Help the patient choose a change and incorporate it into daily life.
    d. Educate the patient on the consequences of not making changes.
    C
    Although it is true that small changes over time have a great impact, the nurse needs to do more by helping the patient choose a small change to implement. The nurse should help the patient work on the risk factor he or she is most willing to change. Education is important, but it will not enable the patient to make changes.
  2. A nurse is caring for a patient taking furosemide (Lasix). What assessment finding needs to be reported to the provider immediately?
    a. Weight gain of 1

2 pound (1.1 kg) in 24 hours
b. 2+

4+ pedal and pretibial edema
c. Potassium level: 2.6 mEq

L
d. Sodium level: 138 mEq

L
C
Furosemide is a potassium-wasting diuretic and the patient’s potassium is low. This finding should be reported. The weight gain should be charted but does not need immediate reporting. Without knowing what the patient’s baseline edema is, there is no indication this needs to be reported. The sodium level is normal

  1. An older patient is prescribed nifedipine (Procardia). What teaching topic is most important to discuss with this patient?
    a. Need to monitor blood pressure
    b. Need to follow low-salt diet
    c. Need to change positions slowly
    d. Need to add exercise to daily routine
    C
    Calcium channel blockers such as Procardia can cause orthostatic hypotension in older adults. The nurse educates the patient on preventing this by slow position changes. The other topics are appropriate for all patients on this medication.
  2. A patient had a heart attack and the nurse identifies the diagnosis as activity intolerance. What assessment finding indicates a priority goal for this diagnosis is being met?
    a. Mild chest pain getting into the chair
    b. Feels unsteady when getting out of bed
    c. O2 saturation 98% after using the commode
    d. Less dyspnea when changing positions
    C
    Activity intolerance is measured by changes in vital signs, electrocardiogram (ECG), and symptoms such as chest pain or shortness of breath. The oxygen saturation indicates physiologic tolerance to activity. The other options do not show physiologic tolerance.
  3. An older patient in the internal medicine clinic reports usually being able to walk 1 mile without complaint. However, in the past 2 weeks, after walking just mile, the patient’s legs begin to ache. The pain goes away with rest. What action by the nurse is most appropriate?
    a. Elevate the patient’s legs
    b. Assess the pedal pulses
    c. Take the patient’s blood pressure
    d. Measure the patient for TED hose
    B
    This patient has intermittent claudication, a sign of peripheral arterial disease. The nurse assesses the patient’s pedal pulses. Elevation will further compromise circulation and should be avoided. A blood pressure reading is taken during all health care visits. The patient does not need TED hose at this point.
  4. A patient has peripheral vascular disease. What statement by the patient indicates a need for further teaching?
    a. I will have the podiatrist cut my toenails.
    b. I will be sure to wear sturdy shoes.
    c. I can only walk limited distances now.
    d. I will report any injury to my foot or leg.
    C
    Patients with venous insufficiency are encouraged to begin a graduated exercise program. The other statements show good understanding.
  5. The nurse best maximizes an older adult’s potential to avoid developing a postsurgical respiratory infection by:
    a. walking the patient to the bathroom instead of using the bedside commode.
    b. encouraging compliance with prescribed antibiotic therapy.
    c. evaluating the patient’s ability to effectively cough and deep breathe.
    d. offering fluids every hour while the patient is awake.
    C
    Older adults have a decrease in the number and effectiveness of cilia in the tracheobronchial tree, which results in increasing difficulty clearing secretions. The other activities also help avoid atelectasis and infection, but evaluating the patient’s ability to cough and deep breathe can indicate that other treatment measures may be needed postoperatively.
  6. An older adult’s pulmonary function studies indicate that his vital capacity is reduced and his residual volume is increased. The nurse recognizes that these test results are observed in the patient’s:
    a. ineffective cough reflex.
    b. shallow breathing.
    c. slow respiratory rate.
    d. frequent respiratory infections.
    B
    Normal aging results in the progressive loss of elastic recoil of the lung parenchyma and conducting airways as well as reduced elastic recoil of the lung and the opposing forces of the chest wall. The lung becomes less elastic as collagenic substances surrounding the alveoli and alveolar ducts stiffen and form cross-linkages that interfere with the elastic properties of the lungs. Any and all of these structural changes make it more difficult for the older person to ventilate.
  7. The nurse is concerned about an older adult patient developing toxic levels of the prescribed theophylline when it is determined that the patient has a(n):
    a. one pack a day smoking habit.
    b. elevated serum potassium level.
    c. history of chronic bronchitis.
    d. chronic, nonproductive cough.
    A
    Theophylline is a medication that is affected by smoking, which increases serum drug levels. The other factors do not affect the pharmacokinetics of this drug.
  8. The nurse is aware of the typical occurrence of comorbidities in the older adult. Motivated by this knowledge, the nurse assesses a patient with diagnosed respiratory dysfunction for possible:
    a. poor wound healing of the legs and feet.
    b. ineffective absorption of vitamins and minerals.
    c. abnormal urine protein levels.
    d. visual problems including retinal detachment.
    A
    In addition, older patients are more likely to have comorbidities involving the cardiovascular and respiratory systems. Peripheral circulation is a possible cardiovascular problem that would result in poor wound healing. The other options are not related to having a respiratory dysfunction.
  9. Because the older adult is not as likely to exhibit the typical signs of ineffective gas exchange, the nurse is particularly suspicious of:
    a. a nonproductive cough in an afebrile patient.
    b. irritability in a usually pleasant patient.
    c. pale nail beds in a patient of color.
    d. an elevated white blood cell (WBC) count in an 82-year-old patient.
    B
    An early sign of respiratory problems is a change in mental status. Because the physiologic responses to hypoxemia and hypercapnia are blunted in older patients, compensatory changes in heart rate, respiratory rate, and blood pressure may be delayed and cerebral perfusion may suffer. Mental status changes may include subtle increases in forgetfulness and irritability.
  10. The nurse is preparing information for the caregivers of a patient with chronic respiratory issues. The nurse will make the greatest impact on their ability to provide quality care while maintaining the patient’s emotional well-being by including:
    a. suggestions regarding proper nutrition and exercise for the caregiver.
    b. an explanation on how to preserve the patient’s sense of autonomy.
    c. encouragement for the primary caregiver to set aside time for his or her own interests.
    d. recommendations of periodic self-reflection regarding the stressors the patient experiences.
    B
    Many patients with respiratory illness feel a loss of control over their lives because of their symptoms. They may become demanding and controlling in dealing with their families and friends. Well-being is enhanced by having some control over one’s life. The other options relate to the caregivers.
  11. An older patient admitted to the hospital with symptoms strongly suggestive of tuberculosis (TB) has a negative Mantoux test. The nurse correctly anticipates that:
    a. the purified protein derivative (PPD) test will be administered.
    b. a chest x-ray will be ordered to detect possible infiltration.
    c. therapy consisting of a combination of bactericidal drugs will be initiated.
    d. the skin test will be repeated to achieve a booster effect.
    D
    Tuberculin skin testing in older patients is an unreliable indicator of TB because they are more likely to have false-negative results because of reduced immune system activity. If skin testing is used, it is recommended that the standard 5 tuberculin unit (TU) Mantoux test be given and then repeated to create a booster effect. The PPD is not recommended. The skin test is followed up with a chest x-ray. Drug therapy should not begin until the patient has a diagnosis.
  12. An older adult patient who has tuberculosis is being treated with the drugs isoniazid 300 mg daily, rifampin 600 mg daily, and pyrazinamide 1500 mg daily. The nurse stresses the importance that the patient:
    a. wear tinted glasses when out in the sun.
    b. minimize contact with children younger than 3 years old.
    c. avoid alcohol while on the drug therapy.
    d. eat and drink dairy sparingly.
    C
    Patients should not drink alcohol while taking isoniazid. The other recommendations are incorrect.
  13. An 80-year-old patient is concerned about contracting pneumonia. The nurse educates her that the key to prevention is:
    a. early recognition of the symptoms.
    b. being vaccinated per government guidelines
    c. minimizing contact with the public during the winter months.
    d. supplementing one’s daily diet with various vitamin C sources.
    B
    The key to pneumonia prevention is being appropriately vaccinated. All individuals should be vaccinated at age 65 unless they have conditions that lead them to earlier vaccination. Revaccination is indicated in certain circumstances. Signs and symptoms are subtle in the aging population. Minimizing contact during winter months is an appropriate suggestion, just not the best one. Vitamin C may have immune system benefits.
  14. The nurse gives priority to assessing an older patient who presents with symptoms of acute respiratory distress for which other condition?
    a. Substernal chest pain
    b. A history of panic attacks
    c. Any known allergies
    d. Bruising on the chest
    A
    The symptoms of asthma and respiratory distress mimic other conditions such as myocardial ischemia. The nurse assesses for this condition as the priority over the others.
  15. The nurse is caring for an older adult who has been prescribed inhaled corticosteroids for asthma. What does the nurse teach about this medication?
    a. Taken just before retiring for the night
    b. Reserved for acute attacks only
    c. Used in increasing doses as needed
    d. How to use and rinse the inhaler
    C
    Corticosteroids are an effective long-term control medication that can be used in increasing doses as needed for asthma and related disorders. It is given by the inhalation method, so the nurse teaches the patient how to use and maintain the inhaler. The other teaching tips are incorrect.
  16. An older patient with severe peripheral arterial disease wishes to quit smoking. The nurse provide education to this patient on which of the following?
    a. “Cold turkey” method
    b. Gradual reduction
    c. Nicotine patches
    d. Bupropion hydrochloride (Zyban)
    D
    Older patients should be offered assistance to quit smoking. The cold turkey and gradual reduction methods may not work if the patient is a long-term smoker. The patient with peripheral arterial disease should not use nicotine in any form as it causes vasoconstriction. Zyban is an appropriate choice.
  17. An older adult with chronic obstructive pulmonary disease (COPD) asks why he should quit smoking now. What response by the nurse is best?
    a. “It will keep your disease from getting worse.”
    b. “There are many benefits to quitting even now.”
    c. “It will decrease the risk of getting cancer too.”
    d. “You’re right; there really isn’t a reason to quit.”
    B
    There are many benefits to smoking cessation including reduction in the number of respiratory infections, improvement in the function of the mucociliary clearance of the lungs, decreased coughing and dyspnea, increased appetite, and decreased sputum production. This is a more comprehensive answer than keeping the disease from worsening and lowering the chance of getting cancer.
  18. A patient has been taught about nutrition related to COPD. Which menu selection may indicate a need for further teaching?
    a. Bagel and cream cheese
    b. Broiled chicken breast
    c. Beans and peas
    d. Tofu stir-fry
    A
    Carbohydrates should not make up more than 50% of the daily intake of calories because they break down into carbon dioxide, worsening breathing. The other selections show good understanding. Of course, the nurse needs to take into consideration the amount of carbohydrates in the entire day and not just one selection.
  19. An older patient is hospitalized with influenza and is prescribed amantadine (Symmetrel). What assessment finding would indicate this drug is not appropriate for the patient?
    a. BUN 22 mg

dL
b. Creatinine 3.2 mg

dL
c. Sodium 132 mEq

L
d. Potassium 4.2 mEq

L
B
Amantadine can cause behavioral changes, delirium, hallucinations, agitation, and seizures, mostly in patients with impaired renal function. The creatinine is high, indicating renal disease. The other lab values are normal.

  1. A frail, older patient is in the emergency room in severe respiratory distress. The patient has had repeated hospitalizations for the same thing. After stabilizing the patient, which action by the nurse is most appropriate?
    a. Determine what the patient’s end-of-life wishes are.
    b. Assess the family caregiver for compliance with treatment.
    c. Administer intravenous (IV) fluids at a rapid rate.
    d. Prepare to vaccinate the patient against pneumonia.
    A
    Because of the lifesaving modalities needed to care for such a patient, the nurse and physician work together to determine what the patient’s end-of-life wishes are. In the emergency department, patient stabilization comes first, but once this has been accomplished a discussion should occur with the patient and family about further treatment desires. The family caregiver may or may not be adherent, or the patient may assume all self-care. IV fluids should not be given at a rapid rate because of the risk of heart failure. The patient should receive an immunization against pneumonia per guidelines.
  2. A patient has a pulmonary embolism and asks the nurse to explain the purpose of the heparin infusion. What response by the nurse is best?
    a. “It helps dissolve the clot in your lungs.”
    b. “It keeps you from getting septic.”
    c. “It prevents the clot from getting bigger.”
    d. “It prevents clots from forming in your heart.”
    C
    Heparin keeps the clot from getting bigger and hopefully prevents further clots from forming. It does not dissolve the clot. It does not specifically target the heart. It does not prevent sepsis.
  3. The nurse caring for patients using continuous positive airway pressure (CPAP) knows what about treatment effectiveness?
    a. Effectiveness depends on compliance.
    b. It’s too expensive for many older adults.
    c. It is rarely effective for sleep apnea.
    d. Complicated settings make it hard to use.
    A
    Effectiveness is determined by compliance for nearly any regime, and unfortunately compliance with CPAP is less than 50%. The other statements are incorrect.
  4. The nurse caring for an older adult with type 2 diabetes mellitus places importance on assessing the patient for:
    a. painful nodules on the fingers and toes.
    b. reddened rash and brittle nails on the hands.
    c. heartburn and flatus after meals.
    d. skin temperature and hair growth pattern on the legs.
    D
    Insulin resistance causes increased production of inflammatory cytokines correlating with the development of type 2 diabetes mellitus and atherosclerotic vascular disease, therefore skin temperature and hair growth pattern on the legs should be assessed.
  5. The nurse recognizes that an older adult on both antihypertensive and antidepressant drug therapies has a specific need for:
    a. regular blood pressure monitoring.
    b. an effective history focusing on sexual function.
    c. an increase in daily fluid intake.
    d. frequent assessment of emotional stability.
    B
    Drugs such as oral contraceptives, hormone replacement, antihypertensives, antidepressants, or sedatives can cause a sexual arousal disorder as a side effect. In women this can manifest as female sexual dysfunction (FSD), and in men it can manifest as erectile dysfunction (ED).
  6. Aware that older adult patients often present with nonclassic symptoms of type 2 diabetes mellitus, the nurse is particularly suspicious of a patient reporting:
    a. bouts of diarrhea alternating with periods of constipation.
    b. recent problems reading and an infected sore on the toe that will not heal.
    c. periods of depression and severe indigestion after eating.
    d. dizziness when getting up too quickly and a red rash on the hands.
    B
    Often a newly diagnosed older individual will describe symptoms of fatigue, blurred vision, weight change (gain or loss), and infections. The other symptoms are not related.
  7. The nurse observes signs that a patient being assessed may have an underactive thyroid. The data supporting this suspicion includes:
    a. heat intolerance, low-grade fever, and patchy hair loss.
    b. polycythemia, tachycardia, and oral candidiasis.
    c. muscle cramps, fatigue, and cold intolerance.
    d. increased blood pressure, postural hypotension, and blurred vision.
    C
    Older patients are seen with complaints of fatigue, cold intolerance, weight gain, muscle cramps, paresthesias, and confusion, which are symptoms of hypothyroidism that are often attributed to old age. Heat intolerance is often associated with hyperthyroidism. The other options are not related to thyroid dysfunction.
  8. The nurse is preparing to provide an older, newly diagnosed diabetic patient with information regarding type 2 diabetes. The nurse initially:
    a. asks if the patient prefers a video or a pamphlet.
    b. invites the patient’s spouse to be present during the instruction.
    c. selects a quiet, well-lighted space for the class.
    d. ensures that the patient is pain-free and comfortably seated.
    A
    Cognitive function and learning styles vary, so knowing the patient’s preferred learning style facilitates education. Some individuals prefer to learn by visual methods, others by listening, and still others by experiencing contact in a hands-on approach. Controlling pain and ensuring the patient is comfortable will also facilitate learning, but it is more important to meet the patient’s preferred learning style. The other options are helpful but not as vital.
  9. Which documentation demonstrates that the nurse effectively assessed an older adult diabetic patient’s cardiac status?
    a. radial pulse: 88 and regular
    b. carotid pulses equal and strong
    c. BP 126

78 recumbent and 122

78 sitting
d. nail beds pale in color
C
To assess circulation, the nurse should take an apical pulse, noting rate and rhythm; check pedal pulses bilaterally; and note the presence of hair on the lower extremities. The nurse should take blood pressure measurements with the patient in both recumbent and sitting positions, note any dizziness associated with a change of position, and assess the respiratory rate, depth, and chest sounds.

  1. The nurse teaching a 79-year-old with type 2 diabetes about the importance of regular exercise suggests that the patient:
    a. swim 10 laps in the community center pool three times a week.
    b. enroll in a daily lunch time aerobics class at the senior center.
    c. lift 5 pound weights in a routine of 10 repetitions in each arm.
    d. walk on the treadmill each morning for 30 minutes.
    D
    Older adults may derive the greatest benefit from morning exercise because that is the time of greatest insulin resistance. However, any exercise is better than no exercise at all. Aerobic exercise should be balanced with weight training.
  2. What assessment findings support an older patient’s diagnosis of hypothyroidism?
    a. A 2-cm wound noted on medial aspect of left foot
    b. An apical rate: 98

min
c. A patient report that “I always wear a sweater”
d. A weight loss of 10 pounds over 6 weeks
C
Older patients are seen with complaints of fatigue, cold intolerance, weight gain, muscle cramps, paresthesias, and confusion. The other assessments are not related.

  1. Which assessment findings support the suspicion that an older patient has osteoporosis?
    a. The patient’s reports an allergy to dairy products.
    b. A lactase enzyme is a part of the patient’s drug regime.
    c. Bones in one of the patient’s lower legs are shorter than in the other.
    d. The patient is inch shorter than at his or her previous physical.
    D
    Dorsal kyphosis, chronic back pain, and loss of height are common signs of primary osteoporosis in older persons. The other signs do not relate to this disorder.
  2. An older patient has been diagnosed with metabolic syndrome. What action by the nurse takes priority?
    a. Educate the patient on medications.
    b. Teach lifestyle changes the patient can manage.
    c. Encourage 60 minutes of aerobic activity daily.
    d. Instruct the patient on a low-fat diet.
    B
    Lifestyle changes are the mainstay of treatment for this disorder. Nurses have the primary responsibility for teaching. The patient should be included in planning so that lifestyle changes are reasonable and “doable” for the older patient. Activity and diet are part of the changes needed, but activity does not need to be so extensive and diet should not be the only topic taught. Medications are not generally used.
  3. A patient has type 2 diabetes. The family reports the patient has become very forgetful. What response by the nurse is best?
    a. “We should assess her for Alzheimer disease.”
    b. “Forgetfulness is a common sign in diabetes.”
    c. “Have her blood sugars been under good control?”
    d. “Does she recognize you and know your names?”
    B
    Many diabetics report depression and memory problems, so the nurse explains this fact. Forgetfulness does not necessarily indicate dementia. Asking about blood glucose is appropriate, but not related. Not recognizing family is not the same as forgetfulness.
  4. The family of a patient who has type 2 diabetes calls the clinic to report a very small sore on the patient’s foot. What action by the nurse is best?
    a. Have the patient come to the clinic today.
    b. Have the family wash and bandage it.
    c. Tell the patient to check for a fever.
    d. Have the patient go to the emergency room.
    A
    Any ulcer or sore on a foot requires medical attention because on superficial inspection, the true degree of injury can be hidden. The patient should come to the clinic today. The family should not attempt to care for the wound and the patient does not need to take a temperature before coming in. The patient does not need to go to the emergency department at this time.
  5. A patient has been admitted with new atrial fibrillation. What additional diagnostic testing does the nurse anticipate?
    a. Thyroid hormones
    b. Platelet count
    c. Urinalysis
    d. Blood glucose
    A
    Hyperthyroidism is often seen with atrial fibrillation. Platelet count, urinalysis, and glucose are often done as part of admission, but they are not directly related to atrial fibrillation as is hyperthyroidism.
  6. An older patient has osteoporosis and is reluctant to exercise because “I already have a bone problem, so how will it help?” What response by the nurse is best?
    a. It can improve posture, balance, and reduce falls.
    b. It will give you heart-healthy benefits.
    c. Exercise will make you feel younger.
    d. If you join a gym, you can socialize with new people.
    A
    Exercise not only improves bone health but improves posture, balance, and reduces falls. The nurse should educate the patient on these benefits.
  7. A nurse is preparing to administer metoprolol (Toprol) to an older male patient. What action by the nurse is best regarding endocrine disorders?
    a. Administer the medication as ordered.
    b. Check the patient’s ID using two sources.
    c. Say, “Many men experience ED with this drug.”
    d. Tell the patient to discuss the side effects with his provider.
    C
    The nurse should instruct patients on side effects of medications. The nurse can assess if the patient wants to discuss this issue by opening the conversation with a “normalizing” statement, such as “many men have ED on this drug. Is this something you would like to discuss?” Although side effects do need to be brought up to the provider, the wording of this statement does not indicate a willingness of the nurse to engage in conversation. The other two options are correct but not related to endocrine dysfunction.
  8. A type 2 diabetic patient is admitted to the hospital with a gastrointestinal illness and a blood glucose of 480 mg

dL. After stabilizing the patient, what action by the nurse is best?
a. Educate the patient on safe food handling.
b. Ask if the patient took the diabetic medication.
c. Teach the patient ways to avoid dehydration
d. Delegate frequent blood sugars to the aide.
B
A frequent cause of hyperglycemia requiring hospitalization in diabetics is poor sick day management. The type 2 diabetic still makes insulin and so needs antihyperglycemic drugs even when ill. The nurse assesses the diabetic’s knowledge of sick day management. The other options are appropriate but not the priority. Better sick day management can possibly keep the patient from further, similar, hospitalizations.

  1. A nurse is reviewing possible first-line medications for a new, older type 2 diabetic. What contraindication does the nurse identify for metformin (Glucophage)?
    a. Patient drinks three to four alcoholic drinks

day
b. Patient’s parents both took insulin
c. Creatinine 0.9 mg

dL
d. Potassium 3.8 mEq

dL
A
Patients with hepatic or renal dysfunction should not take metformin. A patient who drinks as much as three to four alcoholic drinks a day has a real risk of liver disease. The use of insulin by the parents is not related. Both kidney lab values are normal.

  1. The nurse is assessing an older patient with elevated plasma triglyceride levels. What other assessment finding leads the nurse to suspect metabolic syndrome? (Select all that apply.)
    a. Blood pressure of 148

90 mm Hg
b. A fasting blood glucose of 109 mg

dL
c. Reports of frequent urination
d. Weight measurement of 50 inches
e. HDL level of 52 mg

dL
A, B, D, E
The clinical criteria for metabolic syndrome includes increased waist circumference (population specific) plus any two of the following: (1) blood pressure greater than 129

84 mm Hg or taking hypertension medication, (2) plasma triglyceride levels over 149 mg

dL or taking triglyceride medication, (3) high-density lipid levels less than 40 mg

dL in men or less than 50 mg

dL in women or taking HDL-C medication, (4) fasting glucose greater than 99 mg

dL (including patients with diabetes).

  1. The nurse is teaching a newly diagnosed diabetic patient about metformin. What information does the nurse include? (Select all that apply.)
    a. Alcohol intake should be limited and taken with food.
    b. Overweight patients sometimes poorly tolerate metformin.
    c. Oral hypoglycemic agents can increase the risk of hyperglycemia.
    d. Metformin has been the cause of anorexia in older patients.
    e. Oral hypoglycemic agents affect vitamin D absorption.
    A, D
    Studies indicate that metformin, classified as a biguanide, may be the drug of choice for overweight patients. Side effects such as anorexia, nausea, and abdominal discomfort may, however, limit its use in older adults. Alcohol can decrease hypoglycemic awareness, so metformin should only be ingested with food. Alcohol use should also be limited.
  2. What assessment findings support a diagnosis of hyperthyroidism in the older adult? (Select all that apply.)
    a. Tremors
    b. Heat intolerance
    c. Tachycardia
    d. Palpable goiter
    e. Atrial fibrillation
    A, D, E
    The classic geriatric presentation of hyperthyroidism includes tachycardia, fatigue, tremors, and nervousness in contrast to tachycardia, heat intolerance, and fatigue in younger patients. An enlarged, palpable goiter is present in 60% of older adults with hyperthyroidism. The most common complication, occurring in 27% of geriatric hyperthyroid patients, is atrial fibrillation that does not convert back to sinus rhythm when a euthyroid state has been achieved.
  3. The nurse assessing patients for diabetes looks for the classic signs, including which of the following? (Select all that apply.)
    a. Polyuria
    b. Polycythemia
    c. Polydipsia
    d. Polyphagia
    e. Polyandrony
    A, C, D
    The classic signs of diabetes are polyuria, polydipsia, and polyphagia.
  4. When teaching an older patient about diet therapy, the nurse plans to assess for barriers to adherence, including which factors? (Select all that apply.)
    a. Lifelong habits
    b. Cultural influences
    c. Finances
    d. Dependency
    e. Inability to learn
    A, B, C, D
    Diet therapy can be problematic for older adults who have a lifetime of food habits, cultural influences on food, finances that may be limited, and dependency on others to buy or prepare food. Older adults are not unable to learn.
  5. The nurse teaches an older patient safety rules for exercising. What do these rules include? (Select all that apply.)
    a. Carry medical identification.
    b. Check blood glucose before exercising.
    c. Drink plenty of water.
    d. Have quick-acting glucose.
    e. Knowing signs of hyperglycemia.
    A, B, C, D
    Rules for safe exercise include all the above except the patient is more likely to experience hypoglycemia, so those signs and symptoms are important related to exercise.
  6. To minimize an older adult’s risk for developing postsurgical atelectasis, the nurse does which of the following? (Select all that apply.)
    a. Regularly assesses and medicates for pain
    b. Teaches effective deep-breathing techniques
    c. Provides oxygen via nasal cannula
    d. Encourages the patient to drink all fluids on meal trays
    e. Assesses lung sounds frequently
    A, B, D
    Promotion of deep breathing, effective pain management, adequate hydration, frequent position changes, and early mobility will decrease the risk of developing atelectasis. Providing oxygen and assessing lung sounds will not prevent atelectasis from occurring.
  7. When teaching older adult asthmatic patients, the nurse stresses the importance of which of the following? (Select all that apply.)
    a. Being alert for the early signs of breathing problems
    b. Fostering an effective relationship with your health care provider
    c. Identifying and avoid personal triggers
    d. Incorporating regular rest periods into your daily routine
    e. Increasing vitamin C consumption, especially during winter months
    A, B, C
    The prognosis for an older adult with asthma is relatively good. Success is based on a partnership between the patient and the health care provider to properly use prescribed medications, avoid asthma triggers, identify early signs of exacerbation, and maintain a healthy lifestyle. Rest may or may not be an issue if the patient has mild asthma. Vitamin C may have immune system benefits.
  8. The nurse is coordinating care for a newly admitted older adult. The patient is diagnosed with hypertension, asthma, atrial fibrillation, mild osteoarthritis, and glaucoma. Before administering the patient’s corticosteroid medication, the nurse is especially interested in which of the following? (Select all that apply.)
    a. The name of the patient’s hypertension medication
    b. What the patient uses to manage arthritic pain
    c. Whether the patient feels the asthma is well controlled
    d. Whether the patient takes low-dose aspirin regularly
    e. Whether the patient has ever had glaucoma-related surgery
    A, B, D
    Asthma may be exacerbated by the use of nonsteroidal antiinflammatory agents for arthritis, aspirin for circulation, nonselective beta-blockers for hypertension, or glaucoma eye drops that contain beta-blockers. Feeling that the asthma is under control and previous surgery are not directly related.
  9. The nurse is evaluating the effectiveness of an older patient’s self-management of asthma. What does the nurse assess as the priority? (Select all that apply.)
    a. How many times a week a rescue inhaler treatment is needed
    b. How well the patient is able to avoid the known triggers
    c. Whether the patient experience frequent respiratory infections
    d. Whether the patient requires rest periods during the day
    e. Whether the patient believes he or she has the support of family and friends
    A, B
    The evaluation of self-management is based on the patient’s success in following through with the plan. Determine the frequency of rescue inhaler use, success at avoiding triggers, and the patient’s ability to monitor and address lifestyle changes.
  10. The nurse encouraging an older patient to start pulmonary rehabilitation shares the benefits of the program, including which of the following? (Select all that apply.)
    a. Socialization
    b. Decreased cardiac risks
    c. Nutrition counseling
    d. Weight management
    e. Sports participation
    A, B, C, D
    There are many aspects to pulmonary rehabilitation, including socialization, decreased cardiac risks, nutrition counseling, and weight management Sports are not included, although exercise is.
  11. The effect of aging on the cardiovascular system is evidenced by which symptoms in an older adult performing a stress test? (Select all that apply.)
    a. Chest pain during exercise
    b. Slow increase of heart rate in response to stress
    c. Exercise induce dyspnea
    d. Slow decrease of heart rate post exercise
    e. Stress-induced arrhythmias
    B, D
    During stress or stimulation, the heart rate increases more slowly; however, once elevated, it takes longer to return to the resting rate. The other manifestations are not related to age-induced physiologic changes.
  12. A novice nurse is aware that normal aging can result in changes in the ECG of a 73-year-old patient. The experienced geriatric nurse explains that these changes may include which of the following? (Select all that apply.)
    a. An inverted T wave
    b. A notched P wave
    c. A prolonged PR interval
    d. Decreased amplitude of the QRS complex
    e. A slurred T wave
    B, C, D, E
    The number of pacemaker cells located in the sinoatrial node decreases with age, which results in less responsiveness of the cells to adrenergic stimulation. Common aging changes that are reflected by the electrocardiogram (ECG) include a notched P wave, a prolonged PR interval, decreased amplitude of the QRS complex, and a notched or slurred T wave.
  13. A nurse is planning to teach a senior citizens group heart-healthy lifestyle choices. Which should be included? (Select all that apply.)
    a. Smoking cessation tips
    b. Low-carbohydrate food choices
    c. Stress management techniques
    d. Regular blood pressure monitoring
    e. Strategies to include exercise into daily routine
    A, C, D, E
    Heart-healthy lifestyle changes concern smoking cessation, stress management, blood pressure control, exercise, weight loss, and a low-fat, low-sodium diet. Low-carbohydrate foods are not considered part of heart-healthy lifestyles.
  14. An older adult recovering from a myorcardial infarction (MI) has been taking subcutaneous heparin but is now to receive oral warfarin (Coumadin). The nurse prepares to teach the patient which topics? (Select all that apply.)
    a. Administration of both medications for up to 5 days
    b. Need to use a soft bristle toothbrush
    c. Use of atropine as an antidote for excessive bleeding
    d. Need to continue drawing partial thromboplastin times
    e. Need to drink at least eight cups of fluids daily
    A, B, D
    Heparin and warfarin (Coumadin) are anticoagulants used to prevent the enlargement of existing thrombi and new clot formation after an MI. Therapeutic effects of heparin are monitored by partial thromboplastin times; the antidote is protamine sulfate. Warfarin is monitored by the international normalized ratio (INR); the antidote is vitamin K. Patients who initially receive heparin for anticoagulation and who need oral anticoagulation for maintenance usually take both forms of medication for 3 to 5 days to develop therapeutic blood levels. Bleeding is a complication. Patients need to be taught bleeding precautions.
  15. A 77-year-old patient is being treated for cardiac arrhythmia. The nurse determines that the patient’s cardiac output is adequate with which assessments? (Select all that apply.)
    a. Urine output of 140 cc over 4 hours
    b. Systolic blood pressure that remains within 20 mm of baseline
    c. Denial of substernal pain
    d. Recollection of the birthdays of all of her grandchildren
    e. Absence of rales and crackles
    A, B, D, E
    The patient will maintain an adequate cardiac output, as evidenced by heart rate and rhythm within normal range, stable blood pressure, adequate peripheral pulses, mental alertness, urine output of 30 mL

hr, and clear breath sounds. Normal mentation also denotes good cardiac output, but the patient may have too many birthdays to remember, so this is not the best indicator of cognitive status.

  1. To minimize the possible complications of polypharmacy among older adult patients, the nurse assesses this population for which of the following? (Select all that apply.)
    a. Number of physicians providing medical care
    b. Location of pharmacies where prescriptions are filled
    c. Presence of chronic illnesses
    d. Tendency to borrow medication from family or friend
    e. Use of over-the-counter medication to self-medicate
    A, B, C, E
    Older adults are especially vulnerable to polypharmacy because many have one or more chronic conditions requiring several medications for management. To complicate matters, patients may see more than one provider for the same health problem and may have prescriptions filled at more than one pharmacy. Additional contributors to polypharmacy include the use of over-the-counter and alternative medicines or supplements in the treatment of conditions. As a result, the patient may end up taking duplicate drugs, similar drugs from the same drug class, and drugs that are contraindicated when taken together. Borrowing medications is not usually an issue.
  2. The nurse must be able to distinguish between alcohol intoxication and alcohol withdrawal to intervene appropriately. The nurse suspects alcohol intoxication when the patient does which of the following? (Select all that apply.)
    a. Slurs his speech when answering questions
    b. Has difficulty remembering his address
    c. Reports seeing snakes in the corner of the room
    d. Documents his blood pressure as 168/90
    e. Experiences difficulty when walking to the bathroom
    A, B, E
    Signs associated with alcohol intoxication include the scent of alcohol on the breath, slurred speech, lack of coordination, unsteady gait, nystagmus, impairment in attention or memory, and stupor or coma. Manifestations of alcohol withdrawal are elevated blood pressure, elevated pulse, and autonomic hyperactivity. In addition, fever; increased hand tremors; insomnia, nausea and vomiting; transient visual, tactile, or auditory hallucinations or illusions; psychomotor agitation; anxiety; and grand mal seizures may occur.
  3. A 69-year-old was prescribed a benzodiazepine 3 years ago. This medication regimen increases the patient’s risk for injury related to drug abuse and requires frequent patient assessment for which of the following? (Select all that apply.)
    a. Daytime sleepiness
    b. Unsteady gait
    c. Shortness of breath
    d. Easy bleeding
    e. Forgetfulness
    A, B, E
    Benzodiazepines can cause excessive sedation, impaired memory, decreased psychomotor performance, and balance disturbances and may lead to drug dependence and should not be prescribed for extended periods of time. Shortness of breath and bleeding are not signs of side effects.
  4. Adherence to prescribed health care treatments by a patient with a chronic disease is best facilitated when the nurse does which of the following? (Select all that apply.)
    a. Provides the patient with information regarding his disease
    b. Assesses the patient’s ability to understand his disease
    c. Defines “health and wellness” for the patient
    d. Helps the patient identify barriers to his personal wellness
    e. Coordinates support services to facilitate the patient’s discharge
    A, B, D, E
    The five As of a patient’s self-management of care includes assess, advise, agree, assist, and arrange.
  5. The student learning about chronic disease and illness in the older population learns which facts about this situation? (Select all that apply.)
    a. One in two adults, or more than 133,000 Americans, has a chronic condition.
    b. Chronic disease is the leading cause of death in those over 65.
    c. About 75% of medical costs each year are spent on managing chronic disease.
    d. Formerly acute conditions are now manageable chronic diseases..
    e. The focus of America’s health care services is now on chronic illness
    A, B, C, D
    One in two adults has a chronic illness, and these problems are the leading cause of death in those over 65 and the largest cost to our health care system. One reason for this is that formerly acute, possibly fatal, conditions are now manageable as chronic conditions. America’s health care system continues to be focused on acute care.
  6. The nurse understands what about the Americans with Disabilities Act? (Select all that apply.)
    a. It outlaws discrimination on the job because of disabilities.
    b. It requires state governments to fund disability services.
    c. It prohibits discrimination in government services to the disabled.
    d. It requires all buildings to be retrofitted to allow access.
    e. It provides funding for barrier-free buildings and parks.
    A, C
    The ADA outlawed discrimination on the basis of disability in employment, in programs and services provided by state and local governments, and in the provision of goods and services provided by private companies and commercial facilities. It does not mandate government payment for disability services, require buildings to be retrofitted, or provide funding for barrier-free facilities.
  7. In 2010, the revised Standards and Scope of Gerontological Nursing Practice was published. The nurse would use these standards to:
    a. promote the practice of gerontologic nursing within the acute care setting.
    b. define the concepts and dimensions of gerontologic nursing practice.
    c. elevate the practice of gerontologic nursing.
    d. incorporate suggested interventions from others who practice gerontologic nursing.
    D
    The current publishing of the Standards and Scope of Gerontological Nursing Practice in 2010 incorporates the input of gerontologic nurses from across the United States. It was not intended to promote gerontologic nursing practice within acute care settings, define concepts or dimensions of gerontologic nursing practice, or elevate the practice of gerontologic nursing.
  8. When attempting to minimize the effect of ageism on the practice of nursing older adults, a nurse needs to first:
    a. recognize that nurses must act as advocates for aging patients.
    b. accept that this population represents a substantial portion of those requiring nursing care.
    c. self-reflect and formulate one’s personal view of aging and the older patient.
    d. recognize ageism as a form of bigotry shared by many Americans.
    C
    Ageism is an ever-increasing prejudicial view of the effects of the aging process and of the older population as a whole. With nurses being members of a society holding such views, it is critical that the individual nurse self-reflect on personal feelings and determine whether such feelings will affect the nursing care that he or she provides to the aging patient. Acting as an advocate is an important nursing role in all settings. Simply accepting a fact does not help end ageism, nor does recognizing ageism as a form of bigotry.
  9. When discussing factors that have helped to increase the number of healthy, independent older Americans, the nurse includes the importance of:
    a. increased availability of in-home care services.
    b. government support of retired citizens.
    c. effective antibiotic therapies.
    d. the development of life-extending therapies.
    C
    The health and ultimate autonomy of older Americans has been positively impacted by the development of antibiotics, better sanitation, and vaccines. These public health measures have been more instrumental in increasing the numbers of healthy, independent older Americans than have in-home care services, government programs, or life-extending therapies.
  10. Based on current data, when presenting an older adult’s discharge teaching plan, the nurse includes the patient’s:
    a. nonrelated caretaker.
    b. paid caregiver.
    c. family member.
    d. intuitional representative.
    C
    Less than 4% of older adults live in a formal health care environment. The majority of the geriatric population lives at home or with family members.
  11. The nurse planning care for an older adult who has recently been diagnosed with rheumatoid arthritis views the priority criterion for continued independence to be the patient’s:
    a. age.
    b. financial status.
    c. gender.
    d. functional status.
    D
    Maintaining the functional status of older adults may avert the onset of physical frailty and cognitive impairment, two conditions that increase the likelihood of institutionalization.
  12. A nurse working with the older adult population is most likely to assess a need for a financial social service’s referral for a(n):
    a. white male.
    b. black female.
    c. Hispanic male.
    d. Asian American female.
    B
    The poverty rate among older black women is substantially higher than that seen among males or females of other ethnic groups. White males had the least poverty.
  13. Which of the following statements made by a nurse preparing to complete a health assessment and history on an older patient reflects an understanding of the general health status of this population?
    a. “I’ll need to document well regarding the medications the patient is currently prescribed.”
    b. “I would like to understand how supportive the patient’s family members are.”
    c. “Most older patients are being treated for a variety of chronic health care issues.”
    d. “It will be interesting to see whether this patient sees herself as being healthy.”
    D
    It is a misconception that old age is synonymous with disease and illness. The nurse should always determine the patient’s sense of wellness and independence when conducting a health and history assessment. An assessment of medication use and family support is important for any patient. Many older adults do have chronic health conditions, but their perception is more important than a single number.
  14. The nurse is caring for an older adult who has been admitted to an acute care hospital for treatment of a fractured femur. The family expresses concern about the patient’s pending transfer to a subacute care facility. What response by the nurse is best?
    a. “Acute care facilities lack the long-term physical therapy support your dad requires.”
    b. “Your dad will be much happier in a more serene, private environment.”
    c. “The subacute facility will focus on helping your dad maintain his independence.”
    d. “Insurance, including Medicare, will cover only a limited amount of time here.”
    C
    The transfer of the patient to a subacute facility is based on the need to maintain the patient’s level of function and independence, a task the acute care facility is not prepared to address once the patient is physiologically stable. The patient may or may not be happier in the new setting; the nurse should not make this judgment. It is true that insurance only pays for a limited amount of time in an acute care facility, but this is not the best reason for the patient to transfer.
  15. To best assure both the quality of care and the safety of the older adult patient who requires in-home unlicensed assistive personal (UAP) assistance, the geriatric nurse:
    a. evaluates the competency of the UAP staff.
    b. assumes the roles of case manager and patient advocate.
    c. arranges for the needed UAP provided services.
    d. assesses the patient for functional limitations.
    A
    As more care traditionally provided by professional nurses is being transferred to UAP, the nurse must assume more responsibility for educating, training, and evaluating the competency of UAP staff to provide safe, effective care for the older adult patient.
  16. The nurse working with older adults understands what information about certification in gerontologic nursing?
    a. It is mandatory for those in long-term care settings.
    b. It is voluntary and shows clinical expertise in an area.
    c. It allows nurses to be paid by third-party payers.
    d. It allows nurses to advance their careers in a job.
    B
    Certification is voluntary and shows that a nurse has additional knowledge and expertise in a certain area of practice. It is not mandatory in specific care settings. It does not allow for third-party reimbursement. It may be part of a career ladder program, but that is not true of all work settings.
  17. A nurse works in a gerontologic clinic. What action by the nurse takes highest priority?
    a. Serving as a patient advocate
    b. Educating patients about diseases
    c. Helping patients remain independent
    d. Referring patients to home health care
    C
    One of the challenges and priorities of the gerontologic nurse is helping patients maintain their independence.
  18. A nurse is caring for an older patient in the emergency department. What information about the patient will be most helpful in creating a plan of care?
    a. Baseline physical and cognitive functioning
    b. Living conditions and family support
    c. Medications and current medical problems
    d. Results of the Mini Mental State examination
    A
    The nurse is encouraged to view older patients as individuals and consider their baseline physical and cognitive functional status as a standard by which to compare the patient’s current status. The other information is also important, but the basis of individualized care begins with the patient’s strengths and weaknesses.
  19. The faculty member explains to students that many older Americans continue to work past the “retirement age.” What best explains this trend?
    a. Feeling healthier longer
    b. Changing financial outlook
    c. Becoming bored in retirement
    d. A desire to give back
    B
    As financial situations may have declined as a result of many economic factors, more older adults work past their “retirement age.” The other options may be reasons for some to continue working, but financial necessity is the reason the majority continue to do so.
  20. What information does the faculty member teach students about Medicare?
    a. Covers anyone with end stage renal disease
    b. Part A covers some prescription costs
    c. Part B covers inpatient hospital costs
    d. Part D eliminates the drug “donut hole”
    A
    Although Medicare is primarily for those over the age of 65, it does cover people of any age with end-stage kidney disease. Part A covers hospital costs. Part B is medical insurance. The “donut hole” was fixed by the Affordable Care Act.
  21. A nursing manager notes that many older patients are admitted to the nursing unit for acute problems. What action can the manager take to most benefit this population?
    a. Provide mandatory education on the needs of the older patient.
    b. Provide restorative therapy programs designed for this group.
    c. Ensure staffing numbers are adequate for dependent patients.
    d. Encourage all nurses to obtain gerontologic certification.
    B
    Many older adults need acute care for sudden illness and injury but live in a state of functional decline, which could possibly be prevented by establishing a restorative therapy program. The other actions will help the older patients cared for in the unit, but only to limited degrees.
  22. The dean of a new nursing program wishes to ensure graduates are prepared to care for older patients. What document should guide the dean in designing the curriculum?
    a. The Nurse Practice Act for that state
    b. The American Nurses Association (ANA) code of ethics for nurses
    c. Healthy People 2020
    d. The Recommended Baccalaureate Competencies and Curricular Guidelines
    D
    The Recommended Baccalaureate Competencies and Curricular Guidelines for the Nursing Care of Older Adults is an updated version of The Essentials of Baccalaureate Education for Professional Nursing Practice. This document was first published by the American Association of Colleges of Nursing (AACN) in 2008 and was updated in 2010. The other three documents do not have information about curricular requirements to prepare students to care for the older population.
  23. A nurse wants to plan a community event at a retirement center. What topic would most likely be best received?
    a. Heart healthy living
    b. Financial planning
    c. Avoiding scams
    d. Ethnic cooking classes
    A
    Older adults are demanding more programs and services aimed at health maintenance and promotion and disease and disability prevention. Based on this information, the heart healthy living presentation would be best received.
  24. What does the bedside nurse understand about his or her role in nursing research?
    a. Research is only done by doctorally prepared nurses.
    b. All nurses have a role in delivering research-based care.
    c. A bedside nurse can be part of a hospital research team.
    d. The bedside nurse can collect data if the nurse has been properly trained.
    B
    All nurses are charged to deliver patient-centered care based on evidence-based practice, research, quality improvement, and informatics. The bedside nurse is part of an interdisciplinary team that is responsible for redesigning the health care structure of the future.
  25. The clinic nurse caring for an older diabetic patient with a sixth grade education anticipates that the patient may experience difficulty (Select all that apply.)
    a. recognizing the importance of keeping clinic appointments.
    b. following a low-carbohydrate diet.
    c. paying for insulin and syringes.
    d. deciding on a primary health care provider.
    e. naming a health care surrogate.
    A, B, C
    Even though the educational level of the older population has steadily increased, as a population they are less educated than the general population. This deficiency can account for a lack of understanding regarding the need for medical care and the importance of following a treatment plan. These patients may also have fewer financial resources to devote to health care issues.
  26. The nurse studying the history of gerontologic nursing learns which information about the specialty? (Select all that apply.)
    a. The number of older Americans is diminishing.
    b. The geriatric nursing conference group was established in 1962.
    c. The gerontologic clinical nurse specialist certification was offered in 1989.
    d. There were no writings about the care of older persons until World War II.
    e. The first Standards of Practice for Geriatric Nursing was written in 1969.
    B, C, E
    The geriatric nursing conference group was established in 1962, the gerontologic clinical nurse specialist certification was first offered in 1989, and the first Standards of Practice for Geriatric Nursing was written in 1969. The population of older Americans is the fastest-growing subset of the population. Writings about care of the aged can be found from as early as 1900.
  27. The student asks the gerontologic clinic nurse why so many older people are women. What information does the nurse provide? (Select all that apply.)
    a. Reduced maternal mortality
    b. Decreased deaths from infectious diseases
    c. More deaths from chronic disease in men
    d. More deaths in war occur in men
    e. Women tend to smoke and drink less than men
    A, C, D
    A decrease in maternal mortality, decreased deaths from infectious diseases, and more chronic illness in men account for the disparity in genders as people age.
  28. The gerontologic nurse plans community programming for older women, noting what facts about this population subgroup? (Select all that apply.)
    a. More likely to live alone
    b. Increased chance of living in poverty
    c. Taking care of a spouse
    d. Suffering many chronic diseases
    e. Living with extended families
    A, B, D
    Older women have a greater chance than men of living alone and in poverty. They also have a greater degree of functional impairment and chronic disease.
  29. The nurse knows that the most common causes of death in the older population result from which diseases? (Select all that apply.)
    a. Cerebrovascular disease
    b. End-stage kidney disease
    c. Heart disease
    d. Cancer
    e. Diabetes
    A, C, D
    The most common causes of death in the older population are cerebrovascular disease, heart conditions, and cancer. End-stage renal disease and diabetes are not among the top three causes of death.
  30. The practitioner who believes in the free radical theory of aging is likely to recommend that the older adult:
    a. avoid excessive intake of zinc or magnesium.
    b. supplement his or her diet with vitamins C and E.
    c. increase intake of complex carbohydrates.
    d. avoid the use of alcohol or tobacco.
    B
    Vitamins C and E are two naturally occurring antioxidants that appear to inhibit the functioning of the free radicals or possibly decrease their production in the body. The free radical theory of aging is not related to zinc, magnesium, carbohydrates, or alcohol and tobacco.
  31. The practitioner who believes in the free radical theory of aging is likely to recommend that the older adult:
    a. avoid excessive intake of zinc or magnesium.
    b. supplement his or her diet with vitamins C and E.
    c. increase intake of complex carbohydrates.
    d. avoid the use of alcohol or tobacco.
    B
    Vitamins C and E are two naturally occurring antioxidants that appear to inhibit the functioning of the free radicals or possibly decrease their production in the body. The free radical theory of aging is not related to zinc, magnesium, carbohydrates, or alcohol and tobacco.

DIF: Applying (Application) REF: N/A OBJ: 2-2

  1. To provide effective care to the older adult, the nurse must understand that:
    a. older adults are not a homogeneous sociologic group.
    b. little variation exists in cohort groups of older adults.
    c. health problems are much the same for similar age groups of older adults.
    d. withdrawal by an older adult is a normal physiologic response to aging.
    A
    The key societal issue addressed by the age stratification theory is the concept of interdependence between the aging person and society at large. This theory views the aging person as an individual element of society and also as a member, with peers, interacting in a social process. The theory attempts to explain the interdependence between older adults and society and how they constantly influence each other in a variety of ways. Variation exists among the members of a cohort. Health problems are not the same for every individual of the same age. Withdrawal by an older adult is not a normal response to aging but may be a sign of depression.

DIF: Applying (Application) REF: N/A OBJ: 2-2

  1. The nurse is using the eight stages of life theory to help an older adult patient assess the developmental stage of personal ego differentiation. The nurse does this by assisting the patient to:
    a. determine feelings regarding the effects of aging on the physical being.
    b. describe feelings regarding what he or she expects the future to hold.
    c. identify aspects of work, recreation, and family life that provide a sense of self-worth and pleasure.
    d. elaborate on feelings about the prospect of his or her personal death.
    C
    During the stage of ego differentiation versus work role preoccupation, the task for older adults is to achieve identity and feelings of worth from sources other than the work role. The onset of retirement and termination of the work role may reduce feelings of self-worth. In contrast, a person with a well-differentiated ego, who is defined by many dimensions, can replace the work role as the major defining source for self-esteem. Determining feelings related to the effects of aging, future death, or what the future may hold is not part of this theory.

DIF: Applying (Application) REF: N/A OBJ: 2-2

  1. A patient is recovering from a mild cerebral vascular accident (stroke). The home care nurse notes that the patient is talking about updating a will and planning funeral arrangements. Which of the following responses is most appropriate for the nurse to make?
    a. “You seem to be preoccupied with dying.”
    b. “Is there anything I can do to help you?”
    c. “Are you worried about dying before you get your affairs in order?”
    d. “Let’s focus on how you are recovering rather than on your dying.”
    B
    According to Peck’s expansion of Erikson’s theory, the older adult who has successfully achieved ego integrity and ego transcendence accepts death with a sense of satisfaction regarding the life led and without dwelling on its inevitability. The patient’s action reflects a healthy transition and should be supported.

DIF: Applying (Application) REF: N/A OBJ: 2-2

  1. Your patient’s spouse died recently from a sudden illness after 45 years of marriage. The patient was the primary caregiver for the spouse during this time. The patient is now depressed and withdrawn and has verbalized feelings of uselessness. Which action by the nurse is best?
    a. Encourage the patient take up a hobby that will occupy some time.
    b. Explain that volunteering would be an excellent way to stay useful.
    c. Assure the patient that these feelings of sadness will pass with time.
    d. Ask the patient to share some cherished memories of the spouse.
    B
    Volunteering will help the patient to interact with people and feel productive and valued for the ability to help others as stated in the activity theory. A hobby does not offer the chance to help others. Assuring the patient that feelings will pass is false reassurance and does nothing to help the patient to be proactive. Reminiscing is a valued activity, but it is not the best choice for regaining a sense of usefulness.

DIF: Applying (Application) REF: N/A OBJ: 2-4

  1. A patient has recently been diagnosed with end-stage renal disease. The patient has cried often throughout the day and finally confides in the nurse that “I am going home to be with my Lord.” The nurse’s best response is:
    a. “There is no reason to believe the end is near.”
    b. “Do you want me to call your family?”
    c. “We have a wonderful chaplain if you’d like me to call him.”
    d. “I think this is the time for us to pray together.”
    C
    It is important for the nurse to acknowledge the spiritual dimension of a person and support spiritual expression and growth while addressing spirituality as a component in holistic care without imposing upon the patient. Because the patient has made reference to the Lord, the nurse can safely offer religion-oriented spiritual care. Telling the patient there is no reason to believe that death is near does not help the patient work through emotions. Asking about calling the family is a yes/no question and is not therapeutic. The nurse is assuming too much by saying it is time to pray.

DIF: Applying (Application) REF: N/A OBJ: 2-2

  1. A nurse is responsible for the care of 20 older adults in a unit of an assisted living facility. In order to best address the needs and wants of the entire unit’s population, the nurse:
    a. strictly adheres to facility policies so that all patients will be treated equally.
    b. encourages specific age cohorts to gather in the dayroom because they share similar interests.
    c. has the unit vote on which television programs will be watched each evening.
    d. schedules the patients’ bathing times according to their individual preferences.
    D
    Older adults continue to feel valued and viewed as active members of society when allowed to maintain a sense of control over their living environment by attention to personal choices and rituals. Adhering strictly to policies does not allow for individualized care. Not all in the same age cohort will have similar interests. Voting on television programs does not ensure each individual feels a sense of worth.

DIF: Applying (Application) REF: N/A OBJ: 2-2

  1. An older patient who reports being “healthy enough to cut my own fire wood” is being assessed prior to outpatient surgery. The nurse recognizes which assessment observation as a possible result of the wear-and-tear theory?
    a. Swollen finger joints
    b. Red, watery eyes
    c. Grimacing when raising left arm
    d. Bilaterally bruising on the forearms
    C
    This theory proposes that cells wear out over time because of continued use. The pain caused by movement of the shoulder is the observation most likely a result of the patient’s practice of cutting his own firewood. The other choices do not demonstrate continued use that is part of the wear-and-tear theory of aging.

DIF: Applying (Application) REF: N/A OBJ: 2-2

  1. A nurse cares for many older patients. Which finding should the nurse identify as pathologic in a 72-year-old?
    a. Two hospitalizations in 6 months for respiratory infections
    b. Patient reports of sleeping only of 5 to 6 hours each night
    c. Thinning hair and brittle nails
    d. Dry, tissue paper-like skin
    A
    Although there is an age-related decrease in immune function, reoccurring infections serious enough to require hospitalization are not considered a normal age-related finding. Decreased sleeping, thinning hair, brittle nails, and dry skin are all normal signs of aging.

DIF: Applying (Application) REF: N/A OBJ: 2-2

  1. In planning the care for an older adult patient, the nurse will best promote health and wellness by:
    a. encouraging independent living and self-care.
    b. scheduling regular cardiac and respiratory health screenings.
    c. effectively delivering health-related educational information.
    d. promoting a nutritious diet and an age-appropriate exercise routine.
    C
    Providing well-prepared and effectively delivered health-related educational information will provide the best means of promoting a patient’s ability to impact his or her wellness and general health. Each of the other options is too narrow to be the most effective way to promote health and wellness.

DIF: Applying (Application) REF: N/A OBJ: 2-2

  1. The student learning about aging theories understands that the main difference between stochastic theories and nonstochastic theories is which of the following?
    a. Stochastic theories view aging as a random, cumulative process.
    b. Stochastic theories view aging as similar among all people.
    c. Nonstochastic theories view aging as a result of psychosocial factors.
    d. Nonstochastic theories are backed by research, whereas stochastic theories are not.
    A
    Stochastic theories view aging as a result of random events and their cumulative effects. Nonstochastic theories view aging as a result of predetermined, timed phenomena. Both are types of biologic theories.

DIF: Applying (Application) REF: N/A OBJ: 2-2

  1. Which theory of aging does the student learn is related to problems with DNA transcription?
    a. Radical theory
    b. Error theory
    c. Cross linkage theory
    d. Wear-and-tear theory
    B
    The error theory poses that errors in DNA transcription lead to aging. The radical theory views the effects of free radicals as critical to aging. The cross linkage theory states that normally separated molecular structures are bound together through chemical reactions and that this interferes with metabolic processes. The wear-and-tear theory postulates that normal activity causes wear and tear on the body, leading to aging.

DIF: Applying (Application) REF: N/A OBJ: 2-2

  1. According to which theory does cancer occur as a possible result of aging?
    a. Radical theory
    b. Error theory
    c. Immunity theory
    d. Pacemaker theory
    C
    Immunosenescence is the term used in immunity theory to describe an age-related decrease in immune functioning. According to this theory, as people age, they are more prone to getting cancer or autoimmune diseases because of this phenomenon. This is a nonstochastic theory. Radical and error theories are both stochastic. The pacemaker theory looks at the interrelated role of the neurologic and endocrine systems and aging.

DIF: Applying (Application) REF: N/A OBJ: 2-2

  1. A nurse assesses an older patient who has lost a great deal of weight in a short time. When asked, the patient states this behavior started after the patient read a magazine article on the benefits of extreme caloric restrictions. What response by the nurse is best?
    a. “That research was done on rodents and not on humans.”
    b. “You shouldn’t restrict your calories so severely.”
    c. “You have lost so much weight you need dietary supplements.”
    d. “You can’t believe everything you read in those magazines.”
    A
    The metabolic theory of aging postulates that organisms have a specific metabolic lifetime and that by lowering metabolic rate, life span can be increased. However, this has been demonstrated in rodents and the nurse should educate the patient on this information. The other options do not give information that will help the patient make an informed decision as to whether or not to follow this activity.

DIF: Applying (Application) REF: N/A OBJ: 2-2

  1. A nurse is caring for an older patient who is sedentary and does not want to participate in any activities. What action by the nurse is best?
    a. Inform the patient about the consequences of immobility.
    b. Promote activity by explaining the “use it or lose it” concept.
    c. Tell the patient he or she will feel better by being more active.
    d. Explain the relationship of being active and being independent.
    D
    Activity increases circulation, provides range of motion, and leads to clearer mental functioning. Activity helps a person remain independent and able to perform activities of daily living (ADLs) and instrumental ADLs. Presenting information in a positive light that encourages the patient to take control of one’s own health is more likely to be successful than stressing the negative such as consequences of immobility or the concept of “use it or lose it.” Telling the patient that he or she will feel better does not give concrete information the patient can use to make decisions.

DIF: Applying (Application) REF: N/A OBJ: 2-2

  1. The nurse working with older patients teaches the student that disengagement theory potentially causes which problem?
    a. Fear
    b. Isolation
    c. Anxiety
    d. Malnutrition
    B
    The no-longer supported disengagement theory posed that older people withdrew from society as they aged and that this was a mutually agreed upon behavior. The result would be isolation as the person became focused solely on him- or herself. Fear, anxiety, or malnutrition could be a further consequence, but isolation and withdrawal from society was “expected” according to this theory.

DIF: Applying (Application) REF: N/A OBJ: 2-2

  1. The nurse working in a long-term care facility used the developmental theory of aging in practice. In caring for a frail, nearly bed bound patient, how can the nurse use this theory?
    a. Engage the patient in intellectually stimulating activities.
    b. Encourage the patient to participate in chair exercises.
    c. Ensure that the patient participates in all the group activities.
    d. Give the patient small “chores” to do for the facility.
    A
    In this theory, being active can mean physical or intellectual activity. The nurse can engage the patient in intellectually stimulating activities that allows the person a sense of satisfaction. The other options all call for physical activity, which the patient may or may not be able to perform.

DIF: Applying (Application) REF: N/A OBJ: 2-2

  1. The nurse working at a long-term care facility notes that one patient who is usually outgoing refuses to participate in games that require keeping score. What action by the nurse is best?
    a. Ask the patient why he or she won’t participate.
    b. Assess the patient’s level of frustration with these activities.
    c. Find other activities for the patient to participate in.
    d. Do nothing; the patient can choose activities to engage in.
    B
    Although it is true that patients should be able to choose activities in which to participate, the best option is to assess the patient for frustration or anxiety associated with these types of activities. Once that is determined, the nurse can find other activities the patient can engage in successfully and is willing to participate in if the games are not an option. Asking “why” questions often puts people on the defensive and is not a therapeutic communication technique.

DIF: Applying (Application) REF: N/A OBJ: 2-2

  1. The nurse planning community events for older people uses sociologic theories to guide practice. Which activity planned by the nurse best fits these theories?
    a. Group exercise programs
    b. Volunteering at a day care
    c. Healthy cooking classes
    d. Reminiscing therapy
    B
    Using the sociologic theories to guide care, the nurse would plan events that allowed the older adult to remain active in the community and a valued member of society. Volunteering would offer the adult a way to stay engaged and provide a service to successive generations. Exercise and cooking classes would more fit in the biologic theories. Reminiscing therapy is a technique using psychological theories. All are good ideas for activities, but the one that specifically uses sociologic theory is the volunteer work.

DIF: Applying (Application) REF: N/A OBJ: 2-2

  1. What statement by a patient most indicates healthy aging according to Jung?
    a. “I wish I had traveled more when I was younger because now I can’t.”
    b. “I am proud of my past accomplishments at work and home raising my kids.”
    c. “My leg amputation makes things harder, but I still find a way to work.”
    d. “I still like to read the paper and novels and enjoy a little gardening.”
    C
    This patient shows acceptance of past accomplishments and finds value in him- or herself despite current limitations, which is healthy aging according to Jung. The person who wants to travel more displays remorse. The focus on past accomplishments does not show current acceptance. Reading and gardening do not show acceptance of past accomplishments.

DIF: Applying (Application) REF: N/A OBJ: 2-2

  1. A nurse is trying to teach a hospitalized older patient how to self-inject insulin. The patient is restless and does not seem to be paying attention. What action by the nurse is best?
    a. Ask if the patient needs to use the bathroom.
    b. Tell the patient you’ll try again later in the day.
    c. Ask if the patient prefers that you teach the family.
    d. Refer the patient for home health care services.
    A
    According to Maslow, physical needs take priority over other activities. This patient may be hungry, cold, tired, or need to use the bathroom. Telling the patient you’ll try again later, asking if you should teach the family, and referring to home health care does not provide for any unmet physical needs.

DIF: Applying (Application) REF: N/A OBJ: 2-2

  1. The new nurse at a long-term care center asks the director of nursing why he needs to learn so many theories of aging. What response by the director is best?
    a. “No theories have been proven yet.”
    b. “A wide range of theories allows for holistic care.”
    c. “It’s required knowledge for certification exams.”
    d. “All the theories are important, so we use them all.”
    B
    Using a combination of different theories, each with its own focus, allows the nurse to plan individualized, holistic nursing care.

DIF: Applying (Application) REF: N/A OBJ: 2-2

  1. According to Maslow, a fully actualized person displays which traits? (Select all that apply.)
    a. Spontaneity
    b. Self-direction
    c. Creativity
    d. Ethical conduct
    e. Acceptance of self
    A, B, C, E
    A fully actualized person displays the following characteristics: perception of reality; acceptance of self, others, and nature; spontaneity; problem-solving ability; self-direction; detachment and the desire for privacy; freshness of peak experiences; identification with other human beings; satisfying and changing relationships with other people; a democratic character structure; creativity; and a sense of values. Maslow does not specify ethical conduct.

DIF: Applying (Application) REF: N/A OBJ: 2-2

  1. A nurse caring for older adult patients shows an understanding of the implementation of standards of care when:
    a. dialing the telephone when the patient wants to call his daughter.
    b. requesting the patient’s favorite dessert on his birthday.
    c. closing the patient’s door when he is praying.
    d. reminding the patient to call for assistance before getting out of bed.
    D
    A standard of care is a guideline for nursing practice and establishes an expectation for the nurse to provide safe and appropriate care, such as reminding the patient to call for assistance before getting out of bed. Standards of care may be established on national or regional levels. Dialing the phone for the patient, closing the patient’s door, and requesting a special dessert are not actions that conform to standards of care.

DIF: Applying (Application) REF: N/A OBJ: 3-1

  1. A nurse new to geriatric nursing asks the nurse manager to clarify how to handle a patient’s claim that she has been physically abused. The nurse manager responds most appropriately when stating:
    a. “I’ll show you where you can find this state’s reporting requirements.”
    b. “As a nurse you are considered a ‘mandated reporter’ of elder abuse.”
    c. “As long as you are reasonably sure abuse has occurred, report it.”
    d. “You need to report any such claims directly to me.”
    A
    To be responsive to the legal obligation to report reasonably suspicious acts of abuse and because there is great variation among the states, nurses should determine the specific reporting requirements of their jurisdictions, including where reports and complaints are received and in what form they must be made. The statements that the nurse is a mandatory reporter and that abuse should be reported if suspected are true, but they do not help the nurse learn to handle the complaint. The manager may want to know about claims of abuse and it may be facility policy to report up the chain of command, but the nurse is responsible for filing the formal complaint.
  2. The nurse recognizes that a nursing aide likely to abuse an older patient is one who has:
    a. ineffective verbal communication skills.
    b. little experience working with the older population.
    c. poor stress management skills.
    d. been a victim of abuse.
    C
    It has been shown that the primary abusers of nursing facility residents are nurse aides and orderlies who have never received training in stress management.
  3. An older adult resident of a long-term care nursing facility frequently attempts to get out of bed and is at risk of sustaining an injury. The nurse’s planned intervention to minimize the patient’s risk for injury is guided by:
    a. the patient’s right to self-determination and to be free to get out of bed.
    b. an understanding that nondrug interventions must be tried before medications.
    c. the knowledge that application of a vest restraint requires a physician’s order.
    d. the patient’s cognitive ability to understand and follow directions.
    B
    The drug use guidelines are based on the principles that certain problems can be handled with nondrug interventions and that such forms of treatment must be ruled out before drug therapy is initiated. The patient does have the right to self-determination, but the staff must ensure the patient’s safety. Vest restraints do require an order, but environmental measures must be tried before chemical or physical restraints. The patient’s cognitive abilities do not allow for unjustified physical or chemical restraints.
  4. During the state inspection of a skilled nursing facility, a surveyor notes suspicion that a particular nurse may not be providing the proper standard of care. The nurse manager informs the nurse to expect:
    a. a review of the situation by the state board of nursing.
    b. termination of employment from the facility.
    c. mandatory remediation related to the suspect care issues.
    d. unannounced reevaluation of performance within the next 3 months.
    A
    In such cases, the surveyor may forward the record showing the relevant findings to the appropriate state agency or board for review of the nurse’s practice, requesting a determination of whether the nurse may have violated the state’s nurse practice act. Regulations do not specify that the nurse be terminated, have remediation, or have an unannounced reevaluation.
  5. An 87-year-old patient is unsure of the purpose of a living will. The nurse describes its purpose best when stating:
    a. “It’s a legal document that Social Services can help you create.”
    b. “It designates a family member to make decisions if you become incompetent.”
    c. “It provides a written description of your wishes in the event you become terminally ill.”
    d. “It assures you won’t be subjected to treatments you don’t want.”
    C
    Living wills are intended to provide written expressions of a patient’s wishes regarding the use of medical treatments in the event of a terminal illness or condition.
  6. The nurse is caring for an unresponsive patient who has terminal cancer with a Do Not Resuscitate order in effect. A family member tells the nurse, “I’ll sue you and every other nurse here if you don’t do everything possible to keep her alive.” The nurse understands that protection from legal prosecution in this situation is provided by:
    a. legal immunity granted when acting according to the patient’s expressed wishes.
    b. the legal view that the duty to put into effect the patient’s wishes falls to the physician.
    c. knowledge of and compliance with facility policies and procedures regarding end-of-life care.
    d. implementing interventions that preserve the patient’s right to self-determination.
    C
    In this case, immunity applies only to the physician and not to the nurse because the physician is given the legal duty to put into effect the patient’s wishes. Consequently, the nurse must rely on effective communication with the physician, patient, and family, and on the quality of the facility’s policies and procedures, to be sure that his or her actions are consistent with the legally required steps.
  7. The nurse is caring for a terminally ill older patient who has a living will that excludes pulmonary and cardiac resuscitation. The family expresses a concern that the patient may “change her mind.” The nurse best reassures the family by stating:
    a. “The nursing staff will watch her very closely for any indication she has changed her mind.”
    b. “We will discuss her wishes with her regularly.”
    c. “She can change her mind about any provision in the document at any time.”
    d. “Your mother was very clear about her wishes when she signed the document.”
    A
    AMD provisions appropriately provide that people can change their minds at any time and by any means. Nurses need to be alert to any indications from a patient. Based on the person’s medical condition, subtle signs such as a gesture or a nod of the head may be easily overlooked. The patient may or may not be able to discuss her condition. Stating that the mother was very clear in her wishes does not take into account the fact that patients can change their minds any time.
  8. A patient residing in a long-term care facility has been experiencing restlessness and has often been found by nursing staff wandering in and out of other patients’ rooms during the night. The nurse views the patient’s PRN antipsychotic medication order as:
    a. an appropriate intervention to help assure his safety.
    b. an option to be used only when all other nondrug interventions prove ineffective.
    c. inappropriate unless the physician is notified and approves its use.
    d. not an option because it should not be used to manage behaviors of this type.
    D
    Reasons for the use of antipsychotic drugs do not include behaviors such as restlessness, insomnia, yelling or screaming, inability to manage the resident, or wandering. The staff must provide nondrug alternatives to help calm the patient.
  9. An alert but disoriented older patient lives with family members. The home health nurse, being aware of the role of patient advocate, recognizes the obligation to report possible patient abuse based on:
    a. a family member stating, “It’s hard being a caregiver.”
    b. assessment showing bruises in the genital area.
    c. observation of mild changes in orientation.
    d. patient’s report of always being hungry.
    B
    Even when a patient exhibits disorientation, any report of mistreatment or neglect is to be considered reasonably suspicious and so should be reported. Bruises in the genital area raise suspicions of abuse. The family stating caregiving is hard does not mean they don’t have enough support to cope. Mild changes in orientation may be expected in a disoriented patient. The patient who is always hungry should be followed up with a nutrition assessment, and this may or may not be a sign of abuse.
  10. An older adult patient has been approached to participate in a research study. The nurse best advocates for the patient’s right of self-determination by:
    a. evaluating the patient’s cognitive ability to understand the consequence of the study.
    b. determining what risks to the patient are involved.
    c. discussing the importance of the study with the patient and his family.
    d. encouraging the patient to discuss the decision with trusted family or friends.
    A
    The right to self-determination has its basis in the doctrine of informed consent. Informed consent is the process by which competent individuals are provided with information that enables them to make a reasonable decision about any treatment or intervention that is to be performed on them. The other options do not address autonomy and self-determination.
  11. A nurse responsible for the care of older adult patients shows the best understanding of the nursing standards of practice when basing nursing care on the:
    a. physician’s medical orders.
    b. stated requests of the individual patient.
    c. care that a responsible geriatric nurse would provide.
    d. implementation of the nursing process.
    C
    Nursing standards of practice are measured according to the expected level of professional practice of those in similar roles and clinical fields. Nursing care is not judged against the physician’s orders, stated requests of the patient, or implementation of the nursing process.
  12. The nurse caring for an older patient who resides in an assisted living facility is asked to obtain and witness the patient’s signature on a living will document. The nurse responds most appropriately when stating:
    a. “I will, because such a document is so valuable to the patient’s plan of care.”
    b. “I’ll ask the patient’s family if they agree that the patient should sign the document.”
    c. “First I need to discuss the purpose of this document with the patient.”
    d. “I’m sorry but I cannot ethically do that.”
    D
    It is not permissible for the nurse to secure the patient’s signature or to witness the patient’s signature on a living will document. Generally speaking, an employee or owner of a facility in which the patient resides cannot witness this document.
  13. A graduate nurse learns about the provisions of the Health Insurance Portability and Accountability Act (HIPAA), which include which of the following?
    a. Requires employers to offer health care insurance
    b. Regulates the amount employers can charge for insurance
    c. Mandates that employers provide specific benefits
    d. Helps maintain coverage when a person changes jobs
    D
    HIPAA has several provisions, one of which is that it helps people maintain health care insurance when they are changing jobs. The other statements are common misconceptions about HIPAA.
  14. The nurse manager in a long-term care facility reviews resident care plans at what interval?
    a. Quarterly
    b. Every 60 days
    c. Annually
    d. When changes occur
    A
    The resident care plan is routinely reviewed quarterly.
  15. The manager of a long-term care facility is evaluating patients’ use of drugs. The resident on which of the following medications would be allowed to continue taking medications to control behavior?
    a. On anxiolytics; now able to participate in group activities
    b. Given a benzodiazepine at night; roommate now sleeps well
    c. Given sedatives; eats 100% of meals if resident is fed
    d. Taking an antipsychotic; no longer wanders at night
    A
    Drugs should not be used to control behavior. If used to manage health conditions, the patient should show improvement. The patient who is now able to participate in activities shows an increase in functional ability, so this medication is therapeutic for this patient. The other patients are given drugs to control behavior.
  16. To meet current guidelines regarding incontinence in a long-term care facility, what action by the director of nursing is best?
    a. Assess residents for the ability to participate in a bladder training program.
    b. Take all residents to the toilet every 2 hours and after meals.
    c. Ensure all residents wear incontinence briefs, which are changed routinely.
    d. Ask physicians and other providers to prescribe medications for bladder control.
    A
    Urinary incontinence is a common problem that can lead to several complications. The extent to which residents participate in bladder training programs is an area of focus for facility inspectors. Some residents may need routine toileting, wearing briefs, and medications, but they should all be assessed for the ability to participate in bladder training.
  17. The director of nursing at a long-term care facility is getting ready for the annual inspection. What information guides the director?
    a. Visits cannot be unannounced.
    b. The director must be off site during the inspection.
    c. Nurses must answer questions from the inspectors.
    d. Results will be shared only through the mail.
    C
    Nurses present during inspections must answer questions posed by the inspectors. Visits can be unannounced. The director should be present during the survey. Results are shared during a conference, then a report is mailed later.
  18. The nursing student learns about the Patient Self-Determination Act. What is a key provision of this act?
    a. It establishes new rights for patients in medical facilities.
    b. It requires facilities to educate patients on their rights.
    c. It allows families to be approached for organ donation.
    d. It spells out the procedures for creating an advance directive.
    B
    The intent of this law is to ensure that patients are given information about the extent to which their rights are protected under state law. It does not establish new rights, is not related to organ donation, and does not specify procedures for advance directives.
  19. To best address the patient’s right to self-determination, which of the follow questions does the nurse ask at the time the patient is admitted to a nursing facility? (Select all that apply.)
    a. “Do you understand what a living will and durable power of attorney are?”
    b. “If you have already prepared an advance care directive, can you provide it now?”
    c. “Are you prepared to discuss your end-of-life choices with the nursing staff?”
    d. “Have you discussed your end-of-life choices with your family or designated surrogate?”
    e. “Would you like help with preparing a living will or a durable power of attorney?”
    A, B, D, E
    All the correct options address the patient’s right to make an informed decision regarding health care issues by using various advance directives. The patient does not need to discuss end-of-life choices with the staff in order to exercise the right to self-determination.
  20. What provisions for nursing service are part of the Omnibus Budget Reconciliation Act (OBRA) as it pertains to long-term care facilities? (Select all that apply.)
    a. Resident assessments
    b. Annual screenings
    c. Minimum staffing
    d. Ensuring resident rights
    e. Registered nurse educational requirements
    A, B, C, D
    OBRA’s service requirements include resident assessments and screenings, minimum staffing requirements, and ensuring resident rights. Educational requirements for nurses are not part of this mandate.
  21. The director of nursing at a certified long-term care facility overhauls the nursing assistant training program to include which features? (Select all that apply.)
    a. 12 hours of classroom content
    b. Training in infection control measures
    c. Instruction on resident rights
    d. 6 hours of quarterly in-service education
    e. Education on safety measures
    B, C, D, E
    Requirements for a nursing assistant’s education includes training in infection control and interpersonal skills, instruction on resident rights and safety procedures, and 6 hours of education through in-services quarterly. Nursing assistants must have classroom training before working with residents, but the amount of time is not specified.
  22. The adult child of a long-term care facility resident receives a phone call from the director of nursing stating that her parent has 30 days to move out of the home. Under what conditions can a facility require a resident to move? (Select all that apply.)
    a. Nonpayment for services received
    b. Needs exceeding what the facility can provide
    c. Stay is no longer required based on the resident’s medical condition
    d. Facility is going out of business
    e. Frequent disruptive behavior during the night
    A, B, C, D
    A facility can require a resident to leave in four situations: nonpayment for services, needs that exceed what the facility can provide, the patient’s medical condition no longer warrants long-term care, or the facility is going out of business. Being disruptive is not a cause for expelling a resident.

Leave a Comment

Scroll to Top